.
.
Math Central - mathcentral.uregina.ca
Quandaries & Queries
Q & Q
. .
topic card  

Topic:

of

list of
topics
. .
start over

1278 items are filed under this topic.
 
Page
1/1
Finding the area of a regular polygon 2007-05-04
From Dana:
We are trying to figure the square footage of a tetradecagon....sort of a round house with 14 sides that are 8 ft' in length. It has a height of 9 ft. How do we figure the square footage of this?
Answered by Stephen La Rocque.
Strings of letters 2022-04-25
From Javar:
How many different combinations can I make with A 12 selection list but only using A&B

Example:

A,B,A,B,A,B,A,B,A,B,A,B
A,B,B,A,A,B,B,A,A,B,B,A
B,B,A,A,B,A,B,A,B,A,A,B

Answered by penny Nom.
Roll of film 2021-11-20
From Brian:
Trying to create a formula to calculate the build up on a roll of film is calculated. Except instead of of a roll, it's essentially a rectangle with two semicircles on each at the end. Example: I start with a 6" diameter core on each side, and I start winding .005" thick film on the core. The distance between the center of those cores, length of the inner rectangle is 7.125. If I continue to do this for a total length of film of 3000 feet, what will the roll length and width be. So what I need is the formula to perform this type of calculation. Can you help me? I want to be able to plug the formula in a spread sheet and to be able to input a core diameter, a film thickness and a total length and the length between the center of the two cores(which is the inner rectangle's length I'm assuming). Thanks
Answered by Harley Weston.
30 % off 2021-09-07
From Corrine:
When I am shopping for clothing that is for sale, the advertisement may say the item is 30% off. The original price was $70. How much would it now be? (how do I figure that out?) Thank you!
Answered by Penny Nom.
The units digit of a square 2021-09-04
From Aarzu:
Sorry this may be a dumb question but how do you find possible unit digits of a perfect square
Answered by Penny Nom.
The diameter of an octagon 2021-05-22
From Sue:
What is the diameter of an octagon with 8 foot sides
Answered by Penny Nom.
The height of a cone 2021-05-13
From Carmen:
If a cone has a slant height of 25 inches and a radius of 7 inches what is the height in inches of the cone?
Answered by Harley Weston.
More on a roll of paper 2021-05-07
From Stephen:
Can you help with the equations to calculate the length of paper required to achieve a target outside diameter when wrapped around a core please?

The inside diameter of the cardboard core is 76mm
The thickness of the wall of the cardboard core is 5mm
The thickness of the paper is 138microns
The desired outside diameter of the finished roll is 320mm

I hope you can help me with a solution, Thank you

Answered by Harley Weston.
The maximum area of a garden 2021-04-28
From Lexie:
suppose you want to make a rectangular garden with the perimeter of 24 meters. What's the greatest the area could be and what are the dimensions?
Answered by Penny Nom.
The volume of a spherical bowl 2021-04-23
From Danny:
How many gallons of water in a glass bowl with the dimensions of 10 inches high and 13.5 inches wide?
Answered by Penny Nom.
The distance between opposite vertices in an octagon 2020-11-14
From joe:
If the dimension from one flat part of an octagon to opposite flat is 500 mm what is the largest diameter (point to point) of the octagon?
Answered by Harley Weston.
An analytic proof that a quadrilateral is a parallelogram 2020-10-26
From Apollo:
Prove analytically that if ABCD is a parallelogram in which points P and Q trisects the diagonal AC, then BPDQ is a parallelogram.
Answered by Penny Nom.
A confusing problem 2020-10-01
From Kgaugelo:
a triangle has the vertices A(-4;-3),B(x;y)and C(2;1). Determine the coordinates of B if the equation of the line BC is y=5x-9
Answered by Harley Weston.
The height of a tree 2020-09-27
From Kerissa:
A man height 1.2 m standing on top of a building 34.6m high views a tree some distance away he observes that the angle of depression of the base of the tree is 35 and the angle of depression of the top of the tree is 29, assuming that the tree stands of level ground. Determine the height of the tree.
Answered by Penny Nom.
my prcentage of years on earth is 2020-09-21
From Bill:
This Tuesday is my birthday and I will be 69. and I just wanted to know life ear is say earth is 6 billion years old I'm just curious to what percentage does my existence on earth reptresent I do realize that it's a flash in the pan
Answered by Penny Nom.
The surface area of a dome 2020-09-13
From Sue:
i need to know the outside square footage of a dome house 24ft across at the bottom and 16ft high so i know how much paint it will take to cover it.
Answered by Harley Weston.
The angular speed of a wheel 2020-09-03
From Catrina:
A car is moving at a rate of 75 miles per hour, and the diameter of its wheels is 2.6 in. Find the angular speed of the wheels in radians per minute.
Answered by Penny Nom.
A diameter of a circle 2020-08-19
From Apple:
One end of the the diameter of the circle with equation x^2+y^2-4x+4y=2 is (3,1). Find the coordinates of the other end of the diameter.
Answered by Penny Nom.
A tank, 5/9 full of water 2020-07-28
From Nat:
A rectangular tank contains 1200cmsquare of water when it is 5 out of 9 full . Its base area is 80cmsquare . Find the height of the tank.
Answered by Penny Nom.
Percentage profit 2020-05-30
From jaheem:
A shopkeeper buys an article for 80cents and sold it for $1. Calculate the percentage profit.
Answered by Penny Nom.
A triangular garden 2020-05-24
From yoonji:
the 3 sides of a triangular garden measure 200 ft., 250 ft., and 300 ft., respectively. the garden is to be divided by a line bisecting the longest side and drawn from the opposite vertex. what is the length of the bisecting line?
Answered by Penny Nom.
Maximizing the volume of a cone 2020-05-18
From Ella:
Hello, this is question - 'If you take a circle with a radius of 42cm and cut a sector from it, the remaining shape can be curled around to form a cone. Find the sector angle that produces the maximum volume for the cone made from your circle.'
Answered by Penny Nom.
An angle i a triangle 2020-05-16
From Ogunjobi:
Two goal post are 8m apart a footballer is 34 m from one post and 38m from the other within what angle must he kick the ball if he is to score
Answered by Penny Nom.
Three numbers have a sum of 45 2020-05-06
From Jayden:
Three numbers have a sum of 45. The greatest of the 3 numbers is 2 greater than the least number. What are the numbers? *
Answered by Penny Nom.
The volume of a stone 2020-05-02
From KHOSI:
A rectangular container is 9cm wide,17cm long and contains water to a depth of 11cm. A stone is placed in the water and the water rises 2,2cm .Find the volume of the stone
Answered by Penny Nom.
The volume under an an uneven surface 2020-05-01
From Solon:
I am trying to fill an area of floor which has an uneven surface. I need to know the volume of fill. If you imagine a rectangular room (4 corners) that measures 189 inches by 240 inches and the depth at each corner is different.
Let us say corner #1 has a depth of 1", corner #2 has a depth of 0", corner #3 has a depth of 2" and corner #4 has a depth of 4".
How would i find the volume of said floor area?

Any assistance would be greatly appreciated!

Answered by Harley Weston.
A system of equations. 2020-04-27
From solomon:
xy + x =28
x + y +4

Answered by Penny Nom.
The width of a rectangle of land 2020-04-26
From Teresa:
If a rectangle shape piece of land is 813 feet long. How many feet width would be 2 acres?
Answered by Penny Nom.
Two chords 2020-04-14
From Frank:
How to find the length of the radius of a circle if you know the lengths of two chords
Answered by Penny Nom.
A pond in a garden 2020-04-09
From Jin:
c) I have a large square pond set inside a square garden: both the pond and the garden have sides which are a whole number of metres, and outside the pond, the garden is grassed over. The area covered by grass is 188 square metres. Find the area of the pond. (5 marks)
Answered by Penny Nom.
The center and radius of a circle 2020-03-25
From Riley:
so it says find the center and the radius of the below circle
x^2+(y-6)^2=121

i need help learning how to solve this

Answered by Penny Nom.
Is there an equation to find what percent of 0 is 1? 2020-03-09
From Asher:
Question: Is there an equation to find what percent of 0 is 1.

I remember learning quite a long time ago that the answer inst 0 and it isn't infinity. I'm pretty sure it was something like %=0 approaching infinity or %=1 approaching infinity. And I know it depends what value you assign the numbers i.e. dollars or temperature. Furthermore, is asking "what percent 1 dollar is of 0 dollars" the same question as "what percent profit do you make from selling something worth 0 dollars for 1 dollar."

Thank you for your time and consideration -Asher

Answered by Penny Nom.
The volume of a berm 2020-02-28
From jason:
how much dirt is in an 20'w 12't 1350'l berm
Answered by Penny Nom.
A straight line graph 2020-02-26
From Penehafo:
I have a problem of drawing a straight line graph by the equation y=2x-1
Answered by Penny Nom.
The height of an isosceles triangle 2020-02-23
From Reagan:
I need to find the height of an isosceles triangle with a base of 6 and sides of 4 units. How do I find it?
Answered by Penny Nom.
30% profit 2020-02-20
From Sidra:
A car dealer bought a car for £7500 and sold it for 30% profit.

For how much did he sell the car?

Answered by Penny Nom.
Sipping juice from a Tetra Pak 2020-02-11
From Anjhelic:
Karen is sipping juice from a 1 in. by 3 in. by 6 in tetra pack at the rate of 0.5in³/sec. How fast is the height of juice in the pack decreasing?
Answered by Harley Weston.
The area of a triangle from two angles and a side 2020-02-10
From Chinmoy:
How to measure the area of a triangle with two angles and length of the included side known?
Answered by Harley Weston.
A system of inequalities 2020-02-01
From Isaac:
Austin has x nickels and y dimes, having a maximum of 15 coins worth a minimum of $1 combined. No more than 4 of the coins are nickels and no less than 11 of the coins are dimes. Solve this system of inequalities graphically and determine one possible solution.
Answered by Penny Nom.
The volume of a coffee cup 2020-01-25
From Amirah:
What is the volume of a polystyrene cup with a height of 8cm, diameter of 5cm at the top and 4cm at the bottom when it is filled with water?
Answered by Penny Nom.
The equation of a line 2020-01-05
From coco:
Good afternoon If relationship between total cost and the number of units made it linear ,and if costs increases by 7$ for each additional unit made ,and if the total cost of 10 units is 180$ .
Find the equation of the relationship b/n total cost (Y) & number of unit made (x).

Answered by Penny Nom.
Dividing the tips 2020-01-02
From Pat:
trying to figure out division of tips I have 3 full time employees 40 hrs a week and 1 part time worker 12 hours a week what percentages do i give them to give them appropriate tips :-)
Answered by Harley Weston.
A circle, a point, and a tangent line 2019-12-31
From Faisal:
A circle has radius 10 units and passes through the point (5,-16). The x-axis is a tangent to the circle. Find the possible equations of circle?
Answered by Penny Nom.
Volume of a tree 2019-12-19
From Maria:
I have trees for sale. I need to calculate the volume of a tree from base which is wider than the stem.The base will be used for the making of hurleys, similar to hockey sticks so the wider base is crucial.
Answered by Penny Nom.
Odds 2019-12-16
From Craig:
I’d like to know what are the odds of a lotto machine giving out a specific 4 digit number as a quick pick?
I know there are 10,000 possible 4 digit combinations. So let’s say the number is 3587 what are the odds the lotto machine spits it out EXACTLY that way on a quick pick?

Answered by Penny Nom.
The equation of a line 2019-12-10
From Jourdan:
Suppose a mining company will supply 96000 tons of ore per month if the price is 80 dollars per ton but will supply 70500 tons per month if the price is 20 dollars per ton. Assuming the supply function is of the form y = m x + b , find the slope, m and y-intercept, b
Answered by Penny Nom.
Roll two dice and record the difference 2019-12-09
From Barbara:
Suppose you roll two dice 100 times. Each time you record their difference (always subtracting the smaller one from the bigger one to get a positive difference). The possible values you get are 0,1,2,3,4 and 5. You record the frequency of each value in the following table:
Difference of two dice 0 1 2 3 4 5
Observed frequency 12 31 26 13 10 8
Let your null hypothesis be that the dice are fair, and the alternative hypothesis be that they are not fair. Using a confidence level of α = 0.10, test the null hypothesis by a goodness-of-fit test.

Hint: begin by
completing table:
x 0 1 2 3 4 5
f(x)

Answered by Penny Nom.
The area and volume of a wedge 2019-11-30
From Jasmine:
what is the volume and surface area of a wedge with definition
Answered by Penny Nom.
The product of 3 integers is -24 2019-11-06
From Rick:
This is a question on my sons pre-ap practice quiz. I think that there is information missing (?)

The product of 3 integers is -24
The sum of 3 integers is -12
What are the 3 integers ?

This is exactly how it was written on his quiz paper. I have wasted to much time on the internet, trying to find a formula(s) to help him.
Please help me.

Answered by Penny Nom.
The sum of the ages of Ruth and her mother is 77 years. 2019-11-06
From Lydia:
The sum of the ages of Ruth and her mother is 77 years. The difference in their ages is 27 years. How old is each?
Answered by Penny Nom.
Algebra 2019-11-06
From Krizhia:
How is y= 1/2 * (x-1)² be x= √2y + 1. Please show me the steps
Answered by Penny Nom.
The divisors (aliquot parts) of a positive integer 2019-10-31
From Kenneth:
Is there a fast and simple method that can be used to determine the aliquot parts, whole, positive numbers that can divide into another whole, positive number?
Answered by Harley Weston.
The center of a circle 2019-10-24
From Patrick:
I'm trying to find a quick calculation to find the center of the circle that will touch the 3 points of an isosceles triangle.
Known dimensions are length of the triangle base and the height from the top point to the middle of the base.
Thanks in advance,
Kind regards,
Patrick

Answered by Penny Nom.
How many numbers from the set have the sum of its digits equal to 2? 2019-10-24
From Marcel:
Hello. My question is the following: Given the set {1,2...........10exp2019}, how many numbers from the set have the sum of its digits equal to 2?
I got some of them : 2 20 200 2000 20000 .......2. 10exp2018 11 110 1100 11000 110000....101 1010 10100 101000 1010000....10011 10010 100100 1001000 1010000.....10001 100010 1000100 10001000,,,,,,100001 1000010 10000100....... My problem is that I have difficulty in finding a strategy to obtain the exact quantity of numbers.
Please help me, thanks

Answered by Penny Nom.
The perimeter of a piece of a circle 2019-10-18
From Arwin:
The radius of a circle is 17 cm. A chord XY lies 9 cm from the centre and divides the circle into two segments. Find the perimeter of the minor segment.
Answered by Penny Nom.
The domain of f + g 2019-10-16
From Angel:
The domain of f+g when f(x)=8x+3 and g(x)=1-5x
Answered by Penny Nom.
How do you find the length and the width if all you had was area? 2019-10-16
From Ali:
So, How do you find the length and the width if all you had was area?
Answered by Penny Nom.
The volume of a fire hose 2019-09-18
From Carlou:
How many liters of water in a 2.5 inches diameter of fire hose with a 20 meters in length
Answered by Penny Nom.
What is -5 squared? 2019-09-10
From Pori:
What is -5 squared?
Answered by Penny Nom.
Vectors 2019-09-06
From Kayla:
Hi! Due to time constraint issues, we skipped vectors in Pre-Calc last year. However, I'm attempting to complete my AP Physics summer homework and there are 3 problems with vectors. As I explained, I have no idea even where to begin and as such I need some help.

Problem 1:
A plane flying at 90° at 45 m/s is blown toward 0° at 62 m/s by a strong wind. Find the plane's resultant velocity. I have tried all sorts of online calculators but none are providing me with a formula I can use to solve this problem on my own.

Problem 2:
If you walk 367 m North and 785 m West what is your total displacement from your original location?
What is the problem asking for when it says displacement?

Problem 3:
A plane travels on a heading of 127.0° at a velocity of 25 km/hr. What are the horizontal and vertical components of the plane's velocity? As you know already, I have never done vectors so I don't know what horizontal and vertical components of velocity are.

Thank you So Much for your help!! I am struggling!!

- Kayla

Answered by Penny Nom.
Using the Laws of Exponents 2019-09-06
From Samiya:
i have been struggling with the laws of exponents so im having trouble with this question. the question is to write (b*exponent2*c)(b*exponent5*c*exponent4*)
Answered by Penny Nom.
The volume of a berm 2019-09-06
From John:
I need to construct a berm:
8' tall
5' across the top
20' at the base
125' long
How much soil will I need

Answered by Harley Weston.
Can one equation with two variables be solved algebraically? 2019-08-30
From Don:
Can one equation with two variables be solved algebraically?
Answered by Penny Nom.
The length of half an acre 2019-08-23
From Angeles:
How many Feet Long In Half An Acre (Rectangle)
Answered by Penny Nom.
The area of a shape 2019-08-14
From Lymp:
Hi, for math we are calculating the area of shapes and my teacher wanted me to research how to calculate the area of shapes so i was wondering if you could tell me how t calculate the area of any shapes. Please tell me how to calculate a cross, parallelogram and a polygon
Answered by Penny Nom.
A cone of maximum volume 2019-08-14
From Refilwe:
The slant height of a cone is 10cm. Determine the radius of the base so that the volume of the cone is a maximum
Answered by Penny Nom.
The radius of a circle of given area 2019-08-14
From shelby:
What's the radius of a circle that has an area of 803.84cm2? I also need examples of how you got the answer.
Answered by Penny Nom.
The volume of a well 2019-07-30
From Jordan:
How much water could my well hold if the depth of the well is 30 feet feet and the diameter of the well is 24 feet ?
Answered by Penny Nom.
The maximum volume of a cone 2019-07-14
From A student:
find the maximum volume of a cone if the sum of it height and volume is 10 cm.
Answered by Penny Nom.
Volume of a spherical cap 2019-07-12
From Shanto:
In a jar of water, we kept a sphere with radius r. Then 75% of its volume went under water. Find out the distance of the top of the sphere from the surface of the water.
Answered by Penny Nom.
Order of operations 2019-07-06
From Carri:
What is the answer (( 5 × 12) / 3 ) + 30 - 50
Answered by Penny Nom.
The perimeter of 200 acres 2019-06-14
From AARON:
How many feet is the perimeter of 200 acres?
Answered by Penny Nom.
How many lines of symmetry does a circle have? 2019-05-28
From raji:
how many lines of symmetry does a circle have???
Answered by Penny Nom.
The volume of a wall 2019-05-08
From Peter:
I’m building a dry stack stone wall which is 36” wide at the base, 42” tall and 24” wide at the top. What would be the volume for a 36” long segment ?
Thanks, Peter

Answered by Penny Nom.
Volume and Surface area of a sphere 2019-05-03
From Caitlin:
Why does the surface area formula of a sphere have a squared radius while the volume of a sphere has a cubed radius?
Answered by Penny Nom.
A dot on a child's car wheel 2019-04-28
From julia:
A wheel of a child’s car revolves at 10 rpm. A painted dot on the circumference of the wheel is 12 cm from the centre of rotation of the wheel. How far does the dot travel in 5 s, to one decimal place?
Answered by Penny Nom.
The length of an arc 2019-04-28
From Patrick:
If an arch is 48 inches wide at the base and 30 inches tall at its apex, what is the length of the arch?
Answered by Penny Nom.
Constructing a hexagonal pyramid 2019-04-25
From REBECCA:
I'm doing a project for my Honors Geometry class and I have to build a regular hexagonal pyramid. I was given the volume of .75 gallons, but I don't know how to figure everything else out. Please help?
Thanks,
Rebecca

Answered by Harley Weston.
The area of a triangle 2019-04-25
From troy:
The base and the height of a right triangle are represented by the expressions below. The area of the triangle is 28 square units. Find the lengths of the base and the height of the triangle

base= x+4
height=x-6

Answered by Penny Nom.
The area of a lot 2019-04-06
From Robert:
Can you help me determine the rough square footage of this lot please(attached)? I think it is nearly 7000 square feet. I am sure I need the degrees of of the corners but I don’t have that. An estimate would be all I need. Thank you for you help. Rob
Answered by Harley Weston.
The height of a building 2019-04-05
From Hayley:
A young man is 36 meters away from a building and looking at a bird sitting on top of the building. The man knows he is 85 meters away from the bird. How tall is the building?
Answered by Penny Nom.
Internal acute angles 2019-04-02
From karan:
Prove that a convex polygon cannot have more than three acute internal angles.
Answered by Penny Nom.
The volume of a pile of dirt 2019-04-01
From Genevieve:
We have a pile of dirt approximately 6 feet high by 15 feet wide by 80 feet long.
We need to know how many cubic yards of dirt are in this pile. Thank you

Answered by Harley Weston.
The volume of water in a pipe 2019-03-21
From Abati:
How can I calculate the volume of water in a pipe with below details;

Pipe diameter - 6 inches
Length of pipe - 127meters

Answered by Penny Nom.
The area of a triangle 2019-03-19
From Dylan:
The area of a triangle is 108in squared. A base and corresponding height are in the ratio 3:2. Find the length of the base and the corresponding height
Answered by Penny Nom.
Four consecutive positive integers 2019-03-12
From Nabila:
The product of four consecutive positive integers in 24024
What is their sum?

Answered by Penny Nom.
A triangle of maximum area 2019-03-07
From Tom:
Triangle ABC is such that AB=3cm and AC=4cm.
What is the maximum possible area of triangle ABC?

Answered by Penny Nom.
Misuse of greater than 2019-03-07
From Kenneth:
I have an old business mathematics textbook. The authors have indicated that the following expressions indicate multiplication:

? is 2/3 greater than 90; ? is 2/3 smaller than 90. They also indicated that the following expression would indicate division: 30 is 2/3 greater than ? and 30 is 2/3 smaller than ?.

How can these phrases indicate multiplication and division? How can 60 be 2/3 greater than 90 and also smaller than 90 as indicated above. What were the authors thinking? I have added the page from the book that indicates what I have explained in my message Kenneth

Answered by Harley Weston.
3 months loss and then a profit 2019-03-02
From Ida:
Let say, I have 10k of capital and I ran business for 4 months. The first months i loss 50%, the second month i loss 50% and so do the third month. The fourth month my profit is 50%

So can I calculate the balance of money that i have out of my capital?

Answered by Penny Nom.
The top half of a cone 2019-02-27
From danica:
What portion of the height from top that the cone be cut into two equal volume?
Answered by Penny Nom.
A quadratic equation problem 2019-02-26
From Em:
One root of the equation 2x^2 + bx - 24 = 0 is -8. What are the possible values of b and the other root?
Answered by Penny Nom.
Composition of functions 2019-02-24
From Joshua:
Let g(x)=(x)^(2)+x-1 Find such a function f such that (fog)(x)=(x)^(4)+2(x)^(3)-3(x)^(2)-4x+6
Answered by p.
The Polynomial Remainder Theorem 2019-02-23
From pasandi:
f(x) is a quadratic polynomial. when f(x) is divided by (x-1),(x-2) & (x+2) the remainders respectively are -1, 4 and 2 how to find the f(x) in a question like this?
Answered by Penny Nom.
Margin with commission as part of selling price 2019-02-13
From Anni:
I need to find selling price to put us at 24% margin.
15% commission is based on selling price and is included in our cost.
I'm trying to do it excel and keep getting a circular reference.
Total cost= cost + commission (15% of selling price)
Selling price=total cost + 24% margin

Answered by Penny Nom.
A cylindrical drum 2019-02-10
From Ronald:
I have a cylinder that is 32 inches round by 25 inches deep how much water can it hold
Answered by Penny Nom.
A fraction of a cent 2019-02-01
From michael:
$ 300 divided by 1,000,000 is 00.0003 ... so, is that three hundredths of one ($ 00.0003) cent or what ... thanks
Answered by Penny Nom.
A median of a triangle 2019-01-24
From Anna:
For triangle J(1,4) K(-3,-1) L(3,4), find an equation for the median from vertex J
Answered by Penny Nom.
A triangle made from 12 matchsticks 2019-01-20
From Palesa:
How many different types of triangles can be made with a perimeter of 12 matches?
Answered by Penny Nom.
7 spheres on a hexagonal tray 2019-01-14
From herm:
what is the length of each side of a hexagonal tray, with the height of each side 0.75 inch, to hold seven spheres, each with a diameter of 3.00 inches? The spheres are placed such that each side of the hexagon is touched by one sphere at its midpoint (and the seventh sphere is place in the center of the "ring" of the other six spheres.
Answered by Harley Weston.
The surface area of a dome 2018-12-18
From Chithrabhanu:
If a dom dia is 4.552 and the height is 1.21 what is the surface area?
Answered by Penny Nom.
The area and weight of sod 2018-12-11
From Susan:
I have equipment that will cut sod 30 inches across and 75 Feet in length? how much will this be in square foot age ? Yards? Approximate weight?
Answered by Penny Nom.
A puzzling equation 2018-12-04
From Manny:
Hello,

Can you explain why this is wrong:
-1 = (-1)^3 = (-1)^{6/2} = ((-1)^6)^{1/2} = (1)^{1/2} = 1 ??

Thank you

Answered by Penny Nom.
The price of eggs rises 10 cents per dozen 2018-11-10
From Maddie:
Hi! I'm currently struggling with a word problem that says: "If the price of eggs rises 10 cents per dozen, one will be able to get 2 dozen fewer eggs with $6.00 than was possible at the lower price. What is the lower price? We are working on completing the square and all of the problems I've done so far in this chapter require using that method. If possible and needed, could you explain how to set up the equation? Thank you so much! I appreciate all your help!!
Answered by Penny Nom.
Why 1/2 in the area of a circular sector? 2018-10-26
From Ariel:
In the equation for area of a sector=1/2r^2theta why is it 1/2? Please explain conceptually not algebraically.
Answered by Penny Nom.
Why 1/3 in the volume of a cone? 2018-10-20
From Keith:
In the formula for the volume of a pyramid, Volume = 1/3 * base * height, where does the 1/3 come from, intuitively?
Answered by Penny Nom.
The area of a trapezoid 2018-09-24
From An other:
AB-272 feet: AC 690 feet; CD-330 Feet; DA-669Feet Calculate the area in feet and Acres.
Answered by Penny Nom.
Two curves 2018-09-23
From Megan:
How do i find the interception points of xy=-2 and y=x+3? Many thanks!
Answered by Penny Nom.
The point-slope equation for a line 2018-09-23
From Sally:
The equation of the line in point-slope form for the line passing through the point ( 4, -6) and parallel to the line y = -6x + 2. Describe the steps.
Answered by Penny Nom.
Predicting the number of fish in a lake 2018-09-16
From April:
A researcher catches 30 fish in a pond, attaches a marker to their fin and releases them. The next day she places a large net in the water and catches 9 fish. 3 have the marker attached to their fin. Can she predict the number of fish in the pond???
Thank you

Answered by Penny Nom.
A locus of points 2018-09-03
From Ericka:
Find the locus of points three times as far from (0,4) as from (2,0)
Answered by Penny Nom.
5 mm of water over 1 hectare 2018-08-10
From Joyce:
Please help me with this question.
How many tonnes of water fell on 1 hectare of land during a rainfall of 5mm?

Answered by Penny Nom.
The surface area of a levee 2018-07-04
From John:
Figuring out the surface area in sq. Ft. Of a levee

Height:10’
Top of levee width: 6’
Width of base (from toe to toe): 40’
Length of levee: 1,250’

Answered by Penny Nom.
The product of two numbers 2018-06-19
From amie:
the product of two numbers is 108.when the larger number is divided by the smaller number the quotient is 3. What are the number.tnx
Answered by Penny Nom.
The domain of f(g(x)) 2018-06-18
From Joshua:
What are the restrictions of the domain of f(g(x))?
Answered by Penny Nom.
The number of terms in an arithmetic sequence 2018-06-15
From Don:
how many terms in arithmetic sequence are there if the first term and the last term are 3&59 respectively in common difference is 4?
Answered by Harley Weston.
The area, radius and slant height of a cone 2018-05-18
From joette:
If you have a cone how to find the slant height when given the area and radius?
Answered by Penny Nom.
Rods,poles and perches 2018-05-13
From roy:
i have some land and its measured in rods poles and perches
how cann i convert to understand it more

Answered by Penny Nom.
Two expressions for the perimeter of a a rectangle 2018-05-13
From Slayde:
A rectangular house has one side 4 metres longer than the other. Create two different expressions for its perimeter. The expressions should be given in factorised and expanded form.
Answered by Penny Nom.
Gauss' Addition of whole numbers. 2018-04-30
From Brad:
I found this on your site. Question: what is the sum of the first 100 whole numbers?? Is there a different formula if the numbers begin at a number other than one? For example What is the series I want to add is goes from 7 - 53?
Answered by Harley Weston.
Spreading soil over 10 acres 2018-04-29
From Joe:
If you have 54000 cubic yards of soil and want to spread it over 10 acres of land, by how many inches would the ground level rise?
Answered by Penny Nom.
A rectangular prism with rounded corners 2018-04-25
From Jackie:
Rectangular prism that is 40 by 20 with rounded corners with a radius of 2.5.

What would the surface area be?

Answered by Penny Nom.
More on the curvature of the Earth 2018-04-23
From will:
the formula for figuring the earth's curve goes against logic, looking at a fixed point and backup 1mi. the point drops 8" then 16" in the next mi. and 32" in the third mi. why shouldn't it be 24" why is the 8" per mi. squared can you tell me in laymens terms why this is it goes against logic it would seem the correct wat would be to add up 8" per mile as you back up from the fixed point 8"- 16"- 24"- 32" not 8/16/32/64"
Answered by Harley Weston.
A volume expression for a pyramid 2018-04-23
From Shaheer:
Do a three sided pyramid and a square pyramid have the same formula if you want to calculate the volume?
Answered by Penny Nom.
How to find the area of triangle? 2018-04-19
From g.a:
how to find the area of triangle
Answered by Penny Nom.
The sides of a circle 2018-04-10
From Reid:
Hello,
I recently was wondering about whether or not a circle has an infinite number of sides, and I ended up searching it on your website. I saw that you guys found the question to be too ambiguous or something of that nature, and I thought about your process of reasoning involving vertices and such. I soon realized that I may have come up with the solution to the question, but I want to confirm it with you guys. Allow me to explain:
A circle, unlike any other typical 2 dimensional polygon, can sustain an infinite number of straight lines coming in contact with only one point on it. A square, for example, cannot. A square only has 4 locations that can sustain a such a line, each of those being its corners. The flat edge of a square cannot support a tangential line, because the line would either be crossing the edge of the square or coming into contact with multiple points along its edge.
This concept is consistent in every 2 dimensional polygon: pentagons sustain 5 locations for tangential lines, hexagons 6, nonagons 9, etc. The reason a circle has an infinite number of sides is simply the fact that it must have an infinite number of "corners", assuming it can be defined as a polygon like any of the shapes I described above.
Corners can only exist assuming there is two sides coming into contact with one another. If there is an infinite number of corners in the circle, which is apparent due to the above reasoning, there must be an infinite number of sides coming into contact with each other. Thus, a circle is a polygon that consists of an infinite number of sides coming into contact with each other.
Please review this reasoning and let me know if it is solid.
Thank you!

Answered by Penny Nom.
A circle problem 2018-03-30
From ahamed:
the diameters of a circle AB and CD intersect each other at O perpendicularly. Then a chord DF is drawn. AB and DF intersect at E. DE=6cn and EF=2cm.find the area of thw circle
Answered by Penny Nom.
The volume of a tent with a hexagonal base 2018-03-27
From shohel:
A tent has its base in the shape of a regular hexagon whose sides are 10m . If the height of the tent is 12m , then find its volume.
Answered by Penny Nom.
Distance between a chord and its arc on a circle 2018-03-23
From Doug:
Specifically, what is the subject distance for the Earth orbiting for 27 days. Assume the orbit of the Earth to be a circle have a 93 million mile radius. Assume the angle of arc to be (27/365) x 360 degrees. Thank you.
Answered by Penny Nom.
Walking a fraction of a mile 2018-03-20
From Jamie:
Who walked a fraction of a mile that is closer to neither 0 nor 1 ? Explain
Avery 1/6
me.Nunez 5/6
Ms. Chang 1/3
Mr. O’Leary 4/8
Miss Lee 4/6

Answered by penny Nom.
The sum and difference of two numbers 2018-03-13
From samima:
Two numbers have a difference of 0.85 and the sum 1.What are the numbers?
Answered by Penny Nom.
The dimensions of a rectangle 2018-03-03
From Mia:
A wire of length 36cm is bent to form a rectangle. Find the dimensions of the rectangle.
Answered by Penny Nom.
Expand (5x-9)(5x+9) 2018-03-01
From adil:
expand the following:
(5x-9)(5x+9)

Answered by Penny Nom.
The perimeter of a square 2018-02-25
From Alicia:
What's the perimeter of a square with 441cm square
Answered by Penny Nom.
Dividing a region in half 2018-02-24
From mandy:
There is a line through the origin that divides the region bounded by the parabola y=4x−5x2y=4x−5x2 and the x-axis into two regions with equal area. What is the slope of that line?
Answered by Penny Nom.
The slope of a line 2018-02-22
From Steve:
my current slope is 6%. I want to know how many inches higher I will be at 18' Thanks
Answered by Penny Nom.
A footballer angle 2018-02-14
From Kim:
Two goal posts are 8m apart. A footballer is 34m from one post and 38m from the other. Within what angle must he kick the ball if he is to score a goal.
Answered by Penny Nom.
Related rates 2018-02-11
From angelo:
hi admin please help me answer this question. thank you! At a certain instant of time, the angle A of a triangle ABC is 60 degrees and increasing at the rate of 5degrees per second, the side AB is 10cm and increasing at the rate of 1cm per second, and side AC is 16cm and decreasing at the rate of 1/2 cm per second. Find the rate of change of side AB?
Answered by Penny Nom.
Water in a cone 2018-02-10
From Shuvo:
The diagram shows a vertical cross-section of a container in the form of an inverted cone of height 60 cm and base radius 20 cm. The circular base is held horizontal and uppermost. Water is pursed into the container at a constant rate of 40 cm3/s.
Show that, when the depth of water in the container is x cm, the volume of the water in the container is (πx^3)/27 cm3.
Find the rate of increase of "x" at the instant when "x" = 2.

Answered by Penny Nom.
Positive and negative values of a function 2018-01-30
From Grayson:
f(x)=x^6-x^4
Interval: ( negative infinity, negative one )
Test Value: negative two
Function Value f(x): positive forty eight
Interval: ( negative one, zero )
Test Value: negative one
Function Value f(x): zero
Interval: ( zero, positive one )
Test Value: positive one
Function Value f(x): zero
Interval: ( positive one, positive infinity )
Test Value: positive two
Function Value f(x): positive forty eight

What is the sign of f(x) for each Interval?

Answered by Penny Nom.
The sum and difference of two numbers 2018-01-25
From Ali:

Hello,
I was looking at the original question that was posted and answered by
your team:
http://mathcentral.uregina.ca/QQ/database/QQ.09.07/s/donna1.html

I used the 7 and 4 as example and not looking for 39 per original
question:

A=7
B=4

7+ 4 =11 and 7-4 = 3

We end up with 11 +3 = 14

A= 14/2 give use 7

B = how do you solve for B or 4 with out knowing anything about 3 or any other numbers ?

Thank you
Ali


Answered by Penny Nom.
The equation of a line 2018-01-23
From Gloria:
Write the equation of a line in standard form through the point (1, 6) and perpendicular to the line 2x –y = 9.
Answered by Penny Nom.
y as an expression in terms of x 2018-01-20
From Mark:
What is y as an expression in terms of x ?
Answered by pennt.
Profit margin and markup 2018-01-18
From Oyewumi:
Mr A sold a car to Mr B at 20% profit margin and Mr B sold the car to Mr C at 30% markup on cost. If Mr C got the car at $x.
1. How much did Mr A buy the car in terms of x.
2. How much did Mr A buy the car if x is $2800000

Answered by Penny Nom.
When are the hands of a clock perpendicular? 2018-01-11
From Nazrul:
How many times in a day are the hands of a clock perpendicular to each other? How can I find the times? Please help me.
Answered by Penny Nom.
A fraction of a mile 2018-01-06
From Sierra:
There are 5280 feet in a mile. what fraction of a mile is represented by 660 feet?
Answered by Penny Nom.
A puzzling sequence 2017-12-19
From Alan:
My son has the following sequence to work out the formula for the nth term (and fing 5th and 6th terms) and I have tried a variety of ways but can't help!

1/2, 5/6, 1, 11/10

I have converted all to 15/30, 25/30, 30/30, 33/30 and so think the next two terms are 37/30 and 45/30 but I have no idea how to prove or if I am right!

Any help appreciated.

Answered by Penny Nom.
Needing the area of an irregular Pentagon 2017-12-06
From Dale:
I'm needing to find the square footage of an irregular pentagon.
I've seen the other questions similar to this one, but without a great deal of difficulty can't get you the angle degrees, or the length between opposing angles.
I'm hoping that a different bit of information can help. So here goes:
The longest side is 67ft long.
One of its legs is 39ft long, and the other is 18ft long.
The leg off of the other end of the 39ft leg is 35ft long.
The leg off of the other end of the 18ft is 49ft long, and meets the other end of the 35ft side.
The information that I DO have, which I'm hoping makes the difference, is that the 39ft and the 18ft sides are parallel.
Also, while the angles at each end of the 67ft side are not right angles, they are fairly close to it, approximately 80 degrees at the 39 ft side corner, and 100 degrees at the 18ft side corner.

Dale.

Answered by Harley Weston.
Heat equation 2017-11-23
From Max:
What does du\dt=a(triangle)^2u mean. Can it be solved for t.
Answered by Penny Nom.
How many miles did he drive in one hour? 2017-11-21
From Ava:
Graham drove 42 1/3 Miles in 1 1/3 hours. How many miles did he drive in one hour?
Answered by Penny Nom.
An expression for the area of a triangle 2017-11-19
From hari:
why is area of a triangle drawn from centre of a circle i.e radii forming the sides is 1/2 *r^2 sin theta where r is radius
Answered by Penny Nom.
Splitting twice 2017-11-11
From Terry:
4 people are splitting 1/3 of something. 1 is to receive twice as much as the other 3. what it the fractional interest for each?
Answered by Penny Nom.
The graph of graph y= -2x-1 2017-11-11
From Natividad:
How do you graph y= -2x-1?
Answered by Penny Nom.
Surface area of a hemisphere 2017-10-28
From Shaina:
If the total surface area of a hemisphere be 36.22/7 cm then its radius will be 3 cm is it true or false
Answered by Penny Nom.
f(x)=-x^2-6x find f(x-2) 2017-10-27
From Kenneth:
f(x)=-x^2-6x find f(x-2)
Answered by Penny Nom.
The distance between the origin and a moving point on a graph 2017-10-16
From Paulina:
Find the rate of change of the distance between the origin and a moving point on the graph of y=x^2 +1 if dx/dt=2 centimeters per second
Answered by Penny Nom.
The volume of a cone without calculus 2017-10-02
From Akash:
How to find the volume of a cone without the knowledge of calculus?
Answered by Penny Nom.
The square root of 6 is irrational 2017-10-02
From John:
http://mathcentral.uregina.ca/QQ/database/QQ.09.06/sylvia1.html
In the initial assumption of that proof, root 6 is assumed to be a/b where a and b have no common factors, but why does having a common factor make it irrational?

Answered by Penny Nom.
Four multiples of 10 2017-09-23
From Laudacir:
Four multiples of 10 are added together.the total is a 3 digit number with three consecutive digits. What could the four number be?
Answered by Penny Nom.
The surface area of a sphere 2017-09-17
From penny:
The surface area of a sphere is 1519.76cm. Taking 3.14 , find the radius of sphere
Answered by Penny Nom.
Quadrilateral ABCD is inscribed in a circle 2017-09-11
From Joie:
Quadrilateral ABCD is inscribed in a circle such that side DA is the diameter. AB=2m., BC=4m., CD=6m., angle BAD=75.93degrees. Find the area of the quadrilateral.
Answered by Penny Nom.
The volume of an attic 2017-09-07
From paul:
trying to determine volume of an attic base is rectangular 29 ft by 37 ft peak runs parallel to longest base side and is 7 ft long roof lines run from each end of the peak to the respective corners the peak is 3.5 ft above the base
(so seen from above there are 5 visible lines: the peak and 1 line to each of the 4 corners)

Answered by Harley.
The perimeter of a sector of a circle 2017-08-11
From Alisa:
find the perimeter of the sector with radius 7 cm in which the angle at the centre is 60 degrees leave your answer in terms of pi
Answered by Penny Nom.
Teams of 4 from 20 people 2017-08-04
From Christopher:
math team has 20 people on the team but only 4 can enter the competition. The team captain has to pick 4 of them but he knows that he cannot put James, Bill or Todd on the team with one another or there will be problems. what would the formula be to find the solution to how many possible team combinations the are
Answered by Penny Nom.
3 consecutive multiples of 11 2017-07-22
From nisha:
using the multiples formula shown at ypur site how can we solve finding 3 consecutive multiples of 11 whose sum is 363
Answered by Penny Nom.
The equation of a circle 2017-07-03
From Chen:
Find the center radius and equation of a circle in standard form given the following conditions:

1. Tangent to 3x+2y=0 at the point (0,0) and passing through (1,-1) and (6,0)

Answered by Penny Nom.
The surface area of a hexagonal pyramid 2017-06-16
From Emilie:
Hello, I need help with this question:

A pyramid has a regular hexagonal base with side lengths of 4 and a slant height of 6. Find the total area of the pyramid.

Thank you in advance if you can help me...

Answered by Penny Nom.
The maximum area of a rectangle with a given perimeter 2017-06-02
From Bob:
How would I go about finding the maximum area of a rectangle given its perimeter (20m, for example)?
Answered by Penny Nom.
Two wires between two buildings 2017-05-11
From Bill:
I need to find the height of the intersection point of two building wires< br /> Building A is 860 feet high and building B is 480 feet high. The wires existent from the top of one building to The bottom of the other < br /> The distance between is 32 feet
Answered by Penny Nom.
Canada's population as a percentage 2017-05-11
From bethmarie:
What percentage of the world population (7.5 billion) is Canada's population (36 million). Thank you very much!
Answered by Penny Nom.
Slicing an inverted bowl at various heights 2017-04-30
From Joel:
Find a formula to calculate the circumferences of an inverted bowl at various heights.

E.g. Take an inverted bowl with a diameter of "x" cm and a depth / height of "y" cm.

How can I calculate the circumferences at distances of various heights above the base?

Alternately, what would be the formula to calculate the distance of the line segment resulting from a line which intersects both sides of a parabola in which that line is drawn parallel to the tangent of the vertex of the parabola at any given distance from the vertex?

Answered by Penny Nom.
1 acre as a 2 foot wide strip of land 2017-04-25
From alan:
How can one convert 2 acres (a square - area) of land into a strip of land that is 2 feet wide and (?) feet long?
Answered by Penny Nom.
The length of a train 2017-04-24
From Stefano:
If a freight train is coming towards you and you were traveling at 85 miles an hour and it takes 35 seconds for the train to pass you how long is the train

Figure 50mph for train

Answered by Penny Nom.
Forming a cone from a circle 2017-04-15
From Tasha:
A sector of a circle subtends an angle of 216 degrees at the centre, If this sector is used to form a cone of vertical height ,8cm, calculate the base radius of the cone
Answered by Penny Nom.
Why does 10x10x10 give 1,000.0000000000001? 2017-04-12
From Randolph:
Hi, On your calculator I found that a box 10 by 10 by 10 inches has a volume of 1,000.0000000000001 cubic inches. Can you explain the numeral 1 thirteen places past the decimal? Thanks, Randy
Answered by Penny Nom.
The height of a isosceles trapezoid 2017-04-03
From Riham:
Hi
How can I find the height in an isosceles trapezoid if I have the measurements of all of its sides ? Thank u in advance.

Answered by Penny Nom.
The sum and difference of two numbers 2017-03-20
From Olivia:
The sum of two numbers is 87 and their difference is 29. What are the two numbers
Answered by Penny Nom.
2^a, 2^b, 1^1 and 1^0 2017-03-17
From Émile:
If you had 2^a = 4 and 2^b = 4 you could assume that a = b right? Yet we can observe that 1^1 and 1^0 both equal 1, yet 1 = 0 isn't true. So if given a log base 1 number 1 the would the answer be 1 AND 0?
Answered by Penny Nom.
The ratio of two ages 2017-03-15
From Angel:
The sum of Keith's age and Alan's age is 36. Four years ago, the ratio of Keith's age to Alan's age was 4:3. Find the ratio of their present age
Answered by Penny Nom.
What is the grade of the incline? 2017-03-12
From Jon:
If I am climbing 16 feet in elevation over 80 feet horizontal. What is the grade of the incline
Answered by Penny Nom.
More on calculating the area (acreage) of a four-sided lot 2017-03-06
From Kerri:

Can you please show the formula used to calculate areas as provided on this page:

Calculating the area (acreage) of a four-sided lot


Answered by Penny Nom.
The inside diameter of a silo 2017-02-28
From JOHN:
I need to find the inside diameter of a cement silo with an outside circumference 118 ft and a wall thickness of 7 inches. thank you John Kerr.
Answered by Penny Nom.
The volume of a slab 2017-01-21
From Russell:
Working with a segment of a circle. The O.D. is 3.440" the length of the chord is 3.130 I need information on the volume of the largest area. I would like to know (in thousands of an inch) how tall would the largest area need to be to equal .1143 cubic inches If you need more info I can send it Thanks in advance
Answered by p.
The average rate of change of cot(t) 2017-01-18
From Brianna:
Hello! It's been a while since I've taken a math course, and I'm stuck on a problem in my calculus course.
The question is this:

Find the average rate of change of the function over the given interval.
h(t)=cot(t) a) [5pi/4, 7pi/4]

Answered by Penny Nom.
One millionth of one percent of 90,000,000,000 2017-01-16
From Gene:
What absolute number is one millionth of one percent of 90,000,000,000
Answered by Penny Nom.
8^3/2(2+2) 2017-01-13
From Mary:
8^3/2(2+2)
Answered by Penny Nom.
Water leaking from a trough 2016-12-28
From Kathryn:
A trough is 6 m long, and has uniform cross-section of an equilateral triangle with sides 1 m. Water leaks from the bottom of the trough, at a constant rate of 0.1 m3/min. Find the rate at which the water level is falling when the water is 0.2m deep.
Answered by Penny Nom.
20+30*0+1=? 2016-12-26
From Mr:
20+30*0+1=?
Answered by Penny Nom.
Two concentric circles 2016-12-21
From shrestha:
Two concentric circles have radii of 14 cm and 7 cm respectively. Find the area of space between them.
Answered by Penny Nom.
The inverse of y=x^2+x+c 2016-12-21
From Sam:
What is the inverse of y=x^2+x+c for x?
Answered by Penny Nom.
The inverse of y = x(x - 2) 2016-12-21
From Sasha:
Hello, I recently stumbled upon this question and I haven't been able to figure it out.

Why is the inverse of y=x(x-2) not a function?

Suggest a domain restriction which would ensure that the inverse is a function.

Thank You :)

Answered by Penny Nom.
The area and radius of a circle 2016-12-19
From india:
How do you find the radius of a circle if only the area is given to you?
Answered by Penny Nom.
980 cubic yards of dirt 2016-12-17
From Jondi:
I am trying to visualize 980 cubic yards of dirt by using a metaphor.
Would this be the size of a car?
A fire engine?
A semi-trailer?
How many trucks would you need to haul away 980 yards of dirt?

Answered by Penny Nom.
1/1-cosine(2x) - 1/1+ cos(2x) 2016-12-14
From Sean:
1/1-cosine(2x) - 1/1+ cos(2x)
Answered by Penny Nom.
How far apart are the boats? 2016-12-13
From Halley:
Two boats leave port at the same time. Boat A travels east at a speed of 12 km/hr. Boat B travels southwest at a speed of 14 km/hr. After two hours, how far apart are the boats? North is 0 degrees. How do I figure this out. Thanks
Answered by Penny Nom.
Profit and loss on two shirts 2016-12-06
From Mahesh:
A man purchases one shirt and one T-Shirt for Rs. 6000. He sells the shirt at a profit of 20% and T-Shirt at a loss of 10%, as a result he gains 2% on whole transaction. what is the cost price of the T-Shirt ?
Answered by Penny Nom.
Gallons and cubic inches 2016-12-05
From Theresa:
If I did not know the cubic inches in a liquid gallon how would I find it? In other words what is the formula for this?
Answered by Penny Nom.
The amount of material remaining on a roll 2016-11-11
From Yoh:

Question from Yoh:

Hello,
I am trying to find impressions on a roll (either full roll or partial).
Let's say I have the following information.
- Outer Diameter of roll
- Inner Diameter of roll (cardboard core)
- thickness per layer
- Each cut off length (impression length)

Now let's say a roll has a 40in outer diameter, the cardboard core has a 3.75in diameter and a thickness of .002. Each impression has a cut off of 14inches.
With this the roll will have approximately 2,700 impressions. How would I find the remaining impressions if the outer diameter of the roll is only 6.5inches?

Thank you.


Answered by Penny Nom.
The derivative of the inverse of a function 2016-10-28
From Kate:
Hi, I'm in a College level Calculus course and I can't seem to figure out the answer to this question.
Find the derivative of f^-1(4) if f(3)=4 and f'(3)=1/7

Answered by Penny Nom.
y = mx + c and Ax + By + C = 0 2016-10-18
From Erick:
please explain to me how to convert an equation from the form y=mx+c to Ax+By+C=o form.
Answered by Penny Nom.
The degree of 2x + 7 2016-10-17
From Deejay:
How about when the given has no degree and the other has a constant only, For example 2x + 7, where's the leading term and what is its leading coefficient?
Answered by Penny Nom.
The equation of a line 2016-10-01
From Miriam:
write the equation of the line that passes through (6,7) and is perpendicular to 3x+5y=0
Answered by Penny Nom.
The equation of a circle 2016-09-30
From Trent:
write the equation of circle B with center B(-2,3) that passes through (1,2)
Answered by Penny Nom.
Two equations 2016-09-17
From Nilesh:
How to solve (x-y)^1/2 + 3(x-y) = 30 ; xy + 3(x-y) = 11?
Answered by Penny Nom.
The radius of a coffee cup 2016-09-12
From Brett:
What is inside radius, in centimeters of a coffee cup if it holds 350g of coffee when filled to a depth of 9.5 cm? Assume coffee has the same density of water, 1.00g/cm3. A numeric value is expected and not an expression.
Answered by Penny Nom.
The equation of a ski slope 2016-08-31
From Hannah:
The information provided is
---A ski slope has a slope of -0.2. You start at an altitude of 10,000ft. Then the question is "Write the equation giving altitude (a) as a function of horizontal distance moved (d).
I know this is probably a very simple question but I didn't take a math class my senior year and now I'm a freshman in college, so some of it is very frustrating to me.
Any help is much appreciated, thank you!

Answered by Penny Nom.
The dimensions of a rectangle 2016-08-31
From Hazym:
How do I find the length(which is 15m longer than the breadth) and the breadth of a rectangle just by its perimeter which is 70 m?
Answered by Penny Nom.
Covering an octagonal region with sand 2016-08-13
From Lynn:
How much sand is needed to fill a 20 ft. wide octagon area 6 inches deep?
Answered by Penny Nom.
The area of a circle 2016-08-12
From sandeep:
if circumference of a circle is πd what is the area?
Answered by Penny Nom.
The diameter of a cylindrical tin 2016-08-04
From Scott:
a cylindrical tin is 15 inches high and holds 5 gallons of paint when it is full. Find the diameter of the tin.
(1 gallon = 0.1605 cubic feet)

Answered by Penny Nom.
The dimensions of a rectangle 2016-07-25
From Krunox:
A rectangle is twice as long as it is wide. Its perimeter (the sum of the lengths of its sides) is 60 cm. Find its length and width.
Answered by Penny Nom.
The number of sides of a polygon 2016-07-23
From Shriya:
All the angles of a polygon are either 155° or 140°.
There are twice as many angles of 155 °or 140°.
Find the number of sides of the polygon

Answered by Penny Nom.
A congruence theorem for two right angled triangles 2016-07-17
From Sayari:
Hello. Is it possible for two right angled triangles to have the same length of base and height and a different hypotenuse? If not so, then why in the congruence criteria RHS the hypotenuse is given more importance? It can also be like- 'two triangles are congruent if they are right angled and have the same base and height.' Thank You.
Answered by Penny Nom.
The composition of a function with itself 2016-07-17
From Mel:
If f(1 + 3x) = 1 * x, solve f(f(x))
Answered by Penny Nom.
Triakis 2016-06-27
From Gordon:
Some authorities treat "triakis" (and related terms "dyakis", "tetrakis", etc.) as attached prefixes; others treat them as separate adjectives. Thus I see one of the Catalan solids described as both "triakistetrahedron" and "triakis tetrahedron". Which usage is correct?
Answered by Chris Fisher.
Subsets 2016-06-26
From Kats:
How Many sub sets are in set k={6,7,3}
Answered by Penny Nom.
Cubic feet of soil 2016-06-19
From H:
A garden is 8 ft x 7 ft that needs to raised 6 inch's How many cubic feet of soil required ? What am I missing ?

Thanks

H

Answered by Penny Nom.
Profit 2016-06-15
From Toyia:
12.87 profit from 21.99 sale. What is the percentage that was taken off the 21.99
Answered by Penny Nom.
A barn roof 2016-05-29
From Joe:
Is it possible to build a barn roof (irregular pentagon?) with a 12' base and the other 4 sides 4' each? Thanks.
Answered by Penny Nom.
A cylinder in a cube 2016-05-25
From Sonia:
A cylinder has the same height and diameter as the sides of a cube use 3.14~ which has the greater volume
Answered by Penny Nom.
The area of a 4-sided lot 2016-05-25
From prasad:
How to find area of a land whose sides are 41ft,33ft,32.3ft and 33.2 ft. Pl give me the formula and proof.
Answered by Penny Nom.
Two pieces of rope 2016-05-21
From Render:
Fred cuts a 12- inch piece of rope. Then he cuts a second piece of rope that is 3/2 longer than the first piece. Is the second piece shorter or longer than the first piece? Explain.
Answered by Penny Nom.
A trough with a triangular cross section 2016-05-21
From Clarice:
A trough having an equilateral triangle end sections has sides equal to 0.4 m and 7m long.what is the volume of the liquid in the container if the depth of the water is one half the depth of the trough?
Answered by Penny Nom.
The volume of a truncated rectangular pyramidal pond 2016-05-13
From Paul:
How do you calculate a partially filled truncated rectangular pyramid if you always know the bottom rectangle, the maximum height top rectangle perimeter, but have a varying height. Similar to filling up a pond you know the current height and dimensions at the max rectangle how do you calculate it half full i.e. 10x30 outside perimeter with a 2x8 base and a max height of 6ft how do you calculate it at 3ft without re-measuring the top perimeter. Thanks, Paul
Answered by Harley Weston.
A pair of equations with fractions 2016-05-11
From tiya:
hello, i want to know how to solve this question.

m/6+2n/3=6
-m/10=2n/5=2

Answered by Penny Nom.
Solve 2^2x + 3(2^x) - 4 = 0 2016-04-27
From Lloyd:
Solve the equation 2^2x + 3(2^x) - 4 = 0
Answered by Penny Nom.
The inverse function for f(x)=2x^2+5x 2016-04-23
From Ashini:
The inverse function for f(x)=2x^2+5x
Answered by Penny Nom.
A sector of a circle 2016-04-21
From mustafa:
In a sector of a circle, the arc length is equal to half the perimeter of a sector.find the area of a sector in terms of r
Answered by Penny Nom.
15.86 acres 2016-04-21
From Ronald:
If a rectangular piece of land is 1320 ft. on one side, how many feet would be required on the other side to equal 15.86 acres?
Answered by Penny Nom.
A square of side length 4x-3 units 2016-04-20
From Bryan:
Find an expression that represents the area of a square with side length 4x-3
Answered by Penny Nom.
The derivative of x! 2016-04-16
From Sang:
How to find the derivative of x! and integral of x!
Answered by Penny Nom.
tan15° 2016-04-11
From JOHN:
find the exact value of tan15° in surd form.
Answered by Penny Nom.
Filling a hole with 14 tons of rock 2016-04-08
From Barry:
If I had 14 tons of inch and one half sized aggregate rock, how large of a square or rectangular shaped hole would I need to hold that amount?
Answered by Penny Nom.
The last two digits of a phone number 2016-04-03
From Joshua:
I want to find the last two digits of a phone number whose first eight numbers of a ten digit phone numbers are known.
Answered by Penny Nom.
The volume of a cone 2016-03-31
From Odum:
Find the volume of a cone with radius 6.5cm and height 12.6cm
Answered by Penny Nom.
The volume of a box with a hexagonal base 2016-03-27
From Frank:
how many gallons of water in an 8" hexagon shaped box. 6" sides x 12.5" high ? The walls are 1" thick , beveled @ 30 degrees, so inside dimensions are 6"
Answered by Penny Nom.
Travelling in a spacecraft 2016-03-25
From julie:
if i was travelling in a spacecraft at the speed of light and i turned on my headlight would i see the beam ?
Answered by Robert Dawson.
A rotating schedule for a softball team 2016-03-25
From Caroline:
I couldn't find a similar question, hopefully there isn't one!

I have a softball team of 18 people (7 girls and 11 boys), but am limiting each game to 13 players (6 girls and 7 boys). The season is 10 games long. How do I create a rotating schedule which allows members to play evenly? I tried separating the boys and girls to create rotation for each but I got confused. Some are unable to attend all games but for the simplicity of a rotation schedule can you please help?

Answered by Robert Dawson and Victoria West.
The inverse of a function 2016-03-20
From Billy:
f(x) = Square root(x^2 + 2x)

What is the inverse?

Answered by Penny Nom.
The perimeter of a rectangle 2016-03-17
From Linda:
Noel correctly adds the length of three sides of a rectangle and gets 88 cm her brother Ryan correctly adds the lengths of three sides of the same rectangle and gets 80cm. What is the number of cm in the perimeter of the rectangle
Answered by Penny Nom.
A block wall 2016-03-15
From Robert:
I have a wall 40 ft. long and 4 ft. tall as well as another one that is 30 ft by 3.5 ft. tall that I am trying to build. how many 6 inch by 12 inch landscaping blocks do I need?
Answered by Penny Nom.
The sides of a triangle 2016-03-15
From Hitarth:
Why we cannot construct a triangle ofngiven sides 5cm,5cm and 10cm?
Answered by Penny Nom.
Angles 2016-03-12
From Laurynn:
What are angles in general (please include the 'angle of incline')

Thank you
Laurynn

Answered by Penny Nom.
The integral of a sum 2016-03-10
From Rahul:
How to solve definite integral of a sum. The specific problem is as follows,
Integral of ( 1+ sum of x^k, k=1 to k=n), x=0 to x=b *dx.
The answer is b + sum of b^(k)/k, k=2, to k=n+1. I understand only the integral of first term. But integral of the sum I do not understand at all.

Answered by Penny Nom.
The area of a rectangle 2016-03-03
From Lucas:
The area of a rectangle is given by A=x2+18x+72
-use factoring to find an expression for the dimensions of the rectangle.
-if the area of the rectangle is 7 square feet, what are the possible values of x?
-what are the dimensions of a rectangle?

Answered by Penny Nom.
Corrosion inhibitor in a pipeline 2016-03-02
From Edu:
I have a 12 in pipeline and 5 miles long. I need to coat it with corrosion inhibitor how much chemical will I need
Answered by Penny Nom.
A rectangular prism 2016-03-02
From Paula:
Trying to help with a math problem. I don't know how to set up the equation for this.

A rectangular prism has a volume of 7.875m3rd and a height of 3.5 m. What is the area of the base of the prism?

Thanks for your help!

Answered by Penny Nom.
An algebra problem with a sphere 2016-02-29
From Sania:
If the number of square centimeters on the surface area of a sphere is equal to the number of cubic centimeters in its volume find its diameter
Answered by Penny Nom.
10800 sq. ft equal approx what part of an acre? 2016-02-27
From David:
10800 sq. ft equal approx what part of an acre
Answered by Penny Nom.
The sum of the angles of a triangle 2016-02-24
From Sophia:
Does every triangle add up to 180 degrees? (Such as a unique triangle)
Answered by Penny Nom.
The weight of some dirt 2016-02-24
From Michael:
How many tons of dirt are in a triangle 14M x10M x 8.1M x .8M deep?
Answered by Penny Nom.
The domain and range of a function 2016-02-19
From Genius:
State the domain and range g(x)=x(x-1)
Answered by Penny Nom.
The interior and external angles of a regular polygon 2016-02-17
From percy:
a regular polygon has n sides .The size of each interior angle is eight times the size of each exterior angle .
1.find the size of each exterior angle
2.calculate the value of n

Answered by Penny Nom.
The equation of a vertical line 2016-02-07
From Kagiso:
The equation for a vertical line that goes through the point (7; -3) is?
Answered by Penny Nom.
The midpoints of the sides of a quadrilateral 2016-02-05
From Andrea:
The segments, joining, in order the midpoints of consecutive sides of a quadrilateral form a parallelogram.
Answered by Penny Nom.
The height of a triangle 2016-02-03
From Brooklyn:
How do I figure out the height of a triangle when it only tells me the base of he triangle? And vice versa.
Answered by Penny Nom.
The diagonals of a polygon 2016-01-29
From Sofia:
What are diagonals and if you drew an octagon and drew all the diagonals how many would there be? Please explain because this is pretty confusing! Thank you!
Answered by Harley Weston.
Consecutive angles of a parallelogram 2016-01-28
From Hanna:
The consecutive angles of a parallelogram measures
Answered by Penny Nom.
The range of a function with a finite domain 2016-01-25
From Hannah:
Solve y=(1⁄4)x-1 if the domain is (-4,-2,0,2,4). I don't get how to do this. Can anyone help?
Answered by Penny Nom.
A system of linear equations 2016-01-24
From kareem:
my name is kareem
and i am a student i have a math puzzle and i tried to solved it but it always have same mistake
x-y=9
x+z=12
z-n=14
y+n=2

Answered by Penny Nom.
Fractions of two quantities 2016-01-22
From Melody:
Kate ate 1/4 of her orange. Ben ate 2/4 of his banana. Did Kate and Ben eat 3/4 of theit fruit? Explain.
Answered by Penny Nom.
The perimeter of a square 2016-01-16
From dennis:
If I have eleven Square Acres, what would the perimeter dimensions be?
Answered by Penny Nom.
A limit of a rational function 2016-01-16
From selvamani:
F (x) = x^3+3x^2-9x-2 / x^3-x-6 and limit x tends to 2, f (x) exist then limit x tends to 2, f (x) is equal to ? How to answer this problem.
Answered by Penny Nom.
Complex numbers in standard form 2016-01-15
From Michael:
express the following complex numbers in standard form (2+3i)+(5-2i)
Answered by Penny Nom.
The range of a function defined piecewise 2016-01-13
From sarah:

f(x)={x-2, x<-2
       {x^2-4, -2_<x_<2
       {x+2, x>2

find the range
Answered by Penny Nom.

The perimeter of a triangular plot of land 2016-01-11
From Janelle:
I need to fence off 6.5 acres. The plot is triangular shape. How many feet would I be fencing off?
Answered by Penny Nom.
The area of a rectangle 2016-01-08
From ranjan:
if we make a rectangle with a 100cm of wire, then its area will be?
Answered by Penny Nom.
Two transport companies 2016-01-06
From Brynleigh:
The Sugar Sweet Company will choose from two companies to transport its sugar to market. The first company charges $5096 to rent trucks plus an additional fee of $200.75 for each ton of sugar. The second company charges $5500 to rent trucks plus an additional fee of $175.50 for each ton of sugar. For what amount of sugar do the two companies charge the same? What is the cost when the two companies charge the same?
Answered by Penny Nom.
A graph of the water level of a river 2016-01-05
From emoni:
Suppose that the water level of a river is 34 feet and that it is receding at a rate of 0.5 foot per day.
Answered by Penny Nom.
The height of a parabolic arc 2015-12-30
From Tom:
Is there an algebraic means to determine the highest point of a parabolic arc if the base and perimeter are known?
Answered by Penny Nom.
Strings of characters 2015-12-28
From Ali:
If I have an app and offer a two, three and four digits name and it can be only number, letters or both?
I want to know how many options will I have for each?
For example
the two digits: aa, 11, a1 or 1a
the three digits: 111, a2a, 222
the four digits: 1111, 2ge3 or 1234
So if I used all the letters and all the numbers but nothing starts with a zero How can I calculate them separately? Or what's the result for each?
Thank you
Ali

Answered by Penny Nom.
A diameter of a circle 2015-12-27
From sahil:
Find the equation of diameter of circle x2 +y2-6x +2y-8 which passes through the origin.
Answered by Penny Nom.
The angles of a triangle 2015-12-17
From Faith:
Does the measure of angle determine the length of its side? For example two angles are congruent then the sides are also congruent because from my understanding the angle determine the shape of triangle.
Answered by Penny Nom.
Proof that an erroneous algebraic statement is false 2015-12-14
From Berteanu:
I need help with this proposition:
"It exists x a real number that for every y real number 5*x-2*y*y=1
This is false.
Let x be from R.
And I need an y real number that 5*x-2*y*y!=1
Please,could you help me?

Answered by Penny Nom.
The line of intersection of two planes 2015-12-09
From Rabz:
Determine the vector equation for the intersection between the two given planes 2x+7y equal to 14,x+z equal to 7
Answered by Penny Nom.
Drawing an arc 2015-12-04
From hassan:
how to draw a curve long 1m with an angle of 22.5?
Answered by Penny Nom.
The height of a truncated cone 2015-12-04
From Jack:
I need to build a truncated cone that has a top of30 inches and a base of 64 inches . The sides need to be at a 64.5 degree angle. This will determine the height. Can this be calculated?
Answered by Penny Nom.
The sweep angle of a video camera 2015-11-25
From Franc:
a television camera is 30ft from the sideline of a basketball court 94 ft long. The camera is located 7 ft from the mid court. through what angle must it sweep in order to cover all action on the court?
Answered by Penny Nom.
The range of f(x)=(2x+2)/(x-1) 2015-11-25
From Nazrul:
f : R to R is a function which is defined by f(x)=(2x+2)/(x-1).
Is f(x) an onto function? Please explain.
What is the range of this function?
Thank you very much for your previous help.

Answered by Harley Weston.
How much does the Earth curve over a one foot distance? 2015-11-24
From Sean:
Hi, I am trying to figure out how much the earth curves over a one foot distance. I'd like to be able to draw the exact arc on a piece of paper. I am an artist and am looking to make glass vessels with the exact curvature of the earth. I read on your site that it curves approximately 8 inches per mile. can I just use simple ratios to break it down into inches?? Thank you so much for your help.
Answered by Harley Weston.
The measure of an angle in terms of its complement 2015-11-22
From Pam:
Can you please help me so I can help my daughter the equation is the measure of angle v is 4 time the measure of its complement what is the measure of angle v when the equation is 4x+x=90
Answered by Penny Nom.
20 million as a percentage of 7.3 billion 2015-11-18
From Nick:
Based on the estimate of the worlds current population (7.3 billion people), I'm trying to work out what percentage of that figure 20 million people would amount to?

(Apologies in advance that I've no idea if the 7.3 billion figure is an American Billion or UK billion)

Thanks for any help.

Nick.

Answered by Penny Nom.
The area of a domed roof 2015-11-13
From Brandon:
I have a tank with a 13' radius that has a domed roof of 3.5' tall How do I figure out the area in SQFT?
Answered by Penny Nom.
A concrete container 2015-11-12
From Karstin:
The exterior of a concrete container will be 10 feet by 8 feet by 4 feet tall. The walls and the bottom are 6 inches thick. What will it cost to construct it if concrete is $98.95/cubic yard?
Answered by Penny Nom.
Roof Square footage 2015-11-11
From Todd:

Question from Todd:

Good Day.
I have to figure out the square footage of a quonset style roof that's not playing by the rules The building dimensions (rectangular) are 63'x135' the height of the roof is 9.25'. It not an entire Quonset, It's that style of roof,(curved). There are concrete block walls 10' up to the metal roof.

Thank you!

Todd


Answered by Harley Weston.
The derivative of x^1/3 2015-11-08
From omar:
hi can help me
am teacher ask me about x^1/3 Derivation definition .

Answered by Penny Nom.
The domain of cos2x/(sinx-2) 2015-11-02
From Ameen:
Find the domain of:
f(x) = cos2x/(sinx-2)

Answered by Penny Nom.
The sum of two repeating decimals 2015-10-22
From Kaitlin:
Here is the question I am struggling with:

Is the sum of two repeating decimals always repeating decimal? Explain your answer and give an example.

I answered this question thinking that no you cannot but only when adding 0.9 repeating, but my professor said this was incorrect.

Thanks for the help!
-Kaitlin

Answered by Penny Nom.
The length of a rectangle 2015-09-26
From Tris:
Hi,
I'm trying to figure out the length of a rectangle but i forgot how. So, the width is 5cm and the perimeter is 34cm.

Answered by Penny Nom.
Order of operations 2015-09-12
From Tanisha:
I would just like to double check if something like 5x squared times 4x cubed equals 20x to the power of 5??
It's just that we were told you can only answer an equation like this if the base is the same...so does that mean the x part or the whole thing like 5x? I'm sorry if that didn't make sense!

Thank you for your help!!

Answered by Penny Nom.
The diameter and circumference of a circle 2015-09-01
From Tracey:
Hello,
I own a custom workroom, I am figuring out fabric quantities or an estimate. When at the job site, I forgot to measure the diameter of the semi circle shape that I have to make a cushion for!!!
If the circumference of the semi circle is 165" what would the diameter be, maybe half this measurement????

I appreciate any help...I basically need the width from left to right to see if I can fit the cushion on one width of fabric that is 52" wide!

Answered by Penny Nom.
The distance over a Quonset 2015-08-20
From jane:
total base of hemisphere is 30 ft apex height is 20 feet what is total length over dome
Answered by Penny Nom.
Parallelogram area 2015-08-04
From Nigel:
To find the area of a parallelogram you multiply the base by the height of the parallogram, the height being determined by an imaginary line drawn at right angles to the base. However if I made a parallelogram with joints at each corner and maipulated it to an upright position where the base and sides were at 90 degrees to each other,, I could then simply multiply the base times the height to get the area. Since the sloping side of the parallelogram does not vary in my imaginary jointed model, why can I not simply multiply the base by the sloping side to get the area in the same way that I multiply the base times the height of a square to find its area ? I cannot find the answer to this on the internet although I have searched.
Answered by Chris Fisher.
Why is the area of square not conserved when it changes to a rhombus? 2015-06-28
From shubham:
Why is the area of square not conserved when it changes to a rhombus, both have equal sides still rhombus have less area than square.??
Answered by Penny Nom.
1 + 1 + 1 + 1 + 1 + 1 + 1 + 1 + 1 + 1 + 1 + 1 x 0 + 1 = ? 2015-06-18
From Sharon:
1 + 1 + 1 + 1 + 1 + 1 + 1 + 1 + 1 + 1 + 1 + 1 x 0 + 1 = ?

I got 1 as my answer despite BODMAS making it 12 because logic tells me I ought to place brackets around the first set of repeated addition. Could you please clarify this for me? Thank you 😊

Answered by Harley Weston.
2^48 - 1 2015-06-13
From Soham:
The number 2^48-1 is divisible by two numbers between 60 and 70. The sum of the two numbers is?
Answered by Penny Nom.
The amount before profit and overhead 2015-06-12
From kelly:
I have an existing amount of $22,750.00. this includes the profit at 10% and overhead at 10%. How do I remove the profit and overhead in order to come up with the original starting figure?
Answered by Penny Nom.
The surface area of a triangular prism 2015-06-03
From Alex:
Find the surface area of a triangular prism with Isoceles triangles and dimensions 4 feet by 6 feet
Answered by Penny Nom.
The volume of a pipe 2015-06-02
From gordon:
What is the formula to find out how many gallon of water in a length of pipe tried your answer on 8inch at 50feet did not work out
Answered by Penny Nom.
A pentagon inscribed in a circle 2015-05-30
From Victoria:
find the area of a regular pentagon inscribed in a circle with radius 3 units
Answered by Penny Nom.
The perimeter of a rectangle 2015-05-28
From Imran:
The length of a rectangle is x+3 centimetres.
The width of the rectangle is x-1 centimetres.

Find an expression in terms of x for the perimeter of the rectangle.
The perimeter of a rectangle The perimeter of a rectangle
Give your expressions in it's simplest form.
The perimeter of a rectangle The perimeter of a rectangle
Thank You.

Answered by Penny Nom.
Prove that you cannot factor x squared + 5 2015-05-28
From lily:
the question is: prove that you cannot factor x squared + 5
Answered by Robert Dawson.
The perimeter of a quarter circle 2015-05-21
From Bethany:
The perimeter of a quarter circle is 3.57. What is the quarter circle's radius?
Answered by Penny Nom.
The intersection of a plane and a cone 2015-05-16
From Tom:
Is there a way to derive an equation that describes the perimeter of the intersection of a plane and a cone regardless of the angle of the plane to the cone. Assume that the plane does not cut through the base of the cone, the x, y, z location of the vertex is known, the distance from the vertex to the plane through the axis is know., and that the angle of the cone is known.
Answered by Chris Fisher.
A pile of mulch 2015-05-15
From Justin:
I have a pile of mulch that measures 19 feet wide at the bottom and has a height of 6 feet. I need to know how many yards of mulch I have.
Please help and help with the formula for further use. Thank you!

Answered by Penny Nom.
An angle of depression problem 2015-05-14
From haxir:
find the height of the balloon directly above a town if the measure of angle of depression of another town 5km from the first town is 20°!6`
Answered by Penny Nom.
A word problem 2015-05-08
From JAMES:
A businessman bought a number of similar articles for a total of sh. 6000. Three of the articles turned out to be defective with no resale value. He sold the remaining articles at 12.5% each above the cost price making a total profit of sh 480. How many articles had he bought?
Answered by Penny Nom.
The surface area of a hemisphere 2015-05-05
From Mandy:
How can I show that the total surface are of any solid hemisphere is 75% of the area of the full sphere?
Answered by Penny Nom.
The method of elimination 2015-05-01
From oreanna:

Question from oreanna, a student:

How do u solve 2x+9y=3

7x-4y=-25 in elimination


Answered by Penny Nom.
Filling a pool with dirt 2015-05-01
From Mike:
I have a hole which a 24 ft pool in it is 10" deep in the the centre and goes to 1" inch at the edge want to fill it in with dirt how many yards of dirt would I need to fill it in
Answered by Penny Nom.
The volume of a sphere 2015-04-30
From Cassidy:
How do you find the radius of a sphere that has volume 36pI?
Answered by Penny Nom.
The volume of a cube 2015-04-30
From megan:
What's the volume of a cube with a side length of 6?
Answered by Penny Nom.
The surface area of a cut pipe 2015-04-27
From ARUN:
dear Sir,

please advise me , how to calculate the surface area of a pipe with diameter of 630 mm and thickness of 67 mm which is cut in a angle of 22.5 degree.

please show me how to calculate the surface area of the pipe which cut in an angle.

Thanking you.

Answered by Harley Weston.
A piecewise function 2015-04-25
From uday:

f(x)={x^2+3x+2 / x+2, x(not equal to)=-1
{4, x=-1
how to find domain and how the graph looks like


Answered by Penny Nom.
Locating the center of a circle that forms an arc 2015-04-23
From Ken:

Find the Cartesian coordinates of the center of an arc with the given location of the beginning and end points and radius length. Not the midpoint of the circumference but the actual point that the arc
is drawn around.

I know their are two answers depending on the direction of the arc. Unless we assume that all arcs are drawn counter clock wise.

Thanks
Ken


Answered by Harley Weston.
Two cars approaching an intersection 2015-04-16
From Engabu:
Car A is traveling west at 50km/h & car B is traveling north at 60km/h. both are headed for the intersection of the two roads. At what rate are the cars approaching each other when car A is 3km & car B is 4km from the intersection?
Answered by Penny Nom.
Two airplanes 2015-04-14
From john:
two planes leave an airport at the same time, one going northwest (N35*W)at 400 mph and the other going east at 332 mph. How far apart are the planes after 4 hours to the nearest mile?
Answered by Penny Nom.
The area of a circle given the circumference 2015-04-12
From F:
How do you find the area of a circumference of 6?
Thanks.

Answered by Penny Nom.
The perimeter of a quarter circle 2015-04-10
From Riley:
How do you find the perimeter of a quarter circle when the radius is 56?
Answered by Penny Nom.
On what day of the week did 1994 begin? 2015-04-05
From tayyaba:
there were 365 days in the year 1993 the first day of the year was friday. on what day of the week did 1994 begin?
Answered by Penny Nom.
The weight of a steel block 2015-04-04
From Mark:
the weight of a steel cube 37 inches X 28 inches X 7 inches
Answered by Penny Nom.
n^2 is a multiple of 100 2015-03-30
From Rahul:
I have to prove that n^2 is a multiple of 100 is necessary or Sufficient condition (or both) for n being multiple of 10
Answered by Penny Nom.
The game of 24 2015-03-28
From Suzanne:
I love this game! Cannot figure out 4,9,10,16 to equal 24? Help!
Answered by Penny Nom and Claude Tardif.
The weight of two dogs 2015-03-17
From Renee:
Darlene's dog weighed 5 times as much as Leah's dog. Together, the dogs weighed 84 pounds. How much did each dog weigh? Write an equation and solve
Answered by Penny Nom.
A cone of maximum volume 2015-03-16
From Mary:
I have to use a 8 1/2 inch by 11 inch piece of paper to make a cone that will hold the maximum amount of ice cream possible by only filling it to the top of the cone. I am then supposed to write a function for the volume of my cone and use my graphing calculator to determine the radius and height of the circle. I am so confused, and other than being able to cut the paper into the circle, I do not know where to start. Thank you for your help! -Mary
Answered by Robert Dawson.
The height of an equilateral triangle 2015-03-12
From anna:
I am anna and i am in 7th grade.
i am trying to find the height of and equilateral triangle, all sides equaling 4 inches

Answered by Penny Nom.
The area of a quadrilateral 2015-03-11
From Joel:
Diagonal ac of quadrilateral ABCD is 60cm and the lengths of perpendiculars to It from the opposite vertices are 4.2cm and 5.8 cm find the area of the quadrilateral ABCD
Answered by Penny Nom.
Angles of elevation and depression 2015-03-08
From Timmy:
Joel is walking down a street and sees a 115 ft tall building in front of him. He stops 190 feet from the base of the building at the tip of the building's shadow. Round answers to three decimal places.

A. If there was a piece of rope from the top of the building to Joel, how long would it be?
B. What is the angle of elevation from Joel to the top of the building?
C. Margaret says that she could find the angle of depression from the top of the building to Joel by subtracting the angle of elevation from 90°. Is she correct? Explain.

Answered by Penny Nom.
A man is standing on top of a building 2015-03-06
From kiki:
a man of height 1.5m standing on top of a building of height 48.5m views another building across the square. he observe that the angle of depression of the bottom of the building is 40 degrees and the angle of depression of the top of the building is 5 degrees. Both buildings stand on the same level
a) calculate the distance of the man from the base of the building across the square measured along the line of sight

Answered by Penny Nom.
Solve x=r(t-sin(t)) for t 2015-02-28
From David:
I don't know how to write the function of t(x) by x=r(t-sin(t)).
Can you teach me that?

Answered by Harley Weston.
The radius of a cylinder 2015-02-26
From Rose:
Hi. I want to know how to find the radius of cylinder.
When I have height (35cm) and area of the curved surface(880sq.cm).
I know the formula is 2πr(h+r). But I can't understand how to find it's radius.
Please help me. I really need your help.

Answered by Penny Nom.
A limit 2015-02-19
From genc:
Lim (27x^3-1) / (3x-1)
X-> 1/3

Answered by Harley Weston.
The center and radius of a circle 2015-02-06
From ariana:
I need to find the center and the radius of this circle. I don't know how to put 9/2 than square it into a fraction.

x^2 + 2x + y^2 +9y=5

Answered by Penny Nom.
The volume of a cone 2015-02-05
From Linda:
How do you calculate the volume of a cone that is 25cm high and has an angle of 20 degrees?
Answered by Penny Nom.
Ratio of gas to oil 2015-02-02
From Chris:
I would like to find the gas oil ratio for a 2 cycle engine. I have the gallons of gas and the ounces of oil used by the engine, but I want to find what the ratio is. For example I used 5.675 gallons of gas and 5 ounces oil what is the ratio? Please explain step by step, and I understand there are 128oz in a gallon.
Answered by Harley Weston.
128/(-16)/(-2) 2015-01-28
From jackie:
128/(-16)/(-2) I was wondering if you can show me how to work this question out
Answered by Harley Weston.
Scheduling meetings with pairs of people 2015-01-22
From Jacey:
I am trying to figure out a formula/system to pair up a list of people so they meet with each other every month, but they rotate who they meet with. Right now I have 13 people and I would like to just type in their names and then have the system put each person with someone else every month and rotate so no one gets the same person twice. Can this be done?
Answered by Victoria West.
The game of 24 2015-01-21
From aaron:
24 game whants me to use 11 14 5 and 9 to make 24 allowed (+ - dividing and multiplying)
Answered by Penny Nom.
The height of a building 2015-01-20
From Emily:
A man 2m high observes the angle of elevation to the top of the building to be 70 degrees. And the angle of depression to the bottom of the building to be 19 degrees. How tall is the building?
Answered by Robert Dawson.
The length of a roll of paper 2015-01-12
From Peggy:
roll of paper 3ft wide and roll equals 500 sq ft how long would the roll be?
Answered by Penny Nom.
A difference quotient 2015-01-12
From Sasha:
Simplify the difference quotient

f(x) − f(a)/ x-a
if x ≠ a.
f(x) = x^3 − 12


Answered by Penny Nom.
The volume of a lake 2015-01-10
From Peter:
If a lake is 8 acres in size and 10 feet deep how many gallons of water does it contain
Answered by Penny Nom.
Selling items at a reduced price 2015-01-09
From Kent:
I have an item I sell for $19.95 and make a profit of $10.97 for each sale. If I reduce the price to $14.97, my profit is $5.99 each sale. Is there a formula to determine how many more I would need to sell at $5.99 per sale to make the same net cash (profit) as selling at $10.97 per sale?
Answered by Penny Nom.
The units digit of a prime 2015-01-08
From Patricia:
Every prime number greater than 10 has a digit in the ones place that is included in which set of numbers below 1 3 7 9 or 1 3 7 or 0 2 4 5 6 8 or 1 3 5 9
Answered by Walter Whiteley.
The degree of a polynomial 2015-01-05
From Nichole:
How do I determine the degree of polynomials? I've searched this on sites but they are all so confusing! Is there a simple explanation or way to find what the degree is? Some examples are: 6x^4 10x^2yz^5 and 3m^2n^7-10m^8. I also have to say that I am under the impression that this symbol ^ means the number after it is an exponent.
Answered by Robert Dawson and Harley Weston.
1.5% of 1 trillion 2014-12-30
From Kgosi:
How much is 1.5% of 1 trillion?
Answered by Penny Nom.
Curvature of the Earth 2014-12-29
From Jimmy:
Both batteries died in my scientific calculator and I have lost my formula for the heigth of the curvature of the earth between two points on the surface. I used degrees and miles. I only had to enter the distance between the two points on the surface and the formula gave me the hieght the earth raised between the two points.
Answered by Robert Dawson.
The perimeter of a quarter circle 2014-12-26
From Maisy:
This is the question that I got for a math worksheet.
The radius of a quarter circle is 5 miles. What is the quarter circle's perimeter?
Use 3.14 for pi.

Answered by Penny Nom.
A line segment of length root 5 2014-12-15
From angela:
On the dot grid below, draw and label a line segment with length square root 5 the dot grid is 8 by 10
Answered by Penny Nom.
Planar curves 2014-12-13
From ann:
what does planar curve mean in your definition of a cone?
Answered by Penny Nom.
30% profit 2014-12-05
From Lyn:
I would like to know a formula that I would use on a standard calculator to calculate a 30% profit margin on an item. If I purchased an item at $7.70 how would I calculate 30% profit for me, not just 30% mark up?
Answered by Penny Nom.
1999 equations in 1999 unknowns 2014-11-28
From Subrahmanya:
Solve the following system of 1999 equations in 1999 unknowns :

x1+x2+x3=0, x2+x3+x4=0……., x1997+x1998+x1999=0,

x1998+x1999+x1=0, x1999+x1+x2=0.

(in x1,x2,...................,x1999 the numbers 1,2,.............,1999 are subscripts)

Answered by Penny Nom.
What is 20 and 3/5 of 10? 2014-11-26
From Jeannine:
What is 20 and 3/5 of 10? Question is written exactly this way. Is this 20 3/5 times 10 (103/5 times 10), or is it 20 and (3/5 times 10)?
Answered by Penny Nom.
Doubling the dimensions of a cone 2014-11-21
From Hannah:
If the volume of a cone of height 10 cm is 261.8 cm3, show that this volume is increased by a factor of 8 if the dimensions of the cone are doubled.
Answered by Penny Nom.
(x-3)^2-(x+3)^2 2014-11-13
From Bernice:
(x-3)^2-(x+3)^2
Answered by Penny Nom.
What is -3 squared? 2014-11-12
From Christine:
On a math test, it said "What is -3 squared?" It did NOT say "What's is (-3)squared"

The teacher's explanation is that if there are no brackets or parenthesis, you ALWAYS square the number first then do the negative, so the answer should be -9, but I can't find anywhere that confirms this.

Help please.

Answered by Robert Dawson.
Two quadratic polynomials 2014-10-28
From khushboo:
The zeroes of a quadratic polynomial x^2+ax+b are 'c' and 'd' and the zeroes of a quadratic polynomial x^2+cx+d=0 are 'a' and 'b'. find the numerical value of a+b+c+d where a, b, c and d are non zero integers.
Answered by Penny Nom.
A 10-foot piece of PVC pipe 2014-10-21
From David:
A plumber has a 10-foot piece of PVC pipe. How many 9/5-foot pieces can be cut from the 10-foot piece ?
Answered by Penny Nom.
Sieve of Eratosthenes 2014-10-03
From Hope:
using Sieve of Eratosthenes to determine for in instance prime numbers between 1 to 200, to what nth number should i stop?, 4th, which is 11 or what?
Answered by Robert Dawson.
The perimeter of a parcel of land 2014-09-18
From Shirley:
What is the perimeter of a parcel of land that is 564 acres square
Answered by Penny Nom.
A tangent to a curve passing through a point not on the graph 2014-09-15
From Aquilah:
For the curve y = x2 + 3x, find the equations of all tangent lines for this graph that also go through the point (3, 14).
Answered by Penny Nom.
The diagonals of a cube 2014-09-04
From Rukshanth:
How many diagonals does a cube have?
Answered by Harley Weston.
The sum of two numbers is 52 2014-09-01
From Blake:
The sum of two numbers is 52 and the difference is 10. What are the numbers? i used to be real good at this stuff?
Answered by Penny Nom.
The perimeter of a triangle 2014-08-21
From Phyllis:
What is the perimeter of a five acre triangle with one side being 250 feet and the second side would be at a 90 degree angle.
Thank you.

Answered by Penny Nom.
The equation of a circle 2014-08-14
From jennifer:
hi there My name is Jennifer and residing in Denmark. I am a student and I wrote to you because i am having trouble in finding out the equation for the circle using (x-a)^2 + (y-b)^2 =3D r^2.The diameter of this circle is d=3D 44,514 cm. I have attached a drawing of my problem..thanks
Answered by Penny Nom.
Ratios ond percents 2014-08-13
From aswathi:
if the ratio of cost price and selling price of an article is be as 10:11,the percentage of profit is
Answered by Tyler Wood.
The area of a quadrilateral 2014-08-06
From Rahul:
find the area of the quadrilateral whose side measure9cm,40cm,28cm,15cm and in which the angle between the first 2 sides is a right angle
Answered by Penny Nom.
The area of a kite 2014-08-04
From Janis:
How do you calculate the area of a kite with the diagonals 24cm and 48cm?
Answered by Chris Fisher and Penny Nom.
Expand and simplify [x-3][x+3] 2014-07-16
From symion:
Expand and simplify [x-3][x+3]
Answered by Penny Nom.
A 5 sided lot 2014-07-10
From mia:
have the feet numbers of three sides, but the forth side is in a "V" so it has two figures. Question: can i add the two figures of the "v" together to get one figure, so then I would have the figures for all four sides.
Answered by Harley Weston.
A ladder against a wall 2014-07-09
From thabo:
A ladder 6.5m long,leans against a wall so that the top of the ladder is 4.8m from the ground.what is the angle of elevation of the ladder to the top of the wall
Answered by Penny Nom.
The method of elimination 2014-07-05
From leo:
please explain how can i solve this problem

3x-6y=-38
6x-9y=44

using elimination and simultaneous method thank you :)

Answered by Penny Nom.
The width of a rectangle 2014-07-02
From john:
if the area of a rectangular field is 90 sq. m and its length its is 19 m find its width
Answered by Penny Nom.
A triangle on the surface of the Earth 2014-06-26
From Christine:
A, B and C are three towns, the bearing of B and C from A being 310 degrees and 220 degrees, and their distances from A are 510km and 700km respectively. Find the bearing of B from C to the nearest minute.
Answered by Robert Dawson.
50+9-13 multiplied by 2= 2014-06-25
From Adrian:
50+9-13 multiplied by 2=
Answered by Penny Nom.
An oval pool 2014-06-21
From steve:
I have a 16' x 28' oval pool that is buried 24" deep inground. The dig site is dug 2' wider all the way around the pool. I need to back fill this area with stone. I want to fill this area with 6 to 8" of stone. How many tons of stone will this take?

Thanks you
Steve

Answered by Penny Nom.
The number of standard deviations 2014-06-15
From Dawn:
Suppose that, at a certain college, the average weight of all male students is 160 pounds with a standard deviation of 30 pounds. A certain male student weighs 145 pounds. Determine how many SD's his weight is above or below average.
Answered by Penny Nom.
Profit 2014-06-14
From roudha:
find the profit of one item if it was bought for AED 180 and sold for AED 275. assume that the additional costs are AED 25 .
Answered by Penny Nom.
The area of a circle of circumference 32.69 meters 2014-06-11
From coco:
Find the area of a circle with a circumference of 32.69 meters.
Answered by Penny Nom.
Bags of sand 2014-06-06
From Robert:
We have a section that is 20ft long by 14ft wide and 4inches deep, the bags of sand are .5 cubic ft. How many bags of sand do we need to fill this space evenly.
Answered by Penny Nom.
The volume of a wedge 2014-05-21
From steve:
I need to figure the volume of a wedge / right triangle, the dimensions of the right triangle is L1(10'-5"), L2 (10'-6"), H (14'-9.5") height of L1 is 0'-0" height at intersect L2/H is 3'-3"
Answered by Penny Nom.
The equation of a line 2014-05-16
From Michael:
Find the equation of a line that passes through (2,-1); the sum of the x- and y-intercepts is 2. (There are two answers)
Answered by Penny Nom.
A schedule for 2 teams of 10 2014-05-07
From John:
How can I set up a schedule for 2 teams of 10 on a team to play each individually against each other over a 10 day span at 10 different course with 10 different start times each day. I was looking to not repeat any competition. As an example, Team A would consist of players 1-10, Team B would consist of players 11-20. I would want to have 10 start times a day for 10 days where a member of team A plays a member of team B each day without repeating any matches and any of the same courses and not repeating any of the same start times. Any help in this would be GREATLY appreciated!!!!
Answered by Victoria West.
A cone of vision 2014-04-29
From David:
It is known that a fish in water looking up has a 97 degree "cone" of vision that sees "through" the surface of the water. If a fish lies 4 inches below the surface, the cone forms a window (circle) smaller than if a fish lies 8 inches below the surface. What is the ratio of inches of depth to the radius of the circle on the surface that is its visual window?
Answered by Penny Nom.
Profit and loss 2014-04-29
From Anoushka:
A man sells two tables of the same price. On one he makes a profit of 10 percent and on the other he suffers a loss of 10 percent.Find his loss or gain percent on the whole transaction.
Answered by Robert Dawson.
A triangular chicken pen 2014-04-27
From Cierra:
Margaret has two lengths of fence, 20 meters and 24 meters, for two sides of a triangular chicken pen. The third side will be on the north side of the barn. One fence length makes a 75° angle with the barn. How many different pens can she build if one fence is attached at the corner of the barn? What are all the possible lengths for the barn side of the pen?

Not sure what they are asking here... please show step by step what to do! Thank you so much!

Answered by Penny Nom.
The volume and radius of a sphere 2014-04-27
From grace:
How do you find the radius of a sphere when all you know is the volume?
Answered by Penny Nom.
The derivative of sin(x) 2014-04-26
From Lucky:
f(x)=Sin(x), by first principle its f'(x)...show me how to solve such problem.
Answered by Penny Nom.
What was the selling price? 2014-04-25
From Con:
Apple built 40 32GB wifi iPads, gave away two in a contest, and sold the rest at two times the manufacturing price. If the company's total profit was $10,782, what is the selling price of one of these iPads?
Answered by Penny Nom.
A quadratic equation given the roots 2014-04-23
From Sarah:
How do you find the quadratic formula when you're given just the roots ... X=-3 X= 1/3 Thanks !
Answered by Robert Dawson.
Golf for 6 2014-04-21
From barbara:
We are a group of 6 golfers. We play in groups of 3 for 5 days. How can I arrange the groups that everybody plays with everybody else at least once? Thanks.
Answered by Victoria West.
Golf with 3 teams of 20 2014-04-21
From Joshua:
Thank in advance for this great service

Golf - new - 3 teams of 20 - to create 20 unique groups of 3

For example use 3 teams of 3

1a 2a 3a
1b 2b 3b
1c 2c 3c

So groups are
1a 1b 1c
2a 2b 2c
3a 3b 3c

Groups can only have one a, one and one c - and that combo should be unique

Answered by Victoria West.
Barrels of oil 2014-04-16
From daryl:
Looking for an accurate number to use when calculating barrels per inch in a a tank with the following dimensions:
Nominal diameter - 15'-6", nominal height - 16ft, nominal capacity - 500 barrels. These are commonly referred to as 500x16 tanks.
Currently we are using 2.6 barrels per inch. We need 180 barrels for a full load so I'm trying to figure out how many total inches I need to load 180 bbl. This has been a matter of debate for some time. Thanks for your help.

Answered by Harley Weston.
The surface area of a circular dome 2014-04-10
From Shafiqah:
Is this a dome's surface area formula??
{{2 × π r × h square units}}
Is the surface area of the floor for the dome is calculate too in this formula?

Thanks for answering. =)

Answered by Robert Dawson and Penny Nom.
The sides of a triangle 2014-04-06
From Michael:
I am supposed to sove for the length of side "b" of an irregular triangle. I am given the following:
Side a: 65'
Side b: Find this length
Side c: 50'
Angle A: unknown
Angle B: unknown
Angle C: 52 degrees
I am supposed to use the law of cosines to solve for side "b" and my teacher says there is no mistake in the "givens" for the problem. I do not see how this can be done using the law of cosines and i have not figured out how to sove for angle B to use the law of cosines.

Answered by Penny Nom.
The Pythagorean Theorem 2014-03-30
From brenae:
The Pythagorean Theorem, what is it?
Answered by Penny Nom.
A schedule for 6 people 2014-03-29
From John:
How do I set up a schedule where six people are here for ten of twenty days. Arranged in rotating groups of three, so everyone works with everyone else. Everyone works with everyone else at least once and everyone works ten days.
Answered by Victoria West.
Curvature of the Earth 2014-03-28
From Max:

Recently I read the answer to a question proposed by someone on this site.

The question : What is the rate of curvature per mile on Earth?
The answer given : Use Pythagoras' Theorem to solve for the answer, given a 1 mile side
and a side as the radius. The hypotenuse minus the radius is your answer of drop/mile or curve/mile.

My conjecture : Why go through all of that work if the distance is one? Something like
{1/diameter} would would fine for such a problem. Seems like a lot of work for no reason.

I understand the practical application of Pythagoras' Theorem in this certain situation, as you would need
to use a^2+b^2=c^2 for any distance greater than one [mile]..
It just seems excessive and unnecessary if you're solving for curve / one mile.


Answered by Robert Dawson.
A table of values 2014-03-27
From Marisol:
A table value X 0,1,2,3,4,5,6 and Y 3,7,11,15,19,23,27 . what relationship of X to Y is represent by which equation? A . y= 4x. B y =x+3 . C y =4x+3 or . D y =4x-3
Answered by Penny Nom.
Two overlapping arcs in a square 2014-03-15
From Jean:
I have a square with side 4 cm. There are two overlapping arcs going from vertex to diagonal vertex. The other two vertices are the center of the arcs, which are shaded. How do I find the area of the shaded arcs? The overlapping arcs when shaded resemble a long thin football
Thank you for your help.

Answered by Penny Nom and Walter Whiteley.
The angle of depression 2014-03-08
From Ranger_minor:
A woman of height 1.4 metres standing on the top of a building 34.6 metres high views a tree some distance away. she observes that the angle of depression of the bottom of the tree is 35 degrees and the angle of depression of the top of the tree is 29 degrees.

assume that the building and the tree are on level ground :
1). calculate the distance of the woman from the top of the tree measured along her line of sight.

Answered by Penny Nom.
The equation of a circle 2014-03-07
From Balraj:
I have to draw a circle x^2+y^2 =8x . please tell me how to understand these coordinates ? please elaborate , on how to understand these types of equations.
Answered by Penny Nom.
The diameter of the base of a cone 2014-02-23
From elwin:
i have a sector of a circle that has 120 degree and 6 cm length. What will be the diameter of the base of the cone.
and what is the diameter of the angle is changed to 180.

Answered by Harley Weston.
Three investment partners sharing the profit 2014-02-22
From Ayatullah:
Hello;

We are three partners and bought a property at 2050000 and sold at 2300000.
Details of investment of the partners
X= 1500000
Y= 400000
Z = 150000

My questions are
1) How could i calculate percentage of investment of each partner?
2) How could i deduct 30% commission from each partner in profit?
3) How could i distribute the profit amongst the partners?


Thanks in advance

Answered by Penny Nom.
The growth of a tree 2014-02-16
From Emily:
If a tree is 10 ft tall after 2 years and 25 ft tall after 5 yrs , find the rate of change of growth Of the tree if y is the height in ft and x is the number of years.
Answered by Penny Nom.
The grade of a road as a percent 2014-02-14
From Emily:
What is the grade of the road shown below expressed as a percent rounded to one decimal place?
Rise is 40feet
Run is 380 feet

Answered by Penny Nom.
The volume of a cone 2014-02-08
From hibba:
why is the volume of right circular cone divided by 3?
Answered by Penny Nom.
A triangle has angels in the extended ratio of 2:5:8 2014-02-06
From Rubina:
a triangle has angels in the extended ratio of 2:5:8. find the measure of all three angles?
Answered by Penny Nom.
Order of operations 2014-02-04
From Alex:
Hi there,i got problem with order of operation.Can you please help to find a solution for this?
Thank u beforehand
6-3(7+2)+(2+4)/3-4.

Answered by Penny Nom.
The volume of a frustum 2014-02-02
From mike:
volume of frustum R23", r 18", h 16"
Answered by Penny Nom.
Conics 2014-02-01
From Kassidy:
Hey, I have searched through all the questions about conics and how people use them in the real world, but none of them were very specific on how they are applied and the process, why it's so important etc. I have a project due asking these questions and it's been very difficult finding the right answer, if you could name jobs, how they are use and specifically applied that would be greatly appreciated.
Answered by Penny Nom.
$9.50 in coins 2014-01-15
From dana:
you have $9.50 in coins. it has twice as many nickels as quarters and three times as many dimes as nickels. how many of each coin do you have?
Answered by Penny Nom.
On what day of the week was July 4, 1904? 2014-01-11
From Madi:
Hi guys,
I have a question. July 4, 1903, was a Tuesday. On what day of the week was July 4, 1904? If you could give me an equation that would work for any question like this, that would be GREAT!
Your fellow math scholar (in training),
Madi

Answered by Penny Nom.
When would one flip the inequality sign? 2014-01-09
From Natasha:
Would one flip the inequality symbol in this equation: (explain why) (-9a)/(-9) > 81
And please explain in what circumstances one would flip the inequality sign
THANKS!

Answered by Penny Nom.
Profit as a percentage 2014-01-04
From Dane:
If i brought a card for £5 and sold it for £10 what is my profit percentage? 50% or 100%
Answered by Penny Nom.
25% profit 2014-01-02
From Finn:
Hello,
The question is all about buy-and-sell business.
Problem:
Pencil - $6 for whole sale price
     $8 if I sell the item How do I get the 25% profit? (you can change the whole sale price and the retail price[if i sell the item])
if I buy the pencil at 24 pieces and sell it at 24 pieces.

Answered by Penny Nom.
The slope of the line y=-2x+5 2013-12-29
From victoria:
what is the slope of the line with the equation y=-2x+5
Answered by Penny Nom.
Can 100r^2-81z^2 be factored? 2013-12-08
From Rosa:
Can 100r^2-81z^2 be factored?
Answered by Penny Nom.
Fractions and square roots 2013-12-04
From arionne:
How do you solve a square root with improper fractions like 121 over 49
Answered by Penny Nom.
What is the slope of this line? 2013-12-01
From Charlene:
What is the slope of this line?
Answered by Penny Nom.
How many cords of wood is this? 2013-11-17
From melody:
5.5 ft wide and 24ft long and 10ft wide how many cords of wood is this?
Answered by Penny Nom.
A sloped concrete wall 2013-10-30
From Brad:
We are planning on pouring concrete in a wall form that is sloped. One end is 73 inches tall, and it tapers to 24 inches tall over a distance of 348 inches. It is flat on bottom and sides and is 12 inches thick. Thanks.
Answered by Penny Nom.
The perimeter of a rectangle 2013-10-24
From sana:
if the length is given 70 m and the breadth is given as 40 m find the perimeter of the rectangle
Answered by Penny Nom.
The domain of a derivative 2013-10-10
From Renee:
I am looking to find the domain of a derivative of a radical function, one such as: f(x) = the square root of (8 − x).
I am kind of unclear on how domains work for derivative. I don't understand how you take a function's domain and use that to find the derivative's domain.
Thanks!

Answered by Penny Nom.
A line through (5, 1) 2013-10-08
From allison:
find an equation in standard form for line passing through point (5,1) and perpendicular to line x=4
Answered by Penny Nom.
A triangle problem 2013-10-02
From raneem:
ABC is a triangle in which : BC=20cm. M(<B) =29 and m(<C)=73 . D is the midpoint of BC Find the length Of AB and AD approximated to 2 decimal places
Answered by Penny Nom.
Two teams of 4 playing 4 rounds of golf 2013-09-29
From Tom:
We would like to play a ryder cup format with two teams of 4 playing 4 rounds of golf. Three of the rounds would be pairs playing each other and one round would be singles. How can we set up the foursomes so we balance the number of times we play with every other golfer.
Answered by Victoria West.
Price, revenue and profit 2013-09-22
From lorraine:
What price maximizes revenue? What price maximizes profit?

The only data I'm given is total output, total revenue, and total cost. I'm not sure how to set up a formula

Answered by Penny Nom.
The number of cords on a pile of wood 2013-09-18
From Gary:
a cord of wood is 4 feet by 4 feet by 8 feet, how much wood is a row that is 16 in by 5 foot by 32 feet?
Answered by Penny Nom.
If the earth was made of a ball of string 2013-09-18
From Tammy:
My 9 year old son asked me a question and I have no idea how to work it out! Please can you help?

If the earth was made of a ball of string and you unravelled the string how long would it be? Assuming the string is 5mm thick.

Answered by Penny Nom.
Using trig to find the height of a hill 2013-08-14
From Anna:
From the top of a hill, the angles of depression of two successive milestones on a level road, which leads straight away from the hill, are 5degrees and 15degrees respectively. Fine the height of the hill.
Suggestion: BE is drawn perpendicular to AD. Find BE, then BD, finally CD.

Thanks :)

Answered by Penny Nom.
A system of equations 2013-08-13
From Nina:
3x-2y=8
-x+3y=5
What is the solution of the system of equations

Answered by Penny Nom.
An equation in slope-intercept form 2013-08-12
From Brittney:
write an equation in slope-intercept form of the line that is parallel to the grapg of y=-3/4x+6 passes through -8,-5
Answered by Penny Nom.
Four equations 2013-08-08
From may:
HI how to solve this 4 equations?
A+C = 0
-4A+B-8C+D=1
3A+16C-8D=-29
-12A+3B+16D=5

Answered by Robert Dawson.
Practical uses of trigonometry 2013-08-06
From tharindu:
use of trigonometry
Answered by Penny Nom.
The angle of elevation of the sun 2013-07-03
From Maurice:
A vertical pole with a length of 7m cast a shadow with a length of 5m. Calculate the angle of elevation of the sun and include a diagram.
Answered by Penny Nom.
a cube-b cube-a+b= ? 2013-06-16
From saryu:
a cube-b cube-a+b= ?
find the answer

Answered by Penny Nom.
(4- 4cos^4 x)/(sin^2 x) 2013-05-18
From Agnes:
How I can solve this question :

Simplify (4- 4cos^4 x)/(sin^2 x) and write in terms of sin x

Answered by Penny Nom.
The surface of a solid 2013-05-07
From mustafa:
I am doing a research paper on solid surface applications . Part of the project is to find four at least in our world today and explain what there purpose is. I really need help in this area because I've been searching the internet for where solid surface are used in our world today and I really can't find anything.. Thanking you in advance, mustafa
Answered by Penny Nom.
Profit margin 2013-05-06
From Arron:
I have the retail price of $45.50. This price reflects a gross profit margin of 47%. How do I calculate the original cost of the item?
Answered by Penny Nom.
Points of intersection 2013-05-02
From Bianca:
Find the sets of values of k for which the line y=kx-4 intersects the curve y=x^2-2x at two distinct points.
Answered by Penny Nom.
Bags of pre-mixed concrete 2013-04-12
From Bruce:
Hello there,

I need two cubic yards of concrete for a playgound I am installing for my kids. That much i have figured out. I am stumped over the number of 25kg bags of pre-mixed concrete i will need to accomplish my goal.
Your help would be greatly appreciated.
Thanks
Bruce

Answered by Harley Weston.
The equation of a line 2013-04-04
From Miranda:
My coordinates provided are (6, 6) and (-8, 9) and i need to put this into ax=bx=c format. I remember the equation to find the slope of y2-y1/x2-x1 to get the slope. However when i put that back into the y=mx+b format it still leaves me with a lot of variables. Please help :)
Answered by Penny Nom.
A gravel pile in the shape of a triangular pyramid 2013-04-04
From Casey:
Hello
Right now I am stuck and I feel embarrassed because I feel like the answer is so easy I should know it.

I am working on a project and need to find a volume of gravel it will take to occupy this triangular prism like area. I am not sure what formulas I should use whether it be that for the volume of a pyramid or something more complex? Basically it forms a right triangle at one side then from there all points slope to one singular point about 10412mm away.
I am attaching a picture drawn up in paint with the actual dimensions to clear up any confusion.

Thank you for any help. Casey

Answered by Penny Nom.
Question 2013-04-04
From Casey:
Hello
Right now I am stuck and I feel embarrassed because I feel like the answer is so easy I should know it.

I am working on a project and need to find a volume of gravel it will take to occupy this triangular prism like area. I am not sure what formulas I should use whether it be that for the volume of a pyramid or something more complex? Basically it forms a right triangle at one side then from there all points slope to one singular point about 10412mm away.
I am attaching a picture drawn up in paint with the actual dimensions to clear up any confusion.

Thank you for any help. Casey

Answered by Penny Nom.
We can't write sinx and cosx as a finite polynomial. 2013-03-31
From rimoshika:
prove that we can't write sinx and cosx as a finite polynomial.
Answered by Walter Whiteley.
A markup followed by a discount 2013-03-23
From edward:
A shopkeeper marks his goods to gain 35%. He allows 10% discount for cash payment. What is his profit percent when he sells the goods for cash?
Answered by Penny Nom.
Percent profit: State exam question 2013-03-12
From tony:
which is the correct way to figure 30% profit?
1)
1000 product
500 labor
50 permit
1550 total cost
1550x1.3=2015

2)
1000 product
500 labor
50 permit
1550 total cost
1-.3=.7
1550/.7=2214

Answered by Harley Weston.
state exam question 2013-03-12
From tony:
which is the correct way to figure 30% profit?
1)
1000 product
500 labor
50 permit
1550 total cost
1550x1.3=2015

2)
1000 product
500 labor
50 permit
1550 total cost
1-.3=.7
1550/.7=2214

Answered by Harley Weston.
A messy arithmetic expression 2013-02-24
From niiaryee:
(((((5^2*4^3)1/5+9^3/3)+(25+(4*5)/15)^2-10)/10/2)3)))))
Answered by Penny Nom.
The tangent to a circle at a point on the circle 2013-02-22
From Andrew:
What is the equation of the line tangent to the circle with equation x^2+y^2=25 at the point (-4,3)
Answered by Penny Nom.
Your profit should be 20% of your cost 2013-02-21
From anis:
if the customer says your profit should be 20% of your cost..

1) my selling value should be cost +20% of cost = SP

or

2)cost / 0.80 = SP which is correct...if the second answer is right how should i justify my customer

Answered by Penny Nom.
Related rates 2013-02-17
From Ishaak:
A hemispherical bowl is filled with water at a uniform rate. When the height of water is h cm the volume is π(rh^2-1/3 h^3 )cm^3, where r s the radius. Find the rate at which the water level is rising when it is half way to the top, given that r = 6 and the bowl fills in 1 minute.
Answered by Penny Nom.
The continuity of f(x,y)=ln(x^2+y^2) 2013-02-17
From anu:
the question says we have to find the points in the plane where the function is continuous: f(x,y)=ln(x^2+y^2) . here we aren't given a particular point (x,y) where we have to check a function's continuity. what is to be done if we have to check continuity over the whole domain of the function? please help .
Answered by Harley Weston.
The equation of a circle 2013-02-11
From mhd:
Complete the equation of the circle centered at(0,4) with radius 3
Answered by Penny Nom.
4*4+4*4+4-4*4=??? 2013-02-06
From Jacky:
4*4+4*4+4-4*4=???
Answered by Penny Nom.
What is the domain of f(x)=sin(ln(x))/ln(x)? 2013-02-06
From Behrooz:
Hi, the following problem may be interesting: What is the domain of f(x)=sin(ln(x))/ln(x)? Be careful, domain is not obvious. Best regards Behrooz
Answered by Penny Nom.
The fourth side of an irregular polygon 2013-02-01
From Emran:
I have a irregular polygon. I know 3 of the 4 sides, and 2 of the angles. A-B is 285, B-C is 149, and C-D is 310. Angle B is 135 degrees. and Angle C is 45 degrees. Is there a formula to solve for the final side? Thanks.
Answered by Penny Nom.
We have 2 teams of 6 couples each that play 6 games 2013-01-25
From Denise:
We put together a game night where we have 2 teams of 6 couples each that play 6 games. We haven't been able to figure out an arrangement that allows each couple to play each game with a different couple from the opposite team (i.e. Team A couples play every game with a different couple from Team B). Is this possible? It works with 2 teams of 5 couples each.
Answered by Chris Fisher.
Maximize profit 2013-01-19
From Chris:
A firm has the following total revenue and total cost function.
TR=100x-2x^2
TC=1/3x^3-5x^2+30x
Where x=output
Find the output level to minimize profit and the level of profit achieved at this output.

Answered by Penny Nom.
The weight of a pipe full of water 2013-01-16
From joe:
how do you figure out the weight of a pipe full of water
Answered by Harley Weston.
A shed roof 2013-01-12
From christine:
A roof on a shed is 7.3 ft wide has an incline of 20 degrees what is the height?
Answered by Penny Nom.
3x6)+(12divided by2)-8= ? 2012-12-17
From christina:
hi my question that's been bothering me is what's
(3x6)+(12divided by2)-8= ?????

Answered by Penny Nom.
If n is odd, then n^2 - 3 is even 2012-12-11
From Tracy:
Prove the statement:

For all integers n, if n is odd, then n2 - 3 is even.

Answered by Penny Nom.
The angles of elevation and depression 2012-12-03
From Chelsey:
a person on a balcony of one building looks towards a second building. if the angle of elevation to the top of the second building is 25 degrees, the angle of depression to the bottom of the second building is 17 degrees, and the balcony of the first building is 22 feet above the ground, what is the height of the second building?
Answered by Penny Nom.
Two quadratic equations 2012-11-22
From fahmie:
graph and solve the intersection of the following equations:
x^2+y^2=4
x^2+2y=4

Answered by Penny Nom.
Difference of squares 2012-11-19
From Qelibar:
Please factorise x^2y^2 - 4
Answered by Penny Nom.
The square of any odd number, decreased by 1, is divisible by 8 2012-11-16
From bailey:
Prove that the square of any odd number, decreased by 1, is divisible by 8
Answered by Penny Nom.
How much should I price the earrings? 2012-11-13
From Sharon:
I want to sell some earrings and make a $12.00 profit on them I have to pay a 35% commission on them. How much should I price the earrings?
Answered by Penny Nom.
4+4x5+4= ? 2012-10-31
From Bob:
4+4x5+4= ?
Answered by Harley Weston.
How fast is the distance between the aircraft and the car increasing? 2012-10-24
From Steven:
At a certain instant an aircraft flying due east at 240 miles per hour passes directly over a car traveling due southeast at 60 miles per hour on a straight, level road. If the aircraft is flying at an altitude of .5mile, how fast is the distance between the aircraft and the car increasing 36 seconds after the aircraft passes directly over the car?
Answered by Penny Nom.
Composition of functions and one to one 2012-10-17
From Ariana:
If f o g are one to one function,does it follow that g is one to one? Give reasons for your answers
Answered by Penny Nom.
The degree measure of the central arc of a circle 2012-10-17
From Crystal:
On a circle with radius of 12 cm is an arc of length 20 cm. What is the degree measure of the central angle used to make this arc?
Answered by Penny Nom.
Profit 2012-10-05
From kathie:
find the profit income is $500 expenses $120
Answered by Penny Nom.
Profit and discount 2012-09-24
From ally:
A wholesaler purchased a wheat for $150 per tonne.He sold it at $170 per tonne.Calculate the percentage profit to (1 decimal place)if the wholesaler offered a 12% discount on the selling price.What could be the new selling price?
Answered by Penny Nom.
A profit of 35% 2012-09-07
From Jack:
If a product cost me $10.00 and I want a profit of 35% what is my sell price? How do I figure the sale price to make a 35% profit on the sale?
Answered by Penny Nom.
The volume of a sphere 2012-08-18
From Rohit:
why isn't volume of a sphere = Area of a semicircle x the circumference. i.e. if we revolve a semicircle around its axis we get a sphere
Answered by Penny Nom.
5 + 5 + 5 - 5 + 5 + 5 - 5 + 5 x 0 = 2012-07-29
From Tom:
5 + 5 + 5 - 5 + 5 + 5 - 5 + 5 x 0 =
Answered by Harley Weston.
John's electronic store 2012-07-25
From Jora:
Electronic Store John opened an electronic store in December. During his first month, He sold 10 LCD TVs and 20 Plasma TVs. His income during that month was $12800. In January, he sold 25 LCD TVs and 40 Plasma TVs for an income of $27600. In February he projects to sell 30 LCD TVs and 50 Plasma TVs. If John expenses are $28900 in February, how much money will he have after paying the expenses?
Answered by Penny Nom.
A volume of revolution 2012-07-15
From Tewodros:
Let f(x) = e^x and g(x) = x^1/2 both be defined on [0,1]. Consider the region bounded by f(x), g(x), x = 0, x = 1. Rotate this region about the y-axis and determine the volume using the shell method.
Answered by Harley Weston.
The angular elevation of the sun 2012-07-14
From VINEET:
WHAT IS ANGULAR ELEVATION OF THE SUM
Answered by Penny Nom.
A cylinder is to be filled with peas. 2012-06-12
From Silje:
Hi! How can I solve the following question without the use of a calculator?

"A cylinder is to be filled with peas. It is done like this: At 12:00 o'clock you put 1 pea in, at 12:01 you put 2 peas in, at 12:02 you put 4 peas in, at 12:03 you put 8 peas in, and so on. This continues until 14:00 o'clock (two hours later), when the last peas are put in and the cylinder is full. At what time is the cylinder half full?"

Answered by Robert Dawson.
9-3(2+6)/6-2*5 2012-06-08
From Sammi:
Hi there,

I am doing a practice test for my admittance into a college accounting program and I am really confused by this equation.

The answer I got was way off what the test answer sheet says it should be.

The question is

9-3(2+6)/6-2*5

If you could explain how the answer beccomes 35 that would be greatly apprciated!!

Thank you!

Answered by Robert Dawson.
Multiples 2012-05-28
From Kenneth:
If I understand correctly , a multiple is a product of two numbers. For example some of the multiples of 6 are 6, 12, 18, 24, 30, etc. I just multiplied 6 by 1, 2, 3, 4, 5, etc.

Are the multiples of a fraction, for example, 2/3, determined in the same way? Are they 2/3, 4/3, 6/3, 8/3, 10/3, etc., or are they instead, 2/3, 4/6, 6/9, 8/12, 10/15, etc.?

Or do fractions have no multiples?

Answered by Penny Nom.
Sharing the profit 2012-05-14
From Mafiza:
A and B started a business by investing $6000 and $8000 respectively. At the end of the year, a profit of $2100 is made. How much amount will A get as his share of profit?
Answered by Penny Nom.
An equation of the form y=mx 2012-04-17
From Samiya:
what would be an equation of the form y=mx with the points (-2, 6) and (1, -3) for the line?
Answered by Penny Nom.
Profit as a percent 2012-04-14
From Anson:
The cost price of a CD is $50 and its marked price is $105. If the selling price is $88, find the profit percent.
Answered by Penny Nom.
The spread of a rumor 2012-04-09
From Roohi:
The function f(t) = a/(1+3e^(-bt)) has also been used to model the spread of a rumor. Suppose that a= 70 and b=3 0.2. Compute f(2), the percentage of the population that has heard the rumor after 2 hours. Compute f'(2) and describe what it represents. Compute lim t approaches infinity and describe what it represents.
Answered by Penny Nom.
The period T of a pendulum 2012-03-27
From Ashley:
The period T of a pendulum is given in terms of its length, l, by T=2pi sqrt(l/g) where g is the acceleration due to gravity(a constant)
a. find dT/dl
b. what is the sign of dT/dl
c. what does the sign of dT/dl tell you about the period of the pendulums?

Answered by Penny Nom.
The equation of a line 2012-03-26
From Danielle:
Hello, could you help me figure out what the equation of the line is in the form of Ax+By=C with the points (-2,0) and (0,3)? Thank You!
Answered by Penny Nom.
A vertical line passing through (10,4) 2012-03-16
From Isabelle:
Hi! My question is this:

Write the equation of each line described:
A vertical line passing through (10,4)

I would really appreciate your help!
Thanks,
Isabelle

Answered by Penny Nom.
An increase/decrease in profit 2012-03-13
From Thanikasalam:
It is understood that increase/decrease in profit for current year compared to the previous year is done by the formula (x-y)/y, with x=profit on current year and y=profit on previous year. How do we derive this formula?

If we work on this formula of calculating increase/decrease in profit, we get x/y-1

* = (x-y)/y
= x/y - y/y
= x/y - 1

How do you justify the 1 in the equation above. How to derive the formula and how do i prove it?

Answered by Penny Nom.
A storage box with a slanted roof 2012-02-13
From Sophia:
Hi!
I have another problem.

The diagram shows a side view of a box which is used to store small logs of wood for burning in a fire place. The slopping lid has an overhand of 15cm.
a) Calculate the total length of the slopping lid to the nearest cm.
b) When the lid is open above ground will the end of the lid be?

P.S. Please see attached.

Answered by Penny Nom.
The curvature of the earth 2012-02-08
From sean:
Question from sean, a student:

Two people 1.8 metres tall walk directly away from each other until they can no longer see each other (due to the curvature of the earth, which has a radius of about 6378 km).
A) Find a function relating the height of two identical objects with the distance between them using the scenario above as an example.
B) Sketch this function (you may use Graphmatica if you wish). Over what domain and range does the function exist?
C) Describe this relation in practical terms.

Answered by Harley Weston.
Two sectors of a circle 2012-02-03
From Wayne:
I am not even sure what to ask. I have to two lengths of feet and an angle of degrees, and I have to find the area?
**Problem attached

Thanks,
Wayne

Answered by Penny Nom.
1 + 3 + 3^2 ...+3^(n-1) = 3^n - 1/2 2012-01-27
From Vicki:
I am trying to find out how to do show how this proof was worked.
Here is the end result 1 + 3 + 3^2 ...+3^(n-1) = 3^n - 1/2

This equation was used to find the number of white triangles in the Sierpinski Triangle

Answered by Walter Whiteley.
Four apples and two oranges cost Rs. 30... 2012-01-13
From nasr:
Four apples and two oranges cost Rs. 30, and one apple and 3 oranges costs Rs.15.How much does each apple and each oranges cost?
Answered by Harley Weston.
A volume of revolution 2012-01-11
From john:
find volume of solid generated by revolving the region in the first quadrant bounded by the curve y squared=x cubed, the line x=4 and the x-axis about the line y=8. The answer in the back of the book is 704 pi divided by5
Answered by Penny Nom.
Sum and difference of squares 2011-12-31
From Anne:
Se x e y são números reais distintos, então:
a) (x^2 + y^2) / (x - y) = x + y
b) (x^2 + y^2) / (x - y) = x - y
c) (x^2 - y^2) / (x - y) = x + y
d) (x^2 - y^2) / (x - y) = x - y
e) Nenhuma das alternativas anteriores é verdadeira.

Answered by Harley Weston.
Three sides of a triangle 2011-12-24
From saba:
the three sides of a triangular lot have lengths 10,11and 13cm,respectively. find the measure of its largest angle and the area of the lot?
Answered by Penny Nom.
The volume of a cylinder 2011-12-22
From luke:
what is the volume of a cylinder that is 30 mm in diameter and 70 cm high?

can you please write the equation out in full for me to help understand it better. thanks

Answered by Penny Nom.
The number of digits in a number base m 2011-12-22
From Jash:
Assume there is a number system of base m.

The one property of this system is: If 2 numbers written in this system, which have 'a' and 'b' as the number of digits are multiplied, then the product of the 2 numbers will have a number of digits which is a function f(a,b).

In other words, as long as the number of digits of the 2 numbers are constant, the number of digits of their product is a constant.

Find m.

Answered by Robert Dawson.
Water is flowing into a cup 2011-12-19
From Tim:
A cup has a radius of 2" at the bottom and 6" on the top. It is 10" high. 4 Minutes ago, water started pouring at 10 cubic " per minute. How fast was the water level rising 4 minutes ago? How fast is the water level rising now? What will the rate be when the glass is full?
Answered by Penny Nom.
A cube of ice is melting 2011-12-05
From Emily:
a cube of ice (i.e.) each side is of the same length) is melting at a rate such that the length of each side is decreasing at a rate of 5cm per hour. how fast is the volume of the cube decreasing (in cubic cm per hour) at the instant the length of each side is 25cm?
Answered by Penny Nom.
Water pouring into a conical tank 2011-11-21
From Patience:
Hi my name is patience and I'm having a problem with this question.
Water pours into a conical tank of semi vertical angle 30 degrees at the rate of 4 cm^3/s, where h is the depth of the water at time t. At what rate is the water rising in the tank when h = 10 cm?
Thank you

Answered by Penny Nom.
The slope of a line 2011-11-10
From Sara:
Line k lies in the xy-plane.
The x-intercept of line k is -4.
And line k passes through the midpoint of the line segment whose endpoints are (2,9) and (2,0).
What is the slope of line k?
Give the answer as a fraction.

Answered by Penny Nom.
What was the cost price of the article? 2011-11-10
From tashana:
By selling an article for $320,the profit was 20% of the cost price.what was the cost price of the article.
Answered by Penny Nom.
The area of a circle 2011-10-17
From Winney:
If the diameter of a semicircle is 3 feet what is the area.
Answered by Penny Nom.
One central circle and three tangent circles 2011-10-16
From Margaret:
You have one central circle and three or more circles tangent to the outside of the circle of varying radii. You know the x,y coordinates of the centers of the other circles. If you now remove that central circle (and pretend you never knew where it was), can you calculate its center in x,y coordinates?
Answered by Chris Fisher.
A reservoir has the shape of an inverted cone 2011-10-03
From Roger:
a reservoir has the shape of an inverted cone whose cross section is an equilateral triangle. if water is being pumped out of the reservoir at a rate of 2m^3/sec, at what rate is the depth of the water changing when the depth is 40 meters?
Answered by Penny Nom.
A hemispherical bowl with a lead ball inside 2011-09-27
From Jean:
"(a) Water is being poured into a hemispherical bowl of radius 3 inch at the rate of 1 inch^3/s. How fast is the water level rising when the water is 1 inch deep ?

(b) In (a), suppose that the bowl contains a lead ball 2 inch in diameter, and find how fast the water level is rising when the ball is half submerged."

Answered by Penny Nom.
Composition of functions 2011-09-05
From Jenna:
Let f(x)=x^2-1 and g(x)=1-2x. Find the indicated values.
1. f(g(1)) and g(f(1))

Thanks,
Jenna

Answered by Penny Nom.
The height of a fluid in a horizontal tank 2011-07-24
From jason:
Same set up as many others, cylindrical tank on its side, but I am interested in defining the change in volume and/or fluid level as a function of time at a constant volumetric outflow. I plan on hooking a pump to the tank so "gpms' will be constant. I have a couple different sized tanks and pumps so I want a general equation. Thanks for your help.
Answered by Harley Weston.
A reel of paper 2011-06-03
From keith:
if a full reel of paper has 7750 metres on it with a diameter of 1240mm and a core diameter of 100mm how can work out the thickness of each strip of paper therefore allowing me to work out the metreage of the roll when its half in diameter at say 620mm
Answered by Penny Nom.
A rational number 2011-05-17
From owais:
according to definition of irrational number "a number which cannot be expressed in p/q form is called irrational number" square root of 2 is a irrational number but if i round off the digit correct to two decimal places so it becomes 1.41 and we can easily convert into p/q form so it is a rational number ???
Answered by Penny Nom.
Dividing the profits 2011-04-19
From Tracy:
I need to know how to come up with the answers to the following problems. I need to be able to show my work. So can you help me on how to come up with the answers to questions like these. Thanks.

Three individuals form a partnership and agree to divide the profits equally. X invests $9,000, Y invests $7,000, Z invests $4,000. If the profits are $4,800, how much less does x receive compared to having the profits divided in proportion to the amounts invested by X, Y, and Z?

In printing an article of 48,000 words, a printer decides to use two sizes of type. Using the larger type, a printed page contains 1,800 words. Using a smaller type, a page contains 2,400 words. The article is allotted 21 full pages in a magazine. How many pages must be in smaller type? ___________

thank you

Answered by Penny Nom.
The equation of a sphere 2011-04-16
From kanika:
equation of sphere and how to know its center
Answered by Penny Nom.
Coefficient of variation 2011-04-14
From Braden:
When determining coefficient of variation (CV) or %CV is it possible to calculate %CV for two variables? For instance can %CV be used to determine the precision of 5 data points on a graph using the X and Y coordinates? or does %CV need to be determined for each variable separately?
Answered by Robert Dawson.
PEDMAS 2011-04-10
From Ross:
48 ÷ 2 (9+3)

Is the answer 2 or 288?

Answered by Harley Weston.
Eliminate y 2011-04-07
From Lynn:
2x + y = 8
y + 3z =5
z + 2w =1
5w + 3x = 9

Form three equations with y eliminated

Answered by Penny Nom.
A conical container and a spherical balloon 2011-04-06
From Steven:
Water is running out of a conical container 12 feet in diameter and 8 feet deep (vertex down) and filling a spherical balloon. At the instant the depth of the water in the cone is 4 feet, the radius of the sphere is approximately 4 feet.

The rate of change of the depth of the water in the cone at the instant is approximately ______________ times the rate of change of the radius of the balloon.

Answered by Penny Nom.
Administration costs and profit 2011-04-05
From brian:
Hi , If I have direct job costs of $100. and my administration is 20 % and I want to make a 15 % profit , how would I calculate the administration and profit and what would be the total of each be and also the final total?

Thanks,
Brian

Answered by Penny Nom.
Two ships 2011-04-05
From Gevork:
Ship A is sailing due south at 16 mph. At the same time, a second ship B, 32 miles south of A, is sailing due east at 12 mph.
(a) at what rate are they approaching or separating at the end of one hour?
(b) At what rate are they approaching or separating at the end of two hours?
(c) When do they cease to approach each other and how far apart are they at that instant.

Answered by Penny Nom.
The rate of change of the area of a parallelogram 2011-04-05
From Gevork:
Let a parallelogram have sides of 8 and 12 and let vertex angle A be decreasing at a rate of 2degrees per minute. Find the rate of change of the area of the parallelogram when angle A equals 30 degrees.
Answered by Penny Nom.
I need to make $1000 profit 2011-03-28
From Kat:
I need to make $1000 profit selling plants for $3 each. My direct cost is $2 per plant and my overhead is $5oo. How many pots (plants = 1 per pot) do I need to make $1000?
Answered by Penny Nom.
What is x to the power of 0? 2011-03-23
From Jason:
What is x to the power of 0?
Answered by Robert Dawson.
(3x+4y)^2 - (2x-y)^2 2011-03-16
From Taiwo:
pls could some one help me with this question? thanks as lot

factorize:
(3x+4y)^2 - (2x-y)^2

Answered by Penny Nom.
A family of circles 2011-03-01
From steffi:
Find the equation of the family of the circle passing through the the point of intersection of x^2+ y^2 -4x-28=0 and x^2 +y^2 -4x-20+52=0; the member tangent to x=7.
Answered by Penny Nom.
A camera's line of sight 2011-02-26
From MJ:
A rocket that is rising vertically is being tracked by a ground level camera located 3 mi from the point of blast off when the rocket is 2 mi high its speed is 400mph At what rate is the (acute) angle between the horizontal and the camera's line of sight changing
Answered by Penny Nom.
Prove sin x = sin (pi - x) 2011-02-15
From Janet:
Prove sin x = sin (pi - x)
Answered by Penny Nom.
The equation of a sphere 2011-02-09
From kelly:
what is the equation of a sphere if one of it's diameters has a giving end point of (4,4,4), (8,8,8)?
Answered by Penny Nom.
(7cubed * 8(exponent6))exponent 6 2011-01-29
From Shalaine:
What is the exponential form of (7cubed•8(exponent6))exponent 6?
Answered by Penny Nom.
Profit as a percentage 2011-01-17
From Connie:
I am trying to determine the profit margin on selected items. ex: A bottle of juice costs me $.70, I sell it at $3.75. How can I determine what my profit percentage is?
Answered by Penny Nom.
Identify each polynomial by its degree and number of terms 2011-01-10
From betty:
Write in standard form and identify each polynomial by its degree and number of terms. How do you do this?
Answered by Penny Nom.
If ac = bc ... 2011-01-04
From jamielle:
if ac=bc, then a is not equal to b, c is not equal to zero
Answered by Penny Nom.
Profit as a percent 2010-12-19
From gaurav:
In the following, which calculation is right to calculate the profit percent.
if profit is 20%- 100*1.2 = 120
if profit is 20%- 100/0.8 = 125?

Answered by Penny Nom.
Angle of elevation 2010-12-10
From PANKAJ:
angle of elevation of the sun perpendicular72 and base 88 find angle
Answered by Penny Nom.
The rate of change of (8e^3x)+(27 e^-3x) 2010-11-23
From Aleo:
I am unable to solve this problem: Find the rate of change of (8e^3x)+(27 e^-3x), with respect to x when x= 0.5
Answered by Penny Nom.
A 35% profit 2010-11-19
From Ana:
You have a stamp collection and make a 35% profit by selling it on E-Bay for $2700. Find the original cost of the collection by using the formula:New value × 100 / 100 + percent change = Original value
Answered by Stephen La Rocque.
How far must the pitcher travel to get to the ball? 2010-11-04
From ken:
A baseball player bunts a ball down the first base line. It rolls 35ft at an angle of 26 degrees with the first base path. The pitchers mound is 60.5 ft from the plate. How far must he travel to get to the ball.
Answered by Penny Nom.
What is the speed of each car? 2010-11-04
From Jennifer:
two cars are 420 miles apart and traveling towards each other along the same road. They meet in 3.5 hours. One car is traveling 15mph slower than the other. What is the speed of each car? This must be solved using a system of equation, I have no idea?
Answered by Stephen La Rocque.
I want to make a 50% profit 2010-10-29
From Debbie:
If my cost on an item is $32.00 and I want to make a 50% profit- What would the retail be?
Answered by Penny Nom.
Two algebraic expressions 2010-10-29
From chaptay:
what is the difference between x(n^2) and x(n)^2
Answered by Penny Nom.
The game of 24 2010-10-26
From gabrielle:
What is the answer using the numbers 0 0 0 0 (trig and calc are allowed, all math is allowed) in the game 24?
Answered by Claude Tardif.
Powers 2010-10-20
From dylan:
how do you write 20736 in exponential form .same for 1728 and 50625.

is there a formula to figure out how to express large know numbers in exponential form.

Answered by Penny Nom.
The equation of a circle 2010-10-20
From Silvan:
Hi, I just want to find the x,y values for the circumference of a circle...

Lets take a clock having its centre at (0,0) in a graph. I just want to know how to find the (x,y) co-ordinates for the curved path or the surface of the circle.. Is there any formula to directly align the curved path or the circumference of the circle in a graph for a known radius of a circle..

I feel it will be useful for me to draw a clock in a graph... :-)

Answered by Penny Nom.
The distance between the origin and a moving point 2010-09-24
From Norma:
I am having problems with this question find the rate of change of the distance between the origin and a moving point on the graph of the function below if dx/dt=5 cm/sec y=x^2+2
Answered by Penny Nom.
20% profit on the selling price 2010-09-24
From ata:
My cost is 100 and I want to make 20% profit on my selling price (not 20% of the cost). So, what should be calculation formula? Let's say my cost is 493.71 and I want to make 53.28% profit on the selling price; so how to determine the selling price?
Answered by Penny Nom.
Elimination and substitution 2010-09-18
From Lauren:
Solve one using the method of substitution and the other with the method of elimination.

v a. y=5x+4
x=2y+1

b. 4x+3y=7
6x-3y=13

Answered by Penny Nom.
A building and a flag pole 2010-09-09
From paul:
A flag pole and a building stand on the same horizontal level. From the point p at the bottom of the building,the angle of elevation of the top t of the flag pole is 65 degrees. From the top q of the building the angle of elevation of the point t is 25 degrees.If the building is20 meters high. Calculate the distance pt
Answered by Penny Nom.
Percentage improvement 2010-08-26
From shahrukh:
Hi...

We had a web page that was taking 12 secs to load. Now it loads in 3 secs. What is the percentage of improvement or how much faster is it working now. TO me it looks like it is working 3 times faster or there is a 300% improvement in the loading time. However my client says its ((12-3)/12*)100 = 75% improved. Which seems illogical on the face of it.

Thanks for your help
Shahrukh

Answered by Robert Dawson and Chris Fisher.
What is -2^2? 2010-08-25
From alex:
I know that (-2)^2 is 4 but what is -2^2?
Answered by Robert Dawson and Penny Nom.
How many labels are left on a roll? 2010-08-13
From Melissa:
Is there a simple way to calculate how many labels are left on a roll? I thought there was a way to look at the individual label length, the label thickness and the core diameter and then measure the overall diameter to calculate how many pieces were left, but I'm struggling. I've seen a tool before- I thought we called it a circle calculator, but those parameters were all that I needed to input to spit out the correct count. I never saw what was behind the scenes for this calculation. Ie factoring in revolutions or the number of pieces on each layer, etc. Please let me know if you have any suggestions. Any assistance will be greatly appreciated!!! Thanks!
Answered by Penny Nom and Tyler Wood.
A fruit seller 2010-07-31
From Nazrul:
5% of total number of fruits of a fruit seller has been rotten and also 5% of them has been damaged during transportation. What is the percentage profit by which he sells the rest so that he profit 20% as a whole?
Answered by Penny Nom.
Thickness of sand 2010-07-07
From RHONDA:
In an 18 x 33 ft area how much is 4 yards of sand in inches to cover this area.
Answered by Penny Nom.
Volume of a styrofoam cup 2010-06-23
From Stacy:
how do you find the volume of a truncated cup with height of 3, top diameter of 2.5, and a bottom diameter of 2
Answered by Penny Nom.
Selling price, cost and profit 2010-06-22
From Alex:
The profit on a certain television set is 16.5% of the cost price. If the profit is $330, Find:
a) the cost price
b)the selling price of the television

Answered by Penny Nom.
The number of gallons of water in a basement 2010-05-23
From George:
A basement with dimensions 68 ft long by 42 ft wide by 10 ft deep is flooded to the ceiling calculate the number of gallons of water in the basement. 70
Answered by Penny Nom.
Difference of cubes 2010-05-22
From Anad:
how can we prove a^3 - b^3 is equal to (a-b)(a^2+ab+b^2)?
Answered by Penny Nom.
Line of sight 2010-05-06
From David:
I live in St. Joseph, Michigan and there is an ongoing argument regarding line-of-sight over the horizon.

Standing on a 200 foot high bluff here, people swear they can see the top of the Willis (nee Sears) Tower in Chicago, which is about 1653 feet high.

It is my contention that this is actually a "refracted reflection" and not direct-line-of-sight.

So, to settle the argument, I'd sure like some simple explanation for this, even if-and I hope not-I am incorrect.

David

Answered by Harley Weston.
The rate of change of y with respect to x 2010-04-29
From Tom:
I just had a quick calc question about wording that wasn't ever addressed in class. When the book says "the rate of change of y with respect to x", should it be considered how fast y is changing in comparison to x?

I ask because the textbook says that "y is changing 3 times faster than x, so the rate of change of y with respect to x is 3." I'm use to rate being like velocity, as in units of distance per units of time. All we're told in class is that it's the slope of the tangent line, I was hoping you could clarify for me what exactly is meant by the wording of a "rate of change of something with respect to something else". More specifically, what "rate" and "with respect to" mean within this context?

Thanks for your time

Answered by Harley Weston.
The height of a flag shaft 2010-04-25
From Sarah:
A man standing 20metres away from a tower observes the angles of elevation to the top and bottom of a flag shaft standing on the tower as 62degrees and 60degrees respectively. Calculate the height of the flag shaft.'
Answered by Penny Nom.
Profit 2010-04-21
From joyann:
a Raleigh road runner 16" unisex bike which cost $892 was sold for $1052.56 calculate
A. the profit made on the bike
B. the percentage profit

Answered by Penny Nom.
Exactly two lines of symmetry 2010-04-11
From debbie:
i am looking for a quadrilateral with exactly two lines of symmetry. please help! thank you.
Answered by Tyler Wood.
The area of a field of view 2010-04-07
From Frank:
I know we have a field of view of 90 degrees and the distance we can cover is 6 miles. What is the area we can cover/view?
A formula would be nice to have since the diameter or distance of the wedge area can change and the degree of the field of view can also change.
Thank you and I look forward to hearing from you.
Frank

Answered by Penny Nom.
Fitting a sofa through a doorway 2010-04-06
From Yvonne:
Hi, I would be most grateful if you could help me, because I have no idea how to work this out! I am moving from a house to a flat, and the doors in the flat are 70 cm wide. I have a sofa 88 x 88 x 215. Will it go through the door if we tip it sideways at an angle?

Thanks, Yvonne Beck

Answered by Robert Dawson.
A system of equations 2010-03-30
From samantha:
the equations of two lines are 6x-y=4 and y=4x +2.what is the value of x in the solution for this system of equations?
Answered by Penny Nom.
The sides of a 30-60-90 triangle 2010-03-29
From maris:
Find the sides of a 30-60-90 triangle whose perimeter is 6. The solution must use systems of equations using 3 variables.
Answered by Penny Nom.
The area of a polygon 2010-03-28
From John:
Question from John, a student:

How do you find the area of a polygon with any given amount of sides?

Answered by Harley Weston.
The height of a hill 2010-03-26
From Amber:
A surveying team determines the height of a hill by placing a 12-foot pole at the top of the hill and measuring the angles of elevation to the bottom and to the top of the pole. They find the angels of elevation. Describe how to find the height of the hill.
Answered by Penny Nom.
A radio tower 2010-03-26
From Alex:
The height of a radio tower is 450 feet, and the ground on one side of the tower slopes upward at an angle of 10 degrees. How long should a guy wire be if it is to connect to the top of the tower and be secured at a point on the sloped side 110 feet from the base of the tower?
Answered by Harley Weston.
A roll of paper 2010-03-25
From keith:
how many yds of paper for a roll with a width of 6.875 and a core of 3 in and a diameter of 40in?
Answered by Robert Dawson.
The angle of intersection between two lines 2010-03-25
From Madhumitha:
How to find the angle of intersection between a line x=249 and another line which is of the form y=m*x+c; Line1 has a slope of infinity so what does the angle of intersection turn out to be? Or how can i find it?
Answered by Chris Fisher.
Modelling the roof of a house 2010-03-23
From Sandi:
This question has been haunting my dreams at night. It states the cross section of the roof of a house is modelled by the function y= -5\12|x-12|+5, where y>0 or y. Now I'm suppose to graph the function. b) Find the slope, height, length of sides, and base of the isosceles triangle. c)Explain what transformation must be applied to the graph of y=|x| to obtain the graph of y= -5\12|x-12|+5, where y>0 or y=0. 1st I put the equation into my calculator and graphed it. I got 10 for the height. We have had 2 sub teachers this past week. The first one told me the height should be 12 the other one told me 5. So I'm really confused. If I use my 10 units as height and the Pythagorean Theorem I get my sides to be 26 units the base 48 and my slope I calc to be 5/12. Both teachers are pretty sure I'm wrong and they are right but we all have different answers. Please help put this problem to rest so I can rest soundly at night. Thank you for your time.
Answered by Penny Nom.
A quadrilateral with 4 known sides and 1 known angle 2010-03-19
From samuel:
Name: Samuel
Status: Student

I have a quadrilateral with 4 known sides and 1 known angle, and I'm trying to evaluate the other angles of my quadrilateral.

By the law of cosines, I can easily find my opposite angle (using the diagonal as a basis for the equation).

However, to find the two remaining angles, I have found no other way so far than to use the other diagonal, which can be found with the equation attached (from geometry atlas).

Is there any simpler way?

Answered by Robert Dawson and Harley Weston.
Cooling 2010-03-07
From Lori:
If a house is always at 20 degrees celsius. Water (tea) boils at 100 degrees celsius. 5 minutes later the tea was 70 degrees celsius. Use an equation to predict the temperature after 20 minutes.
Answered by Tyler Wood.
Prove A intersect B =X iff A = X and B = X 2010-03-06
From Gloria:
how would you prove A intersect B =X iff A = X and B = X
Answered by Tyler Wood.
How many CDs and videos did the store sell? 2010-03-01
From dawn:
A used book store started selling CDs and videos. In the first week,the store sold 40 used CDs and videos,at 4.00 per CD and 6.00 per video.The sales for both CDs and videos totaled 180.00 she wrote a system of equations to represent the situation.Then she graph the system of equations

thanks- Dawn

Answered by Tyler Wood.
The test for some disease is 99% accurate 2010-02-24
From baaba:
Assume that the test for some disease is 99% accurate. If somebody tests positive for that disease, is there a 99% chance that they have the disease?
Answered by Chris Fisher.
15% profit 2010-02-21
From daviann:
calculate the cost price,given
selling price=$304
percentage profit=15%
what is the answer?

Answered by Penny Nom.
The angle at which the road is inclined 2010-02-10
From abeth:
a mountain road drops 5 m for every 22 m of road. Calculate the angle at which the road is inclined to the horizontal to the nearest degree.
Answered by Penny Nom.
Order of operations 2010-02-07
From addie:
(3+10) x 10 - 8 x 4 =
Answered by Penny Nom.
Everything in a cube 2 miles long? 2010-01-31
From Naresh:
in a book, i got to read this :

Is it possible to pack the entire population of earth and everything that was created by humankind in a cube whose edges are 2 miles long ?

Is it possible? Thanks.

Answered by Stephen La Rocque.
The height of a roof 2010-01-31
From carl:
Width of my roof I am building is 5M at baseline, and the pitch is 40%. What will the height be, and how can I work this out in the future.
Answered by Penny Nom.
1^3 + 2^3 + 3^3 +4^3 ... n^3 = ? 2010-01-29
From ireimaima:
Hi..
Can u please help me with this question.. I find that when i test eg: n=2 for n (n+1) /4, it seems that it does not giving me the right answer of 1^3 + 2^3 = 9 but 3/2... i'm confuse..can u please help me..thanks so much

Prove that: 1^3 + 2^3 + 3^3 +4^3………………………………..n^3 = n (n+1) /4

Answered by Penny Nom.
Solving a triangle 2010-01-25
From Paige:
how do i solve a triangle with one angle of 73 degrees, one angle of 32 degrees, and one side of 23cm?
Answered by Harley Weston.
Angle of incline 2010-01-20
From Alan:
how do I fnd the angle of an incline with a measurement of 0.042 with an adjacent of 1.2mtrs?. Thank You
Answered by Penny Nom.
A proof by induction 2010-01-12
From Bhavya:
Prove by induction that if Xi >= 0 for all i, then

(Summation Xi from 1 to n)^2 >= Summation Xi^2 from 1 to n

Answered by Penny Nom.
A proof involving real numbers 2010-01-11
From Amper:
Let a,b is an element of real numbers, and suppose that for every x>0 we have a is lesser than or equal to b+x.
(a) Show that a is lesser than or equal to b.
(b) Show that it does not follow that a is lesser than b.
i'm feeling bad of having no idea with this, hope i you can help me. GRACIAS!!

Answered by Penny Nom.
Profit increase from 10% to 15% 2010-01-03
From taha:
A shopkeeper sold a pen for $13.20 to a make a profit 10 percent.In order to earn a profit of 15 percent,he should have sold it for?
Answered by Penny Nom.
A trig question 2009-12-15
From A trig question:
Hey, my name is Candle
I'm in academic math10 and am stuck on my trig... one question I thought I had right because i used the cosine law I got wrong and can't figure out why... here's a copy of the question. (i guessed it was D... but my teacher said it's B)
Thanks
Candle

Answered by Robert Dawson.
A 4 digit number 2009-12-15
From Lisa:
Billy Club was assigned the task of putting numbers on all the playground balls used during daily recess. Billy will number the balls using the following rules:

1. It will be a 4-digit number.

2. The digit in the thousands place will be a 1 or a 2.

3. The digit in the hundreds place will be a 2, 4, or 6.

4. The digit in the tens place will be an odd number.

5. The digit in the ones place will be greater than six.

How many balls can Billy number if he follows these rules?

Answered by Robert Dawson.
A telephone pole on a slope 2009-12-14
From Marissa:
A 10 meter telephone pole casts a 17 meter shadow directly down a slope when the angle of elevation of the sun is 42 degrees. Find the angle of elevation of the ground. Its a law of sines problem.
Answered by Penny Nom.
e = m c^2 2009-12-09
From Jim:
Please explain the following formula: E=MC2 (squared)
Answered by Robert Dawson.
Game of 24 2009-12-09
From Tony:
Checked out all of the previous inquiries into the crazed game, and found none with these particular numbers: 1, 4, 6, 7. I've tried every method I can think of, please Help! Tks.
Answered by Robert Dawson, Penny Nom and Claude tardif.
How fast is the distance between the two cars decreasing? 2009-12-08
From Jenny:
Two cares are on a collision course toward point P. The paths of the two cars make a 30 degree angle with each other. The first car is 40 km from P, and traveling toward P at 16 km/hour. The second car is 50 km from P, traveling at 20 km/hour. How fast is the (straight line) distance between the two cars decreasing. (Hint: Law of Cosines)
Answered by Harley Weston.
(9 - x^2)/(x - 3) 2009-12-04
From Sandy:
9-x^2/x-3
I need to know how to solve this.
Thanks

Answered by Penny Nom.
(24x3)+(7x8)-(20/5)+(4x7) 2009-12-01
From Marilyn:
(24x3)+(7x8)-(20/5)+(4x7)
Thought I had the answer but the form says no. Could you solve it for me please.

Answered by Penny Nom.
Three angles and one side of a triangle 2009-11-16
From Esther:
How do i find the sides of an acute triangle if i know the angels are 60,45,75 and i only know one side which is 10? Thanks!
Answered by Penny Nom.
Maximize profit 2009-11-14
From Willie:
Profit is the difference between Total Revenue and Total Cost. Therefore, to MAXIMIZE PROFIT you must maximize Total Revenue. True or False? Explain answer.
Answered by Penny Nom.
At what rate are the people moving apart? 2009-11-01
From saira:
A man starts walking north at 4 ft/s from a point P. 5 minutes later a woman starts walking south at 5 ft/s from a point 500 ft due east of P. At what rate are the people moving apart 15 minute after the woman starts walking ?
Answered by Harley Weston.
A linear system 2009-10-20
From marissa:
Solve this linear system
2x-y=5
3x+y=-9

Answered by Penny Nom.
Proof that the root of 27 is irrational 2009-10-18
From Scarlet:
How do you prove that the square root of 27 is irrational?
Answered by Victoria West.
A right triangle 2009-10-12
From Josseph:
A right triangle has an area of 84ft sq. and a hypotenuse 25ft long. What are the lengths of the other two sides?
Answered by Penny Nom.
Game of 24 2009-10-02
From trami:
game of 24 by using -4 -7 -7 1?
Answered by Claude Tardif.
Prove by induction 2009-10-02
From Anonymous:
How can you prove the following by induction:

Any fraction (A / B), where 0 < (A / B) < 1, can be expressed as a finite sum
(1 / c(1)) + (1 / c(2)) + (1 / c(3)) + ... + (1 / c(k)),
where c(1), c(2), ..., c(k) are natural numbers greater than 0.

[ex. (20 / 99) = (1 / 9) + (1 / 11)]

Answered by Claude Tardif.
-4 squared and (-4) squared 2009-09-29
From Andrea:
Is -4 squared the same as (-4) squared? I am thinking the first is -16 and the second is +16. I am trying to clarify for my students.
Answered by Penny Nom.
Evaluating -x^2 + (yz - 3)^2 2009-09-28
From Kathy:
My daughter and I are having trouble solving this equation:

-x²+ (yz – 3)²

if x=-4,y=2 and z=0 We are having difficulty with the -x² part of the equation. Thanks

Answered by Penny Nom.
How many terms are there in this sequence? 2009-09-28
From tabby:
How many terms are there in this sequence?
5,1,-3,...,-111

Answered by Penny Nom.
Probability 2009-09-27
From Ed:
My mother died 3 years to the day after her daughter died. what are the odds of that happening by chance? thanks
Answered by Chris Fisher.
Order of operations 2009-09-24
From aman:
my question is regarding the fact i have a formula which looks like this :

(ratio 1 + ratio 2 divided by 2)

so my question is do i add first then divide by 2 or do it all together

Answered by Penny Nom.
5 x 8 + 6 divided 6 - 12 x 2 2009-09-24
From Susan:
5 x 8 + 6 divided 6 - 12 x 2. I am not sure of the rules of operation for this type of question
Answered by Penny Nom.
Two equations in two unknowns 2009-09-18
From Citizen:
x+-3y=7
-x+4y=7

Answered by Penny Nom.
The method of substitution 2009-09-17
From laura:
ok! i really need help with this question plz help!!

x= - 4y +5
x+2y = 7

Answered by Penny Nom.
A trillion grains of rice 2009-09-14
From akaila:
can 1trillion rice grains fit in a classroom with the area of 144.4m3
Answered by Penny Nom.
Simultaneous equations 2009-08-28
From onias:
solve 3/a - 2/b = 1/2 , 5/a + 3/b = 29/12
Answered by Robert Dawson.
y=2x+1 and y=2x-1 2009-08-28
From MARICELA:
Need help of how to work this problems

y=2x+1 and y=2x-1

First of all what is the difference?

Second I think is easy just to work with the X y with the line in the middle

but how do you get the numbers for each side.

Answered by Penny Nom.
The game of 24 2009-08-27
From ANITA:
Help I'm suppose to be a math expert, yet I can not yield 24 from the numbers 8,4,9,9, using each only once. HELP
Answered by Penny Nom.
An antiderivative problem 2009-08-13
From Indrajit:
∫4e^x + 6e^-x/(9e^x + 4e^-x)dx = Ax + Bloge(9e2x - 4) + C

then A=?......B=?.....C=?

plz solve it...."^" stands for "to the power of"....

Answered by Harley Weston.
The leaning tower of Pisa 2009-08-09
From MF:
Would you have any idea how the 'latitude of 44 degrees N" has anything to do with this question and how I would apply it?

The leaning tower of Pisa leans toward the south at an angle of 5.5 degrees. One day near noon its shadow was measured to be 84.02 m long and the angle of elevation from the tip of the shadow to the top of the tower was measured as 32.0 degrees. To answer the question, assume that the tower is like a pole stuck in the ground, it has negligible width. Also, it is important to know that Pisa Italy is at a latitude of approx 44 degrees North because this affects the direction of the shadow.)

Answered by Stephen La Rocque.
Eye Height Level Found from Visual Angle 2009-08-07
From Jolie:
Hello, I am trying to figure out what height from the ground a person's eyes are if they have a 12 degree visual angle to a screen that is 58cm away from their eyes? Thank you.
Answered by Janice Cotcher.
Highest Common Factor of Two Polynomials 2009-07-28
From Nazrul:
If x+a be the h.c.f. of x^2+px+q and x^2+mx+n, how can I prove that (p-m)a=q-n.
Answered by Robert J. Dawson & Janice Cotcher.
Inequalities Proof 2009-07-24
From ABOU:
good morning.......a b c are real positive no zero......proof that sq root(2a/(a+b))+sq root(2b/(b+c))+sq root(2c/(c+a))inferior or equal 3 thank you
Answered by Janice Cotcher.
Properties of Natural Numbers 2009-07-24
From nazrul:
If m,n,k are natural number how can I prove that (m+n)k=mk+nk. In the proof the properties of natural number should be used.
Answered by Janice Cotcher.
Proof of a Unique Solution 2009-07-24
From muele:
Find matrix A such that A is not invertible, and b such that Ax=b has a unique solution
Answered by Robert J. Dawson.
Selling fish 2009-07-22
From Hemantee:
A fishmonger bought 150kg of fish. he sold 70%of it at the price of Rs90 per kg and the rest at Rs 75 per kg. he made a profit of 42.5 per cent. how much did he pay for the fish.
Answered by Penny Nom.
Profit as a percentage 2009-07-15
From Jay:
If I buy an item for $3.00 and sell it for $6.00. What is my percentage of profit?
Answered by Penny Nom.
A roll of paper 2009-07-05
From mark:
is there a simple way of finding out how much is left on a roll of paper. i have read similiar questions and answers on here but all seem very complicated and not being very good at maths does not help me much. some are in inches and all have different figures to mine so if i give my figures hopefully i wil understand it better. the radius of the cardboard core is 52.25mm, the radius of the paper at 2000 linear meters is 158.625mm. the thickness is 0.17mm. when nearing the end of the paper how do i work out how much is left in a simple way. thank you in advance for any help you can give me
Answered by Harley Weston.
Coffee scoops per cup 2009-06-29
From charity:
if my coffee machine takes 10 scoops for every six cups, how many scoops do i need for 4 cups. each scoops equals one tbls(.25 oz)
Answered by Penny Nom.
Prove that the set of all positive odd integers is an infinite set 2009-06-20
From Nazrul:
How can I prove that the set of all positive odd integers is an infinite set.
Thank you in advance.

Answered by Victoria West.
A difference quotient 2009-06-17
From Sue:
When s(x)=x^3+x, compute and simplify the difference quotient s(x+h)-s(x)/h.
Answered by Harley Weston.
The product of gradients between 2 perpendiculars lines 2009-06-11
From Alister:
how do i prove that the product of gradients between 2 perpendiculars lines equal to -1....
Answered by Penny Nom.
Vectors and the Law of Cosine 2009-06-08
From lauren:
once force of 20 pounds and one force of 15 pounds act on a body at the same point so that the resultant force is 19 pounds. Find, to the nearest degree, the angle between the two original forces
Answered by Janice Cotcher.
Profit margin 2009-05-26
From andreya:
what is the profit margin of a business that spends $200 in order to make $1000
Answered by Penny Nom.
Game of 24 2009-05-22
From Sam:
Using the numbers 7, 3, 2, and 2, how do you get the number 24?
Answered by Claude Tardif and Harley Weston.
Maximum profit 2009-05-11
From Sally:
a manufacturer of dresses charges $90 per dress up to 100 units and the average production cost is $60 per dress. to encourage larger orders the company will drop the price per dress by .10 for orders in excess of 100. I need to find the largest order the company should allow with the special discount to realize maximum profit.
Answered by Harley Weston.
A 6 team social softball league 2009-05-02
From Don:
Hi We have a 6 team social softball league with only two fields. Each team plays two games each Saturday. We have three time slots 10:00, 12:00 and 2:00 We play for 11 Saturdays and than have a 2 weekend playoff. All teams prefer double headers vs the split at 10:00and 2:00. How do we schedule as balanced a schedule as possible and minmize the splits ? Thanks Don
Answered by Laura Morrison and Victoria West.
Game of 24 2009-05-02
From Barbara:
using the numbers 1,3 7 and 9 only once how do you get the answer of 24
Answered by Claude Tardif.
The sum of the roots of a quartic 2009-04-21
From dave:
This is a algebra problem that i am confused about: The sum of the roots of x^4-x^3+5x^2+4=0 is: i tried graphing it, but it shows that there are no roots, but the answer is 1. are they wrong?
Answered by Penny Nom.
12 oz. cup 2009-04-19
From Tom:
I am a ceramic teacher and wanted my students to make a 12 oz. cup, what formula should we use?
Answered by Chris Fisher.
Exponential form 2009-04-16
From Pete:
Hi, How do you express ³√h^-4 in exponential form. I am having a lot of trouble with this one.
thanks
Pete

Answered by Stephen La Rocque.
The axiom of choice and constructibe sets 2009-04-10
From sydney:
The axiom of choice asserts the existence of certain sets, but does not construct the set. What does "construct" mean here? For example, does it require showing the existence and uniqueness of some function yielding the set? In general, what does it mean to require the existence of a mathematical object be tied to a construction of it?
Answered by Claude Tardif.
Game of 24 2009-04-08
From Joe:
How do you get 24 using 6,9,10,10 once. Its called the 24 game.
Joe

Answered by Harley Weston.
Multiplication of polynomials 2009-04-07
From Carla:
I am struggling to understand Multiplication of Polynomials. No matter how hard I try to understand Multiplication of Polynomials, I just can't get it!

The problem that I am trying to solve is this :

-3x^3y(-y + 2 -x^2 + x)

Answered by Robert Dawson.
Game of 24 2009-04-05
From Ramona:
math 24 using 16, 9, 20, 7
Answered by Penny Nom.
Profit as a percent 2009-03-26
From Danny:
If I have something that cost me .38 and I want to sell it for $1.00 what is my profit % on this item
Answered by Robert Dawson.
The rate of change of the volume of a sphere 2009-03-25
From Kaylin:
why the rate of change of volume of a sphere is not constant even though dr/dt is constant?
Answered by Walter Whiteley.
The optimal retail price for a cake 2009-03-25
From Shawn:
Your neighbours operate a successful bake shop. One of their specialties is a cream covered cake. They buy them from a supplier for $6 a cake. Their store sells 200 a week for $10 each. They can raise the price, but for every 50cent increase, 7 less cakes are sold. The supplier is unhappy with the sales, so if less than 165 cakes are sold, the cost of the cakes increases to $7.50. What is the optimal retail price per cake, and what is the bakeshop's total weekly profit?
Answered by Robert Dawson.
The sides of a parallelogram 2009-03-17
From Sami:
If ABCD is a parallelogram, prove that line AB is congruent to line CD. Clearly state your reasons and conjectures.
Answered by Penny Nom.
The midpoints of two sides of a triangle 2009-03-17
From Manis:
Prove that the line joining the midpoint of two sides of a triangle is parallel to the third and half of it.
Answered by Robert Dawson.
Related rates 2009-03-14
From Jeevitha:
The side of an equilateral triangle decreases at the rate of 2 cm/s. At what rate is the area decreasing when the area is 100cm^2?
Answered by Stephen La Rocque.
Related rates 2009-03-09
From Megan:
A plane flying with a constant speed of 330 km/h passes over a ground radar station at an altitude of 3 km and climbs at an angle of 30°. At what rate is the distance from the plane to the radar station increasing a minute later?
Answered by Harley Weston.
The game of 24 2009-03-01
From Jeff:
My son in the 5th grade was given a problem for extra credit and could use parents help. He was given the numbers 21, 13, 9, 6 and asked to use each number only once by +, -, /, x the numbers in any order to obtain an answer of 24.
Answered by Harley Weston.
Collecting an army 2009-02-25
From bevaz:
Question from bevaz, a student: A ruler orders his chamberlain to collect an army from 30 houses. The servant goes to the first house alone and collects one man. At each house after that he takes the same number of men as he has already collected, so at the second house he goes with one other and so on. How many men did he collect in all?
Answered by Penny Nom.
Find the resultant of this displacement pair 2009-02-22
From katydidit:
Find the resultant of this displacement pair:
500 miles at 75 degrees east of north and 1500 miles at 20 degrees west of south.
How do I graph this and how do I solve this problem?

Answered by Penny Nom.
Angle of depression 2009-02-18
From Meeka:
An aircraft flying at an altitude of 2000m is approaching an airport. If the angle of depression of the airport is 5 degrees, what is the distance from the plane to the airport measured along the ground? Round your answer to the nearest tenth of a kilometer.
Answered by Robert Dawson.
A rectangular park 2009-02-06
From Debbie:
A park named Writer's Rectangle opened in town. When asked about the dimensions of the rectangle, the city planner, responded with these clues: ---The diagonals of the rectangular park plus its longer sides together measure seven times one of the shorter sides. ---The length of one diagonal is 250 m longer than one of the shorter sides. Use this information to find the area of the park.
Answered by Stephen La Rocque.
How much did the statue originally cost him? 2009-02-06
From Debra:
Jim is able to sell a hand-carved statue for $670 which was a 35% profit over his cost. How much ded he statue originally cost him?
Answered by Penny Nom.
The substitution method 2009-02-06
From amber:
Pleasse help me with this problem
x+y=-2
y-2x=1

Answered by Penny Nom.
Calculating markup 2009-02-03
From Tonya:
I'm trying to figure out the formula for the following, I have a product that I want to receive a certain profit, but with this product I also have to pay a percentage of commission to someone. I need to know how to calculate my new selling price taking into account the markup percentage, but I don't want my cut to be lowered. I have calculated the markup percentage to get my new selling price with the commission, but it lowers my cut once I pay the commission and I don't want it to do that. Thanks
Answered by Penny Nom.
How fast is the visible surface of the earth decreasing? 2009-01-24
From Ray:
A dive bomber loss altitude at a rate of 400 mph. How fast is the visible surface of the earth decreasing when the bomber is one mile high?
Answered by Harley Weston.
In the shadow of a flagpole 2009-01-22
From La:
How fast is the length of the shadow of an 18 foot flagpole growing when the angle of elevation of the sun is 45 degrees and is decreasing at a rate of 10 degrees per hour?
Answered by Harley Weston.
Factor x^2 - y^2 2009-01-20
From Shell:
complete Factor: x^2-y^2
Answered by Penny Nom.
Negative rate of change 2009-01-12
From hemanshu:
when i have to find rate of change of decrease in any value my ans comes in negative why??????????
Answered by Penny Nom.
A T-shirt fundraiser 2009-01-11
From Andre:
Lou's class is selling T-shirts for a fundraiser. The supplier charges 750$ for the initial design and the set-up plus 5$ for each imprinted shirt. The students sell the shirts for 15$ each.
a)How many T-shirts do the students need to sell to break even?
b)How much profit will the students make if they sell 150 shirts?

Answered by Harley Weston.
The table of values for y=3x-10 2009-01-09
From Jade:
How can I know the value of x for y=3x-10 in this table of values pls. include the deatails

if x=-2 y=-16
if x=3 y=-1
but how if y=2 what is x

Answered by Penny Nom.
30% profit 2009-01-06
From bob:
this is a basic question that I just can't get thru my head.

I know all my cost for a product, if I want to sell at a given profit can I divide by a given number, example for 30 % divide by .7 or do I multiply? Is there a simple answer for this? Sure would make my work a lot easier.

Thank you for your time

Puzzled in Ohio

Answered by Penny Nom.
The sum of the roots of a quartic 2009-01-03
From peter:
How do you find the roots of an equation without graphing? like, i have a problem that says what is the sum of the roots of x^4-x^3+5x^2+4=0.
Answered by Harley Weston.
System of equations - comparison method 2009-01-02
From nick:
Hi, I am doing a project on system of equations by using comparison method, but for some reason or another, i can't seem to find any information on the comparison method at all. So, please would you help me by giving me a website that you know of that has some information on my method. And, if you know the answer, can you please tell me why or when do we use system of equations by using comparison method? Thank You
Answered by Robert Dawson.
The area of a triangle 2008-12-23
From ashiya:
Hi,

I have attached figure of the triangle, Can you please help to find the area of Triangle ABC.

Thanks.

Ashiya

Answered by Stephen La Rocque.
Coefficient of variation 2008-12-17
From JR:
I have read your reponses regarding the coeffcient of variation (CV) and find them very useful. I still have a question about interpreting the CV. Let's that the CV of sample #1 is 3% and that of sample #2 is 12%. Can I report that Sample #2 is 4 times more variable than sample #1? Thanks in advance!
Answered by Robert Dawson.
The angle between two lines 2008-12-17
From abhi:
how to calculate the angle between two lines, given the length of the lines.. angle should vary from 0 - 360 in the counterclockwise direction
Answered by Robert Dawson and Harley Weston.
Do all square numbers have an odd number or factors? 2008-12-11
From Amy:
Do all square numbers have an odd number or factors?
Answered by Victoria West.
Related rates 2008-11-26
From Lyudmyla:
How fast is the volume of a cone increasing when the radius of its base is 2 cm and growing at a rate of 0.4 cm/s, and its height is 5 cm and growing at a rate of 0.1 cm/s?
Answered by Harley Weston.
The game of 24 2008-11-24
From Davey:
this has us all stumped. 5 ,5 ,7 ,and9 = 24. Oh may you assist in ending our brain pain
Answered by Penny Nom.
Profit as a percentage 2008-11-21
From Nomar:
Cost Price is $ 556,109.64 and Selling price is $ 1,020,936.00, What is the Profit Percentage?
Answered by Harley Weston.
The path of a small sailboat 2008-11-19
From jane:
a sailor in a small sailboat encounters shifting winds. she sails 2.00 km East then 3.40 km North East, then an additional distance in an unknown direction. Her final position is 6.68 km directly east of the starting point. find the magnitude & direction of the third leg of the voyage.
Answered by Harley Weston.
Factoring 2008-11-19
From Neji:
How do you factor (y-z) (y+z) (y^4+y^2z^2+z^4) and get (y+z)(y^2-yz+z^2) (y-z) (y^2+yz+z^2) as the answer?
Answered by Harley Weston.
How far are the boats apart? 2008-11-14
From dom:
Two boats leave port at the same time. They leave at 150 degree angle. One boat travels at 10mph and the other at 20mph. After two hours how far are the boats apart?
Answered by Penny Nom.
What is -5x+3y=24? 2008-11-13
From Robert:
What is -5x+3y=24???????
Answered by Stephen La Rocque.
The angles and sides of a triangle 2008-11-13
From JAMIE:
a triangle with a side(b)37m an angle(C)70degrees and (a)79m find values of angles A and B and length of side c
Answered by Stephen La Rocque.
How fast is the distance between the airplanes decreasing? 2008-11-10
From Crystal:
At a certain instant, airplane A is flying a level course at 500 mph. At the same time, airplane B is straight above airplane A and flying at the rate of 700 mph. On a course that intercepts A's course at a point C that is 4 miles from B and 2 miles from A. At the instant in question, how fast is the distance between the airplanes decreasing?
Answered by Harley Weston.
Melting ice on a hemisphere 2008-10-20
From heather:
The top of a silo is the shape of a hemishere of diameter 20 ft. if it is coated uniformly with a layer of ice, and if the thickness is decreasing at a rate of 1/4 in/hr, how fast is the volume of ice changing when the ice is 2 inches thick?
Answered by Penny Nom.
How much does Luc pay for his tools?? 2008-10-20
From imran:
Luc makes $1 profit on each tool he sells. Marc makes a $2 profit while paying $5 less for his tools than Luc. Marc thus makes $6 more profit for every $100 invested. How much does Luc pay for his tools??
Answered by Penny Nom.
An arithmetic series 2008-10-17
From Laura:
In an arithmetic series 5+9+13+...+tn has a sum of 945. How many terms does the series have? What formula do I use?
Answered by Penny Nom.
Related rates 2008-10-16
From Gisela:
As sand leaks out of a hole in a container, it forms a conical pile whose altitude is always the same as its radius. If the height of the pile is increasing at a rate of 6 in/min, find the rate at which the sand is leaking out when the altitude is 10in.
Answered by Penny Nom.
The rate of change of the volume of a cone 2008-10-15
From Barbara:
Suppose that both the radius r and height h of a circular cone change at a rate of 2 cm/s. How fast is the volume of the cone increasing when r = 10 and h = 20?
Answered by Harley Weston.
The average rate of change of gasoline used 2008-10-06
From JHulie:
What is the average rate of change of gasoline used, measured in miles per gallons if you travel 212 miles, then you fill your gas tank up again and it takes 10.8 gallons. If you designate your change in distance as 212 miles and your change in gallons as 10.8?
Answered by Penny Nom.
The vertex of a parabola 2008-09-30
From Anne:
How do you find the vertex of the function f(x)=-x squared+7x-6
Answered by Penny Nom.
Lines that pass through a point 2008-09-30
From Abigail:
what is the name for a point that a group of lines pass through that contains the letters c e n p i l?
Answered by Penny Nom.
z(z+1)-x(x+1) / z-x 2008-09-30
From sylvia:
z(z+1)-x(x+1) / z-x

HOW DO I SIMPLIFY THIS

Answered by Penny Nom.
a(a+1) - b(b+1) 2008-09-30
From Shaun:
I need to factor (a-b) out of the following: a(a+1) - b(b+1). I know it is simple but I cannot remember how.
Answered by Penny Nom.
How tall is the wall? 2008-09-29
From ash:
you and bob are separated by a tall wall you stand 10 feet further from the wall than bob your angle of elevation is 37 degrees and his 44 degrees how tall is the wall?
Answered by Penny Nom.
Number of factors 2008-09-18
From Austin:
I am stuck on finding what type of number has exactly two factors and what type of number has an odd number of factors? I need some examples.
Answered by Penny Nom.
The volume of a box 2008-09-16
From Andre:
volume of the box cubic centimeters

____ X _____ X ______ X =_______

The length is 8.5 Width is 3.4

Answered by Penny Nom.
The nth term of a sequence 2008-09-13
From lavett:
what is the Nth term in the sequence when the sequence is 2,4,8,16... and the term numbers are 1,2,3,4...
Answered by Stephen La Rocque.
Angle of elevation 2008-09-09
From kristy:
A man on the tenth floor of a building shouts down to a person on the street. If the angle of elevation from the street to the man in the building is 35° and the man in the building is 40 feet up, about how far away from the building is the person on the street?
Answered by Penny Nom.
The height of a tree 2008-09-09
From danice:
At a certain time of day, the angle of elevation of the sun is 30°. A tree has a shadow that is 25 feet long. Find the height of the tree to the nearest foot.
Answered by Penny Nom.
A security camera 2008-09-07
From Rita:
A security camera in a neighborhhod carnival is mounted on a wall 9 feet above the floor inside a video gallery. What angle of depression should be used if the camera is to be directed to a spot 6 feet above the floor and 12 feet from the wall?
Answered by Stephen La Rocque and Harley Weston.
Mathematical induction 2008-09-05
From James:
I need to prove a problem by induction regarding the Triangle Inequality. The problem is

abs(a1 + a2 +...+an) <= abs(a1) + abs(a2) +...+ abs(an).

Answered by Victoria West.
A line parallel to y= 1/2x + 5 2008-09-03
From Michelle:
Find the equation of a line parallel to y= 1/2x + 5 if the required line passes through ( 3,4)
Answered by Penny Nom.
A roof angle 2008-08-30
From carla:
Use vectors to find the angle in the attached diagram
Answered by Chris Fisher and Stephen La Rocque.
A cubic equation 2008-08-25
From RAM:
The following Cubic Eqn should have three roots - what are they?

x^3-27=0

Answered by Penny Nom.
Radii and Chords Create a Non-Right Triangle 2008-08-22
From Beary:
AOC is a diameter of circle O. Line AB is 12, and lines OA and OC (the radii) are 10. Find the length of line BO and chord BC.
Answered by Janice Cotcher.
Simplifying Algebraic Expressions 2008-08-22
From Jacky:
x^2-y^2+4x+4y
Answered by Penny Nom.
Trough Filling with Water 2008-08-21
From lanny:
a triangular trough is 10 feet long, 6 feet across the top, and 3 feet deep. if water flows at the rate of 12 cubic inches per minute, find how fast the surface is rising when the water is 6 inches deep.
Answered by Janice Cotcher.
The game of 24 2008-08-20
From Tasha:
I have four numbers a 6, a 6 a 5 and a 3, i can add subtract divide or multiply these numbers in anyway but i have to use all of them and call only use them once i have to come up with a total of 24
Answered by Penny Nom.
Factoring x^2 + 729 2008-08-19
From peter:
hello I,am having trouble factorising a polynomial into polynomial factors with real coefficients please can you help the polynomial is x^2+729
Answered by Harley Weston.
Solving for Shared Height of Two Right Triangles 2008-08-17
From Heidi:
find the height of a triangle, which can be split into two right triangles, but the base (50m) is not split equally in half. one end of the base is 40 degrees, while the other is 30 degrees.
Answered by Janice Cotcher.
BEDMAS 2008-08-12
From Rebecca:
I have 3 questions. 1) I don't really understand BEDMAS.Im going in to the 6th grade and im kind of nervis about it.? 2)When you're doing BEDMAS what does the small 3 or 2 above the other numerals mean? 3)How would you answer this:5+2 x 9 - 9 x 12= ??
Answered by Janice Cotcher.
Arc-length and sector-angle 2008-08-06
From Benson:
If chord length, radius are given, How to find the sector angle and arc-length
Answered by Janice Cotcher.
Find the product of 2^35 and 5^38 in sci. notation. 2008-08-03
From Peter:
I am preparing for a competition and a lot of the non-calculator problems are like find the product of 2^35 and 5^38 in sci. notation. How would you do that?
Answered by Penny Nom.
The product of the digits of a four digit number 2008-07-26
From Pete:
I am a student preparing for a competition and this was one of the prep problems: The product of the digits of a four digit number is 6x5x4x3x2x1. how many such numbers are there with this property?
Answered by Stephen La Rocque.
Proofs 2008-07-26
From Taylor:
when doing a proof, how do i figure out the steps in which i find the statements? i find the reasons pretty easily but i do not understand how to get the proving part. that would be great if you can help me! Thanks
Answered by Victoria West.
Alpha level 2008-07-23
From anonymous:
What is the advantage of using an alpha level of .01 versus a level of .05, What is the disadvantage of using a smaller alpha level?
Answered by Janice Cotcher.
The number of digits in a really big exponent 2008-07-21
From Pete:
how would you find the number of digits in a really big exponent without a calculator?
Answered by Penny Nom.
Four Positive Integers 2008-07-20
From william:
let a, b, c and n be positive integers. If a+b+c=(19)(97) and a+n=b-n=c/n, compute the value of a.
Answered by Janice Cotcher.
Number of factors 2008-07-20
From pete:
how do you find the number of factors in a really big number without using a factoring program or listing the out?
Answered by Penny Nom.
solving four simultaneous equations 2008-07-18
From Muhammad:
-2B-2C+4E=1 A+B+C+D=0 -2B-2C-2D+E=0 B+C+4D-2E=0
Answered by Janice Cotcher.
Does the sequence 1 2 4 8 16 32 etc have a name? 2008-07-17
From Richard:
Just an idle thought really. Does the simple sequence 1 2 4 8 16 32 etc have a name?
Answered by Victoria West.
Express as a function of x 2008-07-14
From Rita:
Express the area A of an isosceles right triangle as a function of x of one of the two equal sides.

NOTE: I always see the phrase "Express as a function of x" in math textbooks.
What exactly does that phrase mean?
I also have seen it written as "Express in terms of x."

Answered by Penny Nom.
Profit as a percentage 2008-07-08
From ANNORIS:
Question from annoris, a student:

How to calculate percent of profits using step by step division formula.
I know how to get the profit.
Selling cost was $15.00
Cost amount was $10.00
Total profits its $ 5.00.
But i do not know how to get the percent because i dont know how to divide to well . Can you helppppppp me ?

Answered by Penny Nom.
A net profit of 4% on a sales volume of $100 million 2008-07-07
From Muriel:
If a firm has a net profit of 4% on a sales volume of $100 million and the company has instituted logistics programs that result in cost savings of $1 million, what equivalent sales increase would be required to equal such a cost savings?

What I do know is that the net profit % has increased from 4% to 5% and the profit has gone from $4,000,000 to $5,000,000 but I have no idea what the formula is to figure out how much sales would have to increase to result in that same profit.

Answered by Janice Cotcher.
Sum and product of the roots of a quadratic 2008-07-03
From Gautam:
If a and b are the roots of x^2+px+1=0 and c and d are the roots of x^2+qx+1=0
prove that
(a-c)(b-c)(a+d)(b+d)=q^2-p^2
Regards
Gautam

Answered by Harley Weston.
The sum of the digits of two-digit number is 9 2008-07-01
From Audrey:
the sum of the digits of two-digit number is 9. if the digits are reversed, the new number if 63 greater than the original number. find the number.
Answered by Penny Nom.
Slope and rate of change 2008-06-23
From Lee:
What is the difference between a slope and a rate of change?
Answered by Stephen La Rocque.
111...111 * 2003 2008-06-23
From Peggy:
Number a = 111...111 formed by 2003 1's. Find the sum of the digits of the number 2003 * a.
Answered by Penny Nom.
The sum of the digits of a number 2008-06-23
From Ben:
Question: Using mathematical induction, prove that if the sum of the digits of a number is divisible by three, then the number itself is also divisible by 3.
Answered by Penny Nom.
30% profit 2008-06-19
From Julie:
how do i calculate the price per person that will return a 30% profit on the sales revenue generated?
Answered by Penny Nom.
Cows and chickens 2008-06-16
From damebochie:
I got this problem and want to write a system of equation for it:
Farmer Joe has cows and chickens on his farm. One day he count 76 legs and 24 heads. How many cows and how many chickens are on the farm? write a system of equations and solve.
I don't have any problem to solve it, but I just can;t see how I am going to write this system of equation. Thank you so much for helping?

Answered by Harley Weston.
The rate of change in the depth of the water 2008-06-12
From Liz:
A rectangular pool 50ft long and 30ft. wide has a depth of 8 ft. for the first 20 ft. for its length and a depth of 3 ft. on the last 20ft. of its length and tapers linearly for the 10 ft in the middle of its length. the pool is being filled with water at the rate of 3ftcubed/ min at what rate is the depth of the water in the pool increasing after 15 hours?
Answered by Harley Weston.
Guy wires for a tower 2008-05-19
From larissa:
a radio tower 500 feet high is located on the side of a hill ( the hill has an inclination to the horizontal of 5 degrees.) How long should two guy wires be if they are connected to the top of the tower and are secured at two points 100 feet directly above ( up the hill ) and directly below the base of the tower?
Answered by Penny Nom.
Selling price 2008-05-10
From Roxane:
I'm trying to figure out the formula for the following, I have a product that I want to receive a certain profit margin, but with this product I also have to pay a percentage of commission to someone. I need to know how to calculate my new selling price taking into account the markup percentage, but I don't want my gross margin to be lowered. I have calculated the markup percentage to get my new selling price with the commission, but it lowers my profit margin once I pay the commission and I don't want it to do that.
Answered by Penny Nom.
How many presses should be used? 2008-05-04
From Sarah:
Hi! I am in Calculus and this problem is on my study guide and i just cant figure it out!? A printing company had eight presses, each of which can print 300 copies per hour. It costs $5.00 to set up each press for a run and 12.5+6n dollars to run n presses for an hour. How many presses should be used to print 6000 copies most profitably? Let h equal the number of hours used to print the 6000 copies.
Answered by Harley Weston.
A bowl is the shape of a hemisphere 2008-04-28
From josh:
a bowl is the shape of a hemisphere with diameter 30 cm and water is poured into the bowl to a height h cm. how do i find the volume of the water in the bowl
Answered by Harley Weston.
A volume of revolution 2008-04-24
From Sabahat:
Hi, i have a region enclosed by both axes, the line x=2 and the curve y=1/8 x2 + 2 is rotated about the y-axis to form a solid . How can i find the volume of this solid?. (Please note that y equation is read as y =1 over 8 times x square plus 2.) I will be really grateful if you answer this question. :)
Answered by Harley Weston.
Why does -2^2 = -4 when -2 * -2 = 4? 2008-04-22
From blaine:
Why does -2^2 = -4 when -2 * -2 = 4?
Answered by Penny Nom.
The three sides of a triangle 2008-04-16
From Bridgett:
The sum of the lengths of any 2 sides of a triangle must be greater than the third side. If the triangle has one side that is 11 CM, and the second side of the triangle is 4 cm less than twice the third side, what lengths do the 2nd and 3rd side have to be?

between 0000-00-00 and 9999-99-99


Answered by Stephen La Rocque.
How many bags of mulch? 2008-04-07
From MJ:
I have an area that is 35' X 50' and I want to cover it with 2" of mulch.

Do I take length X Width X depth? Bags of mulch are sold in 2 or 3 cubic feet per bag.....I just want to know how to do the math. can you please send me the equation?

Answered by Penny Nom.
A volume of revolution 2008-04-04
From ted:
Consider the region bounded by y=x^2 + 1, y=5-3x and y=5. Sketch and shade the given region; then set up but dont evaluate teh integrals to find the following:

a) The volume of the solid generated by rotating the region about the line y=5

b) the volume of the solid generated by rotating the region about the y-axis

Answered by Penny Nom.
How many bags of cement do I need 2008-03-30
From Joe:
How many bags of cement do I need if I want to extend my carport 6 inches all around? The carport is 15ft. x 15ft. and the existing slab is 4 inches thick.
Answered by Harley Weston.
The average rate of change 2008-03-29
From Tom:
For the function x/3x-1 find the average rate of change between the interval x=1 and x=5?
Answered by Harley Weston.
A car tire full of concrete 2008-03-27
From robert:
I want to build a volleyball net support. I am using a car tire 24"odx16"id filled with concrete. how much will this weigh? thanks
Answered by Penny Nom.
The product of the roots 2008-03-19
From Rebecca:
Determine the product of the roots of the equation (x-1)(x-2) + (x-2)(x-5)=0
Answered by Penny Nom.
The sum and product of the roots of a quadratic 2008-03-16
From Katelyn:
Write the quadratic equation....

Sum of the roots = 16
Product of the roots = -80

Answered by Penny Nom.
A roll of paper 2008-03-05
From Sheik:
How I Convert My Roll Weight Into Meters?
Roll Width : 241mm
Thickness : 56gsm
Weight :81 Kg

Answered by Harley Weston.
The profit was what percent of the cost? 2008-02-29
From sierra:
The profit was what percent of the cost?
Here is some more of that whole problem.
Selling Price-$26.60
Discount-$6.65
Sale Price after the Discount-$19.95
The Profit-$5.95

Answered by Penny Nom.
A proof in geometry 2008-02-27
From Kimberly:
I'm trying to write a proof for the following: If all altitudes are equal in an equilateral triangle then all sides are equal.
Answered by Stephen La Rocque and Penny Nom.
What number has exactly 51 divisors? 2008-02-26
From Lisa:
What number has a Tau which equals 51? In other words, what number has exactly 51 divisors? It must be a square of some kind!
Answered by Penny Nom, Victoria West and Harley Weston.
10 - 3 + 2 2008-02-21
From Amrit:
what 10-3+2

is it 5 or 9

Answered by Penny Nom.
The slope intercept form of a line 2008-02-17
From chris:
find the slope and y intercept of the line (2x+4)-3y=11(x-1) is this equation in slope point form.
Answered by Penny Nom.
The length of the third side of a triangle 2008-02-16
From mary:
I have an angle of 72 degrees and each of the sides are 5' long. What is the distance from each of the ends of the 5 feet to form a triangle.
Answered by Stephen La Rocque.
5x^2 - 45 2008-02-11
From Tiana:
factor: 5x^2 - 45
Answered by Stephen La Rocque.
What percent profit is this? 2008-02-07
From Vanessa:
Mrs Henderson buys a chair for $215.00 and later sell the chair for $336.00 what percent profit is this?
Answered by Penny Nom.
Total profit 2008-02-03
From Rhonda:
I help to manage a concession stand. We sell a variety of items. Some have a profit of 60%, some 80%, etc. We have done nothing scientific to choose our prices. We just try to stay in the ballpark of schools around us. Our administrator wants to pay those that run the concession stand a percent of what is in the cash box after start-up money is taken out. We have some gathered some data (not much) on how many of each item we sell and can certainly calculate percent profit. Is the profit for the entire concession stand a weighed average? I'm thinking (% profit on candy)(number candy sold) + (percent profit nachos)(number nachos sold) + (percent profit pop)(number pop sold), etc. Will this work? Does the AMOUNT of profit for these different items need to be figured-in? Hmmm. . .
Answered by Victoria West and Harley Weston.
Sum and difference of cubes 2008-01-30
From Amanda:
It has been a really long time since I was in Algebra and I can't remember how to factor cubes such as x^3 +81 or subtracting/adding fractions with variables such as [1/(x+h)+2]-[1/x+2]. Please help!!!
Answered by Penny Nom.
The volume of a tank 2008-01-27
From Fred:
A cubic tank holds 1,000 kilograms of water what are the dimensions of the tank in meters?
Answered by Harley Weston.
Find the slope of the line with equation 7-3y=2x 2008-01-23
From nell:
Find the slope of the line with equation 7-3y=2x
Answered by Stephen La Rocque.
Out of school applications of Pythagoras Theorem 2008-01-23
From Laura:
Hi, I am currently working on a math summative in which I have to choose a real life subject and relate it back to the material in my grade 12 math class. I find the history and discovery behind the Pythagorean Theorem and Identity very interesting, but I have yet to find a real-life application of the equations. Yes, I know they are used for finding distances, heights etc., but realistically, how many people actually use it in those situations? Very few. I was hoping for a new application. Is the pythagorean theorem (sin^2x + cos^2x = 1) even applicable? Thank you, Laura
Answered by Harley Weston.
A parallelogram and a rhombus 2008-01-22
From miguel:
i have a problem proving a parallelogram a rhombus.. if a diagonal of a parallelogram bisects an angle of the parallelogram , then its a rhombus prove
Answered by Stephen La Rocque and Walter Whiteley.
1/2, 1/2, 3/8, 1/4, 5/32, 3/32, 7/128 2008-01-22
From Neil:
Find the next two terms in the following number sequence

1/2, 1/2, 3/8, 1/4, 5/32, 3/32, 7/128

Find a general rule for the nth term of the sequence

Answered by Penny Nom.
The equation of a line 2008-01-20
From Sara:
The question is:

Write the equation of a line that is parallel to y=3x-2 through the point (-5,9). I think the correct equation for this is y=3x+24. But I have to write it in standard form, and I can't seem to figure out how to do that. Please answer soon! Thanks!

Answered by Stephen La Rocque and Penny Nom.
The line through (-22 ;-1) and (-23, -2) 2008-01-20
From Zachery:
i have a slope that equals 1/1 and my coordinates are (-22;-1) and (-23;-2) and i am trying to change it into the equation y=mx+b and the farthest that i've gotten is y=1x_ (i can not find out how to find b.) So i was wondering if you could help me find out what b is because i have got no clue!!!!!
Answered by Penny Nom.
The game of 24 2008-01-17
From Raina:
I have four numbers 3 3 9 5 that need to equal 24 using addition subtraction multipliction and division and I'm so stuck.
Answered by Chris Langdon, Stephen La Rocque and Claude Tardif.
The angles of a triangle given the three sides 2008-01-17
From Lucy:
Is there a way to find the angles of a triangle just by knowing the lengths of it's sides? It seems like the would be a relationship between the two, but I'm not sure.
Answered by Stephen La Rocque and Harley Weston.
Explaining the factoring for the difference of cubes 2008-01-16
From Bill:
A student asked me where did the "difference of cubes" and "sum of cubes" come from. I did not have an answer for her. She is very bright and understands how they work but wanted to know where they derived from. Any help you can offer would be great. Thanks
Answered by Stephen La Rocque.
Two solutions using the law of sines 2008-01-14
From Kate:
I am working on the Law of Sines and I have a problem that says: Find a value for b so that the triangle has 2 solutions.

I am given that A = 36 degrees and a = 5. Now, I learned that for a triangle to have 2 solutions, h < a < b. BUT...my answer key says the answer is: 5 < b < 5/sin 36. I can't figure out how to make this fit with h < a < b.

Answered by Harley Weston.
How far is the jet from the lighthouse? 2008-01-07
From Natalie:
Question: A ship spots a lighthouse that is 53m high, at an angle of elevation of 7 degrees that is directly north of the ship. The same ship spots a jet travelling N62E at an altitude of 1500m with an angle of elevation of 15 degrees. How far is the jet from the lighthouse?

Natalie

Answered by Harley Weston.
The standard normal distribution 2007-12-29
From GEORGE:
Statistics texts state that in a normal distribution, 1 standard deviation covers 68%, 2SD 95% 3SD 99.7%. However, on looking at the tables for normal curve areas, the percentage for a z value of 1 is 84.13. 68 % of values are covered at a z value of 0.47. The discrepancies exist even for other values but are smaller. So why is the rule of thumb so different from the Table for Normal Curve areas?
Answered by Harley Weston.
Profit percentage (markup and margin) 2007-12-18
From Laura:
If my husband says he sold $11,000 worth of tools and made $1400 on the deal, what is the percent of profit?
Answered by Stephen La Rocque.
How many houses must they paint? 2007-12-12
From Natalie:
The question is: Four boys work together,painting houses for the summer.For each house they paint they get 256 pounds. If they work for four months of the summer and their expenses are 152 pounds per a month, how many houses must they paint for each of them to have one thousand pounds at the end of the summer?
Answered by Penny Nom.
System of equations 2007-12-06
From Jenn:
change the equation,x-y=4 to form y=mx+b the solution to the system of equations y=2x and y=-x+3 is
Answered by Stephen La Rocque.
Chicken and goat feet 2007-12-05
From Kim:
Old McDonald raises goats and chickens. The animals have a total of 100 heads adn 360 feet. How many goats and how many chickens does Mr. McDonald have?
Answered by Stephen La Rocque and Penny Nom.
(32-2 x 5) divided by 2 + 8 2007-11-28
From Kim:
Solve

(32-2 x 5) divided by 2 + 8

Answered by Leeanne Boehm.
A curve sketch 2007-11-22
From Ahson:
Find critical points, determine the monotonicity and concavity and sketch a graph of f(x) with any local maximum, local minimum and inflection points labeled:

1. f(x) = x^4 - x^3 - 3x^2 + 1

Answered by Harley Weston.
Bundles of wood 2007-11-19
From Darcy:
When you purchase a "bundle" of wood at the store, it is marked .75 cubic feet. What is the formula for figuring out that measurement for cubic feet and what fraction of a true cord of wood (128 cubic feet) is it. I would appreciate the formula so I can build a spreadsheet to help my secretaries and myself be in compliance to the law when selling firewood. Thanks.
Answered by Harley Weston.
Confidence level 2007-11-19
From Fara:
It is common for public opinion polls to have a " confidence level" of 95%, meaning that there is a 0.95 probability that the poll results are accurate within the claimed margins of error. If six different organizations conduct independent polls, what is the probability that all six of them are accurate within the claimed margins of error? Does the results suggest that with a confidence level of 95%, we can expect that almost all polls will be within the claimed margin of error?
Answered by Harley Weston.
Find the radius of a circle given the center and a point on the circle 2007-11-18
From Raymund:
Find the radius if the center is at (0, -5) and one point on the circle is (2,3)
Answered by Stephen La Rocque.
Expand (a^4 - b^4) 2007-11-17
From Saif:
how would you expand (a^4 - b^4) ???
Answered by Stephen La Rocque and Victoria West.
A geometric proof 2007-11-16
From Julie:
Prove that tangents to a circle at the endpoints of a diameter are parallel. State what is given, what is to be proved, and your plan of proof. Then write a two-column proof.
Answered by Walter Whiteley.
Why do they use the letter b in y=mx+b? 2007-11-15
From virginia:
why do they use the letter b in y=mx+b? im asking this because my math teacher said he will give extra points to anyone who finds this out first and i was hoping you guys would be the ones who could help.
Answered by Penny Nom.
Maximize his profit 2007-11-12
From apoorva:
During the summer months Terry makes and sells necklaces on the beach. Last summer he sold the necklaces for $10 each and his sales averaged 20 per day. When he increased the price by $1, he found that he lost two sales per day.

a. Find the demand function, assuming it is linear.
b. If the material for each necklace costs Terry $6, what should the selling price be to maximize his profit?

Answered by Penny Nom.
6 consecutive multiples of 6 2007-11-11
From jeff:
find 6 consecutive multiples of 6 whose sum is the least common multiple of 13 and 18
Answered by Penny Nom.
Questions of the Month 2007-11-01
From Judith:
Where would I find comprehensive questions, that cover the majority of skills in a unit such as number sense, to use as "Questions of the Month"?
Answered by Victoria West.
The rate of change of the concentration of a solution 2007-10-30
From Nicholas:
A barrel initially has two kg of salt dissolved in twenty liters of water. If water flows in the rate of 0.4 liters per minute and the well-mixed salt water solution flow out at the same rate, how much salt is present after 8 minutes? I tried working backwards given the answer but I can't seen to get their answer of ~1.7kg. Any help would be great! Thanks
Answered by Harley Weston.
Two mirrors 2007-10-24
From Peter:
The reflecting surfaces of two intersecting flat mirrors are at an angle θ (0° < θ < 90°). For a light ray that strikes the horizontal mirror, show that the emerging ray will intersect the incident ray at an angle β = 180° – 2θ.
Answered by Stephen La Rocque.
Turn 81 into the power of three 2007-10-22
From Victoria:
I do not understand how to turn 81 into the power of three.
Answered by Penny Nom.
Maximize profit 2007-10-22
From Dina:
A meat market purchases steak from a local meat packinghouse. The meat is purchased on Monday at a price of $2 per pound, and the meat market sells the steak for $3 per pound. Any steak left over at the end of the week is sold to a local Zoo for $0.50 per pound. The demand for steak and the probabilities of occurrence are as follows: Demand Probability
      20          10%
      21          10%
      22          15%
      23          20%
      24          20%
      25          15%
      26          10%
Determine the amount of stock to maximize the profit. Draw the graph and explain.

Answered by Penny Nom.
The rate of change of the area of a triangle 2007-10-22
From Ahlee:
So my question is: The included angle of the two sides of a constant equal length s of an isosceles triangle is ϑ.
(a) Show that the area of the triangle is given by A=1/2s^2 sinϑ
(b) If ϑ is increasing at the rate of 1/2 radian per minute, find the rate of change of the area when ϑ=pi/6 and ϑ=pi/3.
(c) Explain why the rate of change of the area of a triangle is not constant even though dϑ/dt is constant

Answered by Penny Nom.
Order of operations 2007-10-17
From Devon:
What function precedes the other? ie; 18 - 4 x2 =
Answered by Penny Nom.
Four triangles in a square 2007-10-15
From Kristina:
A square with side lengths of 6 cm is divided into 3 right triangles and a larger isosceles triangle. If the three right triangles have equal area, find the exact area of the isosceles triangle.
Answered by Stephen La Rocque.
15 divided by 5a 2007-10-12
From Susan:
Is "5a" considered an expression to be solved first, or do you simply go in PEMDAS order with the 15 divided by 5 in the following problem: Evaluate when a = 2 15 divided by 5a
Answered by Penny Nom.
How much water is in a hose? 2007-10-12
From Bryan:
I need to know how much water (gal) is in a hose thats 100' long by 5" in dia. Thanks
Answered by Stephen La Rocque.
The average rate of change of a function 2007-10-11
From vern:
Find the average rate of change of the function over the given interval. Compare this average rate of change with the instantaneous rates of change at the endpoints of the interval. f(X)=sinX for the inverval [0,pi/6]?
Answered by Harley Weston.
Substitution method 2007-10-11
From Kevin:
3xx+2y=-36-y=11
Answered by Stephen La Rocque.
Area of a quadrilateral 2007-10-10
From Courtney:
how would i find the area of a quadrilateral..
the sides are a (/) is 6cm, b (—) is 9 cm, and c (\) is 7 cm..
the angle between a and b is 140 degrees and b and c is 115 degrees..

Answered by Stephen La Rocque.
Simplifying algebraic expressions 2007-10-09
From Sakeena:
(2^2*3)^x+1/2^2x*3x
Answered by Stephen La Rocque.
Solving four simultaneous equations (system of four linear equations) 2007-10-07
From Johan:
I need some help in solving this question
x + 2y - 3z + 4w = 12
2x + 2y - 2z + 3w = 10
0 + y + z + 0 = -1
x - y + z - 2w = -4

Answered by Stephen La Rocque.
Solving arithmetic problems in the right order (BEDMAS) 2007-10-05
From Kim:
3+4x2-(10 divided by 5)= what?
Answered by Penny Nom.
Maximizing profits II 2007-10-05
From a student:
Suppose there are three firms with the same demand function. The function is Q=1000-40P. Each firm also a a cost function.
Firm 1: 4000+5Q, Firm 2: 3000+5Q, Firm 3: 3000+7Q.
What price should each firm charge if it wants to maximize profits.

Answered by Harley Weston.
Maximizing profit 2007-10-05
From a student:
Use the following equation to demonstrate how a firm that produces at MR=MC can also maximize its total profit. The equations to use are P=170-5Q TC=40+50Q+5Q^2
Answered by Harley Weston.
y squared over 3 times 8 over y 2007-09-30
From John:
y squared over 3 times 8 over y
Answered by Stephen La Rocque.
The area of a roof 2007-09-27
From RANDY:
I have an area of roof. It's in a sort of triangle. it's 25' across the top 30' at each edge 86' across the bottom
Answered by Stephen La Rocque.
The game of 24 2007-09-27
From Amanda:
I'm really stumped on a 24 math game question from math class. The numbers are 7,11,17,and 20. Help??
Answered by Claude Tardif.
Cubic meters, square feet and office space 2007-09-25
From Kathryn:
Can someone please help and let me know how do you convert 11 cubic meters into square footage?
Answered by Steve La Rocque and Harley Weston.
Prove that any two consecutive integers are relativley prime. 2007-09-18
From Michael:
Im not very good at proofs and I was wandering if you would be able to help me with the following question: Prove that any two consecutive integers is relativley prime. Thanks a million.
Answered by Penny Nom.
A 5 by 5 checkerboard 2007-09-17
From Darren:
Hi, I'm Darren and i have some questions to ask you about this problem: In a 5 by 5 checkerboard : how many 2 by 2 squares are there, what other sizes of squares do you need to count and how many of of each size of squares can you find; how many squares did you find in all
Answered by Victoria West.
Two equations in two unknowns 2007-09-14
From CAMEILA:
NEED T0 SOLVE THIS EQUATION

Y = -2X + 6
Y + 6 = 2x

Answered by Stephen La Rocque.
The area of half a circle 2007-09-14
From Heather:
We need the formula for area of half circle please explain where each number comes from.
Answered by Penny Nom.
The slope of a line 2007-09-06
From Danielle:
Hello. I need some help with this: i am supposed to find the equation of a line with a point (2/3,5) and (-5/6,-4) the answer in the book says y=6x+1. My question is how did they get for the slope?
Answered by Penny Nom.
How many two digit numbers contain at least one 7? 2007-09-06
From Janet:
How many two digit numbers contain at least one number seven?
Answered by Penny Nom.
Fractions, ratios and percentages all mixed together 2007-08-29
From Charon:
Example:
x : 1/4% :: 9 3/5 : 1/200

Answered by Stephen La Rocque.
Two-column proof for a circle geometry problem 2007-08-24
From Kendra:
i have to prove that tangents to a circle at the endpoints of a diatmeter are parallel by stating whats given, whats to prove and a plane, then write a two column proof i dont understand this
Answered by Stephen La Rocque.
A geometry problem 2007-08-20
From samhita:
ABC is a triangle. Let D be a point on side BC produced beyond B such that BD=BA. Let M be the mid-point of AC. The bisector of angle ABC meets DM at P. Prove that angle BAP=angle ACB.
Answered by Chris Fisher.
The length of the third side of a triangle 2007-08-15
From Brooklyn:
What is the equation to find the length of the third side of a triangle if you have the length of A, B, and the angles(s)?
Answered by Stephen La Rocque.
Twenty dining tables 2007-08-13
From priya:
Utkarsh bought 20 dining tables for Rs 120000 and sold these at a profit equal to the S.P of 4 dining tables.Find the S.P of one dining table.(give the answer in statements)
Answered by Stephen La Rocque.
Induction - divisibility 2007-08-04
From Jerry:
How would you prove that for any positive integer n, the value of the expression 3^(2n+2) - 8n -9 is divisible by 64.
Answered by Chris Fisher and Penny Nom.
What fraction of the letters of the alphabet is each word? 2007-08-02
From Clayton:
How do I figure this out, the math question has the answers but I have no idea how to get it. my Mom can't figure it out either.

What fraction of the letters of the alphabet are each word
Man = 3/26
Glasses = 7/26
Integrity = 1/4
computer = 2/9

Answered by Paul Betts and Harley Weston.
The height of a pole 2007-08-02
From lalaine:
Hi, this is my problem.. From a point 50.2 m to the pole, a student measured the angle of elevation to the top of the pole to be 32°. Find the height of the pole if the student's height from his feet to his eyes is about 4 ft.
Answered by Penny Nom.
Multiplcation of two negative numbers 2007-07-26
From Brett:
Someone asked a question about multiplication and division of two negative numbers yielding a positive result here: http://mathcentral.uregina.ca/qq/database/QQ.09.99/butler1.html I was not fully happy with the explanation b/c I want to give me daughter a real-world example and I can't seem to find one.

The following illustrates why multiplying negative numbers has become difficult to explain:

2 X 2 = 4

----(-4)---(-2)---0---2---4
In this example we start with 2 and then want 2 more of them. When we move across the number line from 2 to our answer, which is four, we have moved only 2 units to the right.

-2 X -2 = 4

----(-4)---(-2)---0---2---4
In this example we start with -2 and then want -2 more of them. When we move across the number line from -2 to our answer, which is four, we have moved 6 units to the right.

How can the phenomenon of multiplying two negative numbers being more powerful than multiplying two positive numbers be explained? -Brett

Answered by Stephen La Rocque and Harley Weston.
Simplifying an algebraic fraction expression 2007-07-25
From Jessica:
How do I simplify b/(b2-25) + 5/(b+5) - 6/b?
Answered by Stephen La Rocque.
Angle of depression 2007-07-23
From joyce:
hello, here is my problem......
As you stand on a bridge w/c is 100 ft. above the water you are looking @ an approaching barge. If the A of top of the front of the bridge is 29.04 degrees and the angle of depression of the rear is 17.36 degrees . Find the length of the barge?

Answered by Harley Weston.
What fraction of the company does Mary own? 2007-07-18
From Bridget:
Sam and Mary each owned one-half stock in a printing company. Sam sold 2/3 of his stock to Mary. What fractional part of the printing business does Mary now own?
Answered by Stephen La Rocque.
Angle of depression (declination) between sailboat observations 2007-07-18
From Joyce:
From a cliff 150 ft above a lake, we see a boat sailing directly towards us. The angle of depression of the boat is seen to be 5 degrees and 7 inches and 11 degrees and 18 inches. Find the distance sailed between observations.
Answered by Stephen La Rocque.
Angle of inclination from the horizontal 2007-07-18
From Joyce:
In flying upward for 1260 yards along a straight inclined path airplane rises 156 yards. Find the climbing angle ( the angle of inclination from the horizontal) Thank you in advance
Answered by Stephen La Rocque.
Calculating the area (acreage) of a four sided lot 2007-07-18
From A property owner:
I have a real estate property and the lot size is something I need to find out. I know the lengths of the four sides, but it isn't a rectangle, it is an odd shape. How do I find the acreage?
Answered by Stephen La Rocque.
Height of a tower from two observations 2007-07-16
From joyce:
An observer wishes to determine the height of a tower. He takes sight @ the top of the tower from A & B w/c are 5oft. apart @ the elevation on a direct line w/ the tower. The vertical angle @ point A is 30 degrees & the point B is 40 degrees. What is the height of the tower? Find the value of x in angle tangent 40 degrees and 30 degrees? Show the solution of the value of x?
Answered by Stephen La Rocque.
Height of an antenna (angle of elevation) 2007-07-16
From Fhay:
An antenna stands on the edge of the top of a 52 story building from a point 320 ft. from the base of the building, the angle of elevation to the top of the antenna is 64 degrees in each story is 12 ft. high. Find the height of the antenna
Answered by Stephen La Rocque.
The two towers (angles of elevation trigonometry) 2007-07-14
From joyce:
The angle of elevation of tower B from the top of tower A is 28 degrees and the angle of elevation of the top of tower A from the base is 46 degrees Find the height of tower A if tower B is 120 m high?
Answered by Stephen La Rocque.
Proving a quadrilateral is a rectangle 2007-07-14
From Sonja:
I was having this discussion with another teacher and we need a third opinion. When you are trying to prove a quadrilateral is a rectangle which method should you use:
  1. Prove the shape is a parallelogram by doing slope 4 times by stating that parallel lines have equal slopes. Then proving a right angle by stating that perpendicular lines have negative reciprocal slopes.
  2. Doing the slope 4 times and stating that the shape is a rectangle because opposite sides are parallel because of equal slopes and it contains a right angle because of negative reciprocal slopes.
I guess the real question is do you have to first state that the shape is a parallelogram?

Answered by Stephen La Rocque.
Any regular polygon inscribed in a circle 2007-07-12
From DJ:
Circle with r=12" is inscribed in a regular octagon. What is the length of each octagon segment? Note: Our answer works for any regular polygon inscribed in any circle.
Answered by Stephen La Rocque.
Grams of vodka 2007-07-10
From Andrew:
milliliters to grams..vodka 80 proof?
Answered by Stephen La Rocque.
Derivative of a Function 2007-07-09
From Bob:
What is the derivative of the function a sub n = [n/(n+1)]^n ?
Answered by Stephen La Rocque.
Proof that any side of a triangle is less than half the perimeter. 2007-07-07
From Omkar:
Any side of a triangle is smaller than half of its perimeter, prove this in short ?
Answered by Stephen La Rocque.
Splitting the profits unequally 2007-06-27
From Maci:
We recently hosted a fundraiser. Not everyone will receive a whole share of the profits because they did not participate the entire time. Several will receive only 1/2 or 1/3 share. How do I divide the profits?
Answered by Stephen La Rocque.
Selling price and revenue 2007-06-24
From nick:
hello, this is my first time asking for help from this website in which by the way, i think is great for everyone. My question is as follows... the relationship between the selling price of a sleeping bag, s dollars, and the revenue at that selling price, r(s) dollars is represented by the function: r(s)= -10s^2+1500s
evaluate, interpret, and compare: a) r(29.95)

Answered by Stephen La Rocque and Penny Nom.
Simplifying complex denominators 2007-06-21
From Krys:
How do I simplify completely? ((4+i ) / (3+i )) - ((2-i ) / (5-i ))
Answered by Stephen La Rocque.
Simplifying a quartic rational expression using long division 2007-06-14
From Megan:
x+2/12x^4+17x^3+0x^2+8x-40=
Answered by Stephen La Rocque and Penny Nom.
Angles of depression 2007-06-13
From Phonda:
The pilot of a small private plane can look forward and see the control tower for a small airstrip. Beyond that is a large factory that is 3 milies from the airstrip. The angles of depression are 12.5 degrees and 4.8 degrees respectively. Find the airplane's altitude, to the nearest ten feet.
Answered by Stephen La Rocque.
The area of a quadrilateral 2007-06-10
From Lucy:
Calculate the area of the quadrilateral ABCD. AB= 4.1cm, BC = 7.6cm, AD= 5.4 cm, CD= ? Angle ABC = 117, Angle ADC = 62. Give your answer correct to 3 significant figures.
Answered by Stephen La Rocque and Penny Nom.
The law of sines 2007-06-09
From Felicia:
A parallelogram has one side that is 12.0 cm and one angle that is 65°. The shorter diagonal is 25.0 cm. To the nearest tenth of a centimetre, how long is the other side of the parallelogram? Use the sine law.
Answered by Penny Nom.
Rate of change of distance between the clock hands 2007-06-05
From Jonathan:
A certain Clock has a minute hand with a length of 4 inches long and an hour hand with a length of 3 inches long. How fast is the distance between tips of these hands changing at 9:00?
Answered by Stephen La Rocque.
What happens when you have zero's on both sides? 2007-06-05
From Lily:
On the substitution method what happens when you have zero's on both sides of the equation? Is that considered no solution or infinitely many?
Answered by Stephen La Rocque and Penny Nom.
Area of an isosceles triangle 2007-06-01
From Josh:
In a previous question answered by Sue regarding the area of a regular polygon you gave a formula for the area of an isosceles. My question is how did you get this formula? Can you please explain to mean the process that you used to get that formula? Thanks
Answered by Stephen La Rocque.
The intersection of two planes 2007-05-31
From Sarim:
How to find a intersection of two planes?
Answered by Penny Nom.
Fractions, decimals and profit 2007-05-29
From fatima:
how can i change a fraction into a decimal and how do i change a decimal into a fraction................
anther question please what does making a profit have to do with percentage change!!!!!!

Answered by Penny Nom.
System of equations 2007-05-24
From Chris:
Find all real solutions (x,y,z,w) of the system of equations:
2y= x + x/17, 2z= y + y/17, 2w = z + z/17, 2x= w + w/17

Answered by Penny Nom.
Find the sample size needed 2007-05-13
From Mini:
Find the sample size needed to be 98% confident thata marketing survey on the proportion of shoppers who use the internet for holiday shopping is accurate within a margin of error of 0.02. Assume that the conditions for a binomial distribution are met, and that a current estimate for a sample proportion does not exist.
Answered by Penny Nom.
Find the equation of a line passing through two given points 2007-05-12
From Kenzie:
I am having problems solving an equation of the line passing through (-2,4) and (6,0)
Answered by Penny Nom and Stephen La Rocque.
Angle of Elevation 2007-05-10
From Micky:
Two Buildings are on opposite sides of a street 40 feet wide. The taller of the two buildings is 580 feet tall. The angle of depression from the top of the tallest building to the shorter building across the street is 57 degrees. Find the height of the shorter building.
Answered by Stephen La Rocque.
Are proofs important in geometry? 2007-05-07
From BJ:
Are proofs very important to know how to do?
My daughter has been in Geometry & the teacher skipped proofs.

Answered by Penny Nom.
What is the square root of 729 to the third power? 2007-04-25
From Tori:
What is the square root of 729 to the third power?
Answered by Penny Nom.
The size of a freezer 2007-04-23
From Jacqui:
How many can packages can fit? I have a freezer that is 7cf. But it contains a little tray component. The freezer measurements without the tray is(bear with me, I'm not good with LxWxD) 23 across x 16 W x 25 deep (I'm assuming depth) and the tray is 8 across x 16 w x 15 D. We are trying to see how many 9 L x 6 W x 3.5 D packages can fit in the freezer. Please help me. I'm getting app. 60. I need 84 to be able to fit.
Answered by Penny Nom.
Ax + By = C 2007-04-18
From Diana:
find an equation in general form Ax + By = C with the following properties passing through points (a,b) and (2a, 2b)
Answered by Penny Nom.
The area of a heptagon 2007-04-14
From Chantel:
Would you be able to tell me the formula to work out the area of a heptagon? thank you.
Answered by Stephen La Rocque.
The angle of depression 2007-04-14
From Mary-Beth:
Two towers are 30 m apart. From the 15th floor, 40 m up, find the angle of depression to the base of the taller tower? I think the answer is 53, but the answer book says 37.
Answered by Penny Nom.
Intersection of a line and a circle 2007-04-12
From gaby:
The sum of two numbers is 9. The sum of the squares of the two numbers is 41. Find the numbers.
Answered by Steve La Rocque and Melanie Tyrer.
A volume of revolution 2007-04-08
From christina:
find the volume of the solid formed when region bounded by y=x/3, y=2 and the y-axis. it is revolved about the x-axis.

the assignment was to use both the washer method and the shell method but when i solved for the volume, i got different answers. i think my shell method is wrong because i know i'm having difficulties with using "dy" instead of "dx" here's my work so far:

Answered by Penny Nom.
3 divided by 3 to it's fifth root 2007-04-06
From Annie:
How do I transform the equation 3 divided by 3 to it's fifth root to simple radical form (getting the radical out of the denominator)?
Answered by Penny Nom.
How much did it originally cost? 2007-04-04
From Megan:
My question is simple:

"-Jim is able to sell a hand-carved statue for $650 which was a 35% profit over his cost. How much did ie originally cost?"

I don't even know where to start could you help?

Answered by Penny Nom.
Game of 24 2007-04-04
From teri:
How do you find the answer of 24 by using 12,24,7 and 2 only once.
Answered by Stephen La Rocque.
Using the "difference of squares" formula how do I compute 27 * 33? 2007-04-02
From Sarah:
Using the "difference of squares" formula how do I compute 27 * 33?
Answered by Penny Nom.
Is the inverse of a function always a function? 2007-03-29
From San:
Is the inverse of a function always a function? Please justify. Thank You!
Answered by Penny Nom.
Factoring a trinomial 2007-03-27
From Kim:
Hi, could you please help me solve this.

3x(squared) +20x - 7

Answered by Leeanne Boehm.
The distance between two fire towers 2007-03-23
From tony:
Two fire towers are 30km apart, tower A is due west of tower B. A fire is spotted from the towers, and the bearing from A and B are N76degreesE and N56degreesW, respectively. Find the distance from the fire to the straight line connecting tower A to tower B.
Answered by Stephen La Rocque.
Graphing a line 2007-03-23
From mitchell:
Graph each equation. y = 2x + 4
Answered by Jaymi Peterson and Haley Ess.
The law of cosines 2007-03-23
From chetna:
Q 1) In triangle LMN, l=7, m=5 , n=4. find ANGLE N. After applying the rule and substituting values i'm getting Cos n= 58/40. Is there something wrong. The answer at the back of the book is 34 degrees.
Answered by Penny Nom.
Angles of depression 2007-03-21
From romaine:
a woman of height 1.4m standing on top of a building of height 34.6m veiws a tree some distance away. she observes that the angle of depression of the bottom of the tree is 35 degrees, and the angle of depression of the top of the tree is 29 degrees. assume that the building and the tree is on level ground.
(a) calculate the distance of the woman from the top of the tree measured along her line of sight.
(b) determine the height of the tree.

Answered by Stephen La Rocque.
Angle of elevation 2007-03-13
From Joslyn:
A ship at sea sights a 12m high lighthouse on a cliff which is 80m above sea level. If the angle of elevation to the top of the lighthouse is 27 degrees, calculate the distance from the ship to the shore.
Answered by Haley Ess.
What is pi? 2007-03-09
From Billy:
What is pi?
Answered by Jaymi Peterson and Sara Ulmer.
At what rate is the area of the triangle changing? 2007-02-24
From mac:
two sticks 3.5 feet long are hinged together and are stood up to form an isosceles triangle with the floor. The sticks slide apart, and at the moment when the triangle is equilateral, the angle is increasing at the rate of 1/3 radian/sec. At what rate is the area of the triangle increasing or decreasing at that moment?
Mac

Answered by Penny Nom.
Find the area of the triangle 2007-02-20
From Christina:
Graph the function f(X)= x+1/x-1 and graph the tangent line to the function at the points A:(2,3) and B:(-1,0). The point of intersection of the two tangent lines is C. Find the area of the triangle ABC.
Answered by Stephen La Rocque.
Surface Area of a cone 2007-02-19
From Cari:
I am doing a math project. I am very confused on how to find the surface area of a cone. I have looked at other equtions but i still don't understand. How do you find the surface area of a cone that has a 15cm length and a 3.5cm radius?
Answered by Penny Nom.
Factoring polynomials 2007-02-14
From Joe:
I am in the eighth grade, and we are learning the equivalent of Algebra 2. I have no ides how to factor (x-2)(x^2-1)-6x-6 You help is most aprreciated. Thank you! Joe
Answered by Stephen La Rocque.
The surface area of a dome 2007-02-11
From chris:
I need to figure the surface area of a dome which is 96" across and 48" tall. Could you help please? Thanks!
Answered by Penny Nom and Melanie Tyrer.
Large exponents 2007-02-09
From Nick:
I am trying to figure out an extremely large number. It relates to the estimated number of bacterial divisions in 12775 generations of bacteria.
The problem I need to solve is:
2^12775 or 2 to the power of 12,775.

Answered by Stephen La Rocque and Penny Nom.
The area for a pentagon 2007-02-09
From nicole:
how do you find the area for a pentagon
Answered by Stephen La Rocque.
The capacity of a tank 2007-02-08
From lorn:
what is the capacity of a tank height is 110 feet diameter is 24 feet
Answered by Brennan Yaremko.
An even positive integer cubed minus four times the number 2007-02-07
From Rachael:
I can't figure out the proof or the method to get the proof for this question: any even positive integer cubed minus four times the number is divisible by 48
Answered by Haley Ess and Penny Nom.
How to calculate margin of profit???? 2007-02-03
From A student:
If cost price is Rs. 100/- and our selling price is 139/-. I want to know the gross profit & how much margin in this????
Answered by Penny Nom.
The volume of a cone 2007-01-31
From ajay:
WHY VOLUME OF CONE IS ONE THIRD OF THE VOLUME OF CYLINDER?
Answered by Penny Nom.
The height of an isosceles triangle 2007-01-27
From Brendan:
I need to find the height of an isosceles triangle whose angles are 52, 52 and 76 degrees. The base is 100, and the two equal sides are unknown. How would I go about this?
Answered by Stephen La Rocque.
The area of a ellipse 2007-01-25
From Ranjit:
I have a task in which i have to find the area of a ellipse. i find this difficult because i have only been provided with the perimeter, which is 1000m.
Answered by Chris Fisher.
An algebra exercise 2007-01-24
From Clara:
Given x + 2y = -2 and x - 2y = 18, find:
a) x^2 - 4y^2
b) x^2 + 4y^2

Answered by Stephen La Rocque and Penny Nom.
What is the point of intersection of two sides of a polygon? 2007-01-23
From Tyler:
What is the point of intersection of two sides of a polygon?
Answered by Stephen La Rocque.
Is -2^2 = -4 or 4? 2007-01-23
From Joan:
Is -2 squared, when written without parentheses around the -2, -4 or could this correctly be solved by squaring -2 (-2 x -2) for an answer of 4? Or, to correctly get an answer of 4, would the problem have to read (-2) squared?
Answered by Stephen La Rocque.
Maximizing profit 2007-01-23
From Denise:
Total Profit= Total Revenue-Total Cost P(x)=R(x)-C(x) Where x is the number of units sold. Find the maximum profit and the number of units that must be sold in order to get that profit. R(x)=5x C(x)=.001x^2+1.2x+60
Answered by Stephen La Rocque.
Calculating a person's volume 2007-01-20
From James:
Hi i am currently trying to calculate my volume. I have tried to do this using a tank of water and measuring the difference. i worked out that my volume is 112 litres. my question is that i want to make a box that would hold my volume and so i need to convert it into centimeters. i know that there are 100 cm^2 in a litre. if i divide this by 4 i would be 91x91x91ft. is this simply to do with density. if so how could i make this more realistic?
Answered by Stephen La Rocque.
A formula for figuring a tanks volume at various levels 2007-01-20
From Dave:
I know there has to be a general formula to figure out a tanks volume where as you know the total volume but you need to know what it will hold at various levels measured in 1/8th inches in depth and that you may be able to plug this formula into a program like say excel and thus create a chart unique to that tank. But for the lack of me I can not figure it out.
Answered by Stephen La Rocque.
Irregular multi sided polygon 2007-01-20
From Graham:
I have an irregular polygon. I know the length of all the sides and know approximately all the internal angles. Is there a formula or table that can calculate the area for me?
Answered by Steve La Rocque, Chris Fisher and Penny Nom.
The area of an irregular pentagon 2007-01-18
From Ranjit:
I would like to find the area of an irregular pentagon where the total perimeter is 1000m. Is there a Heron's formula for this?
Answered by Stephen La Rocque.
Answer to skill testing question 2007-01-18
From Mike:
This question was on a ballot. Is the right answer 36 or 1? 5+2-1X6=
Answered by Stephen La Rocque.
Points of intersection 2007-01-18
From Mark:
Find the points of intersection of the given pair of curves. 2x - 3y = -8 and 3x - 5y = -13
Answered by Steve La Rocque and Haley Ess.
The height of a triangle 2007-01-17
From Sharon:
I am needing to determine the height of a triangle with a 50 ft base and one side is 40 ft the other side being 30 ft.
Answered by Penny Nom.
The volume of a frustum of a pyramid 2007-01-17
From Sam:
Find the volume of a frustum of a pyramid with square base of side b, square top of side a, and height h.
Answered by Penny Nom.
The volume of a sphere 2007-01-16
From Anshul:
How to prove or find the volume of sphere?
Answered by Harley Weston.
Where do the medians of a triangle meet? 2007-01-02
From Brittany:
Where do the medians of a triangle meet?
1. Inside the triangle
v 2. On the traingle
3. Outside the traingle

Answered by Stephen La Rocque.
Prove that square root of 3 is irrational. 2006-12-21
From Humera:
Prove that square root of 3 is irrational.
Answered by Stephen La Rocque.
The product of of two rotations 2006-12-17
From Katie:
Is the product of of two rotations over a different center point always a translation?
Answered by Walter Whiteley.
cos(n)pi = (-1)^n 2006-12-14
From Idrees:
How can I prove the following: cos(n)pi = (-1)^n
Answered by Steve La Rocque.
The absolute value of imaginary and complex numbers 2006-12-11
From Keith:
i don't get how to find the absolute value of imaginary and complex numbers here is an examples from the text book the answers are given but they don't show the work so i can follow along just show me the work please and explain how it is done

problem
3+4i

Answered by Stephen La Rocque and Penny Nom.
Factoring m^4-9^n 2006-12-07
From Josh:
I can not figure out how to completely factor m^4-9^n.
Answered by Penny Nom.
The volume of an irregular tetrahedron 2006-12-06
From Hai Van:
Could you please show me the way to calculate the volume of an irregular tetrahedron
Answered by Penny Nom.
Star of Bethlehem 2006-12-05
From Stephen:
I need to construct a star for a Christmas play being done at our church. I want it to be a four-pointed star. The top and side points should be 12 inches long and the bottom point 24 inches. It will be flat on the back but raised on the front with lights inside for illumination. Each point will be made of two triangles with a ridge in the center. I would like the width of the rays at the center (widest point or center of star) to be about five inches and the center ridge at this same point to be raised about two inches from the flat back. I need to know the dimensions and angles of each of the 8 triangles that make up the star so that all of them meet properly at the center of the star so as to have the correct height from the flat back.
Answered by Penny Nom.
The height of a triangle 2006-12-01
From Jeni:
I need to know how to figure out the height of a triangle when all I know are the sides and base. The sides are 5 and the base is 8 What do I do?
Answered by Penny Nom.
The game of 24 2006-11-28
From John:
Math 24 with 9, 8, 4, 2?
Answered by Stephen La Rocque and Penny Nom.
What proportion of cups of coffee have at least one defect 2006-11-28
From Ayman:
coffee machine may be defective since it dispenses the wrong amount of coffee (C) & or the wrong amount of suger ( S ) the probability of these defects are P(C) = 0.05, P(S) = 0.04 & P( C&S ) = 0.01 find
1- what proportion of cups of coffee have at least one defect
2- what proportion of cups of coffee have no defect

Answered by Penny Nom.
Percentages 2006-11-23
From Hyaqcinth:
I just do not understand percentages...if the cost of a product is $1.20, how can I make a profit of 55% ? how do I calculate this problem?
Answered by Stephen La Rocque.
Profit margin 2006-11-22
From Kris:
I just started a new sales position and need to calculate profit margins based on my cost of a given product. For example I have a piece of equipment that costs me $102,077.73. I need to figure a profit margin of 18% from cost to arrive at my sale price. What is the best way to do this?
Answered by Penny Nom.
Factors 2006-11-22
From Jacqueline:
Why do the numbers 16,36,48,60,64,90 and 100 have an odd number of factors?
Answered by Stephen La Rocque and Penny Nom.
Composition of functions 2006-11-19
From RJ:
Let f0(x) = 2/2-x and fn+1 = f0 o fn for n greater than or equal to 0. Find a formula for fn and prove it by mathematical induction. Recall that o represents function composition. i.e., (f o g)(x) = f(g(x)).
Answered by Stephen La Rocque.
A particle moving along a curve 2006-11-18
From Rachel:
a particle is moving along the curve whose equation is: (xy^3)/(1+y^2)=8/5 assume the x-coordinate is increasing at the rate of 6 units per second when the particle is at the point (1,2).
a. at what rate is the y-coordinate of the point changing at that instant?
b. is the particle rising or falling at that instant?

Answered by Penny Nom.
Composition of functions 2006-11-18
From Oryan:
Given f(x)=-2x^3 and g(x)-4x-5, find g(f(-1))
Answered by Stephen La Rocque.
An aircraft and a missile 2006-11-18
From Sarah:
an aircraft is flying at a constant altitude with a constant speed of 600mph. an antiaircraft missile is fired on a straight line perpendicular to the flight path of the aircraft so that it will hit the aircraft at a point P. at that instant the aircraft is 2 miles from the impact point P the missile is 4 miles from P and flying at 1200 mph. at that instant, how rapidly is the distance between missile and aircraft decreasing?
Answered by Stephen La Rocque.
Wheat is poured on a conical pile 2006-11-17
From Rachel:
wheat is poured through a chute at the rate of 10 cubic feet per minute and falls in a conical pile whose bottom radius is always half the altitude. how fast will the circumference of the base be increasing when the pile is 8 feet high?
Answered by Penny Nom.
A system of equations 2006-11-13
From Katrina:
solve the system of equations using either the substitution or the elimination method. 2x-3y= -12 and -4x+6y=15
Answered by Penny Nom.
The rate of change of the perimeter of a square 2006-11-07
From Karli:
Find the rate of change of the perimeter of a square with respect to its area.
Answered by Stephen La Rocque.
A boat is being pulled towards a dock. 2006-11-06
From Cassie:
A boat is being pulled towards a dock. If the rope is being pulled in at 3 feet per second, how fast is the distance between the dock and the boat decreasing when it is 30 feet from the dock?
Answered by Penny Nom.
Profit margin 2006-11-04
From Beverly:
Our costs to make a product is $73.00. We want to sell this product and make a 30 percent profit margin. What is the selling price? What is the simple formula to follow? Is there a chart we can go to that shows us these percentage of profit breakdowns
Answered by Penny Nom.
Selling Price using Profit Margin 2006-10-29
From Chris:
What is the selling price of my inventory, find using the profit margin? Inventory A costs $2,300. The profit margin is 55% Inventory B cost is $4,500. The profit margin is 210%.
Answered by Penny Nom.
An odd number of factors 2006-10-28
From Anthony:
What is the common name used for numbers that have an odd number of factors? This answer should be a two word answer only.
Answered by Stephen La Rocque.
How much labor should the firm employ? 2006-10-28
From Christy:
A dressmaking firm has a production function of Q=L-L(squared)/800. Q is the number of dresses per week and L is the number of labor hours per week. Additional cost of hiring an extra hour of labor is $20. The fixed selling price is P=$40. How much labor should the firm employ? What is the resulting output and profit? I am having a difficult time with this, HELP!
Answered by Stephen La Rocque.
(a^(1/3) – b^(1/3)) ( a^(2/3) + a^(1/3)b^(1/3) + b^(2/3)) 2006-10-24
From Julie:
(a1/3 – b1/3) ( a2/3 + a1/3b1/3 + b2/3)
Answered by Haley Ess.
The area of a regular octagon 2006-10-22
From Farhana:
I have a regular octagon with sides of 0.8m and I need to find the area,
Answered by Karen McIver and Penny Nom.
The height of a triangle as a function 2006-10-19
From Ryan:
Let 2s denote the length of the side of an equilateral triangle. Express the height of the triangle as a function of s
Answered by Penny Nom.
How fast is the water level rising when the water is 1 meter deep? 2006-10-19
From Don:
The cross section of a 5-meter trough is an isosceles trapezoid with a 2-meter lower base, a 3-meter upper base and an altitude of 2 meters. Water is running into the trough at a rate of 1 cubic meter per minute. How fast is the water level rising when the water is 1 meter deep?
Answered by Stephen La Rocque.
(2324/4)-525+214 2006-10-15
From Sherin:
(2324/4)-525+214
Answered by Penny Nom.
A proof by induction 2006-10-02
From Zamira:
i'm studying induction but i don't get how to proof that 1+2+2^2+2^3+...+2^(n-1) = (2^n) - 1.
Answered by Penny Nom.
Prove that 2nCn is less than 4n, for all positive integers n? 2006-10-01
From Anna:
How can I prove that 2nCn is less than 4n, for all positive integers n?
Answered by Penny Nom.
Family of lines 2006-09-29
From Sasha:
could a "Family of Lines" have both parallel and intersecting equations that aren't related directly but otherwise related by each other to create a curve
Answered by Chris Fisher.
BEDMAS 2006-09-24
From Partick:
if you have a question like this
-(4)-(-4)+(-4)+4
how do you solve it step by step

Answered by Penny Nom.
How many items must the company produce to begin to make a profit? 2006-09-24
From Devon:
The profit a manufacturing company makes can be found using the formula P=120n - n^2 - 2200 How many items must the company produce to begin to make a profit?
Answered by Stephen La Rocque.
Parameters 2006-09-15
From Chase:
What is the meaning of the word "parameters" when used in reference to Algebra.
Answered by Penny Nom.
what is 8+(-2)(-5)/4-(-1)-4(-2+3) 2006-09-08
From Jordan:
what is 8+(-2)(-5)/4-(-1)-4(-2+3)
Answered by Paul Betts.
The velocity of a pendulum, part II 2006-09-07
From Erin:
We saw the question in your database about the velocity of a pendulum swinging.....It is the same exact question....but there is another question......it says....

"estimate the instantaneous rate of change of d with respect to t when t = 1.5. At this time, is the pendulum moving toward or away from the wall? Explain."

Answered by Harley Weston.
The lower half of a sphere 2006-08-30
From Yvette:
I am interested in finding out how to represent the lower half of a sphere in the form of z=f(x,y) with r=2 and a centre point (3,1).
Answered by Stephen La Rocque and Penny Nom.
16X27+32÷4-26 2006-08-20
From Amanda:
The question was 16X27+32÷4-26.
Answered by Stephen La Rocque.
Points of intersection 2006-08-20
From Gianella:

Find the points of intersection by solving this problem analytically.

y= x cubed
y= x


Answered by Stephen La Rocque.
Find the points of intersection 2006-08-18
From Ingrid:

The question says find the points of intersection of the graphs of the equation, and check your results analytically.

x squared + y squared=25
2x+ y=10


Answered by Stephen La Rocque.
The development of trigonometry 2006-08-15
From Eugene:
Can you please give the exact time line of trigonometry.
Answered by Penny Nom.
1/x + 1/y 2006-08-11
From Sonya:
what is 1/x+1/y = ? is it equal to 1/x+y or what?
Answered by Penny Nom.
A problem with exponents 2006-08-09
From A student:
(8a to the negative 2 b cube c to the negative 4/4a squared b to the negative 3 c squared) to the negative 2
Answered by Stephen La Rocque.
Order of operations 2006-06-29
From Jaden:

(1) A triangle has vertices at (-3,-3) (7,-3) and (-2,5). What is the area of the triangle?

Is there a way of doing this so that you dont have to graph it? If the x axis is horizontal can i say it is the base and the y-axis is horizontal meaning the height. Would I be able to do this...-3 to 7-->9 spaces, -3 to 5-->7 spaces-----9(7)63/2=31.5

(2) A cube has a surface area of 54 square centimeters. What is the volume of the cube in cubic centimeters?


Answered by Stephen La Rocque.
Order of Operations 2006-06-29
From Marcy:
I have been having problems with this skill testing question. I know the rules of operation but I don't understand why they would put it in brackets. Can you help me. I have two answers but I would like to know which one is right. Im having a disagreement with a friend over this matter. Thank you so much

[ (20 + 82 - 6) ÷ 16 ] x 14

Answered by Steve La Rocque and Paul Betts.
Marking out a circle 2006-06-28
From Peter:
given a straight line. how do i work out the off sets ( at right angles) at several intermediate points. to set out a 5.0m arc that has a 18.0m radius.
Answered by Stephen La Rocque and Penny Nom.
A trig problem 2006-06-24
From Greg:
A and B are two towers, B being 4 km due east of A. The true bearings of a flagpole, C, from A and B are α east of north and α west of north respectively. The true bearings of a second flagpole, D, from A and B are (α + β) east of north and (α - β) west of north respectively. Assuming A, B, C, and D are on level ground, and that α = 25, β = 10, find the distance between C and D.
Answered by Penny Nom.
The chord length of a polygon 2006-06-14
From Krishna:
I have to find out the chord length of a polygon - Tetradecagon ! The Radius of the Circle is 11.5 Cms. The Circle is intersepted by 14 arcs. Then how to find out the chord length?
Answered by Stephen La Rocque.
A volume of revolution 2006-06-07
From Colleen:
Find the exact volume in cubic units generated by rotating a region, R, around the
y-axis, given that R is the region bounded by the curve y = x3 and the lines x = 1 and
y = 8.

Answered by Penny Nom.
The coefficient of variation 2006-05-20
From Glenn:
What is the correct formula for coefficient of variation for a binomial distribution?
Answered by Penny Nom.
More on the game of 24 2006-05-05
From Kathy's daughter:
My daughter's 5th grade class plays the 24 math game. She came home the other day with a real stumper! The numbers on the card are 5,5,2,and 6. The rules say you must use every number,you may only use a number once,you can only add, subtract, multiply and/or divide and the answer is 24. Can you help!!!! No one where I work has been able to figure this one.
Answered by Paul Betts and Penny Nom.
Proof by induction 2006-04-24
From Meshaal:
Find an expression for: 1-3+5 - 7 + 9 - 11 + ... + (-1)^(n-1) * (2n-1) and prove that it is correct.
Answered by Stephen La Rocque.
Geometry proof 2006-04-23
From Jade:
From a point P outside a circle with centre O, tangents are drawn to meet the circle at A and B. a) Prove that PO is the right bisector of the chord AB. b) Prove that
Answered by Stephen La Rocque.
What is the original cost? 2006-04-19
From Charlie:
Selling a piece of furniture for $670 and the profit is 35%. What is the original cost?
Answered by Stephen La Rocque.
Proving a summation formula by induction 2006-04-19
From Sharon:
Prove by induction that the sum of all values 2^i from i=1 to n equals 2^(n+1) - 2 for n > 1.
Answered by Stephen La Rocque.
How do you round off 89,100? 2006-04-19
From Jeannie:
My home work says to round each number to the place of the underlined digit. It has 89,100. The underlined number is 9. What do I do?
Answered by Stephen La Rocque.
What is the cost price? 2006-04-14
From Purushotham:
If the selling price is 120 and the profit is 20%, what is the cost price?
Answered by Stephen La Rocque.
A proof by induction 2006-04-09
From Sharon:
prove by induction: For every n>1, show that
2 + 7 + 12 + ...+ (5n-3) = n(5n-1)/2

Answered by Penny Nom.
Given three angles and a side 2006-04-09
From Jon:
How do you figure out the length of all sides of a scalene triangle if given the measure of all angles, and one side?
Answered by Stephen La Rocque.
An octagon shaped soffit 2006-04-08
From Ken:
I am trying to make an octagon shaped soffit in my ceiling and need help figuring how to divide the circle on the ceiling into an octagon- The circle is 7 foot in diameter.
Answered by Penny Nom.
The average of monthly averages 2006-04-06
From Ray:
I have the averages of some data in percentages. the data is rather large, how can i find the average of these averages without going back to the original data? Is there a Factor or Constant that i could multiply or devide or add and get the average of these averages without resorting to the raw data.
Answered by Steve La Rocque and Harley Weston.
A table of values 2006-03-28
From Cindy:
Create a table of values for each of the following functions: y=5x, y=52, and y=5x. I dont know to do this.
Answered by Stephen La Rocque.
given that p is a prime and p|a^n, prove that p^n|a^n 2006-03-24
From Janna:
given that p is a prime and p|an, prove that pn|an
Answered by Stephen La Rocque.
How far from Earth is Neptune? 2006-03-21
From Scott:
A laser beam travelling at the speed of light bounces off the planet Neptune and returns to Earth in 29 960.72 s. The speed of light is 299 792.5 km/s. How far from Earth is Neptune?
Answered by Penny.
BEDMAS 2006-03-21
From Andie:
It's been a long time since I've used bedmas. I'm still confused as to what the answer to this question would be.
30 - (2 x 9) + (15/3)=

Answered by Penny Nom.
Three towns are located at the vertices of an equilateral triangle 2006-03-20
From A student:
three towns are located at the vertices of an equilateral triangle. The towns are 8, 5, and 3 miles, respectively, from a store. How far apart are the towns?
Answered by Chris Fisher.
The day of the week 2006-03-19
From Neil:

A friend of mine has an excellent memory for dates and events that occurred on that date, but his party piece is the ability to tell you the day of the week that fell of any given date. This talent is all the more extraordinary because the answer comes back in less than 2 seconds (often under 1 second). By his own admission he is no mathematician.

Now his memory and knowledge are without question, but I challenged him that because of its diversity this party piece could not be based upon memory, but on mathematics. I believe that with the correct mathematical approach and the use of a common algorithm, anyone with a basic mathematical mind can do this in their head.


Answered by Chris Fisher.
Gross profit as a percentage 2006-03-18
From Mark:
How do I calculate the gross profit by percentage on and a product I purchased for $23.80 marked up 45% sold for $34.51 = profit of $10.71 GP by percentage????????
Answered by Penny Nom.
The square footage of an area in my backyard 2006-03-17
From Kim:
I need to find out how to calculate the square footage of an area in my backyard that is in the shape of a "slice of pie". There are two sides that are straight lines that come together at the top to form a point, and then at the bottom is a curved line that joins the two other lines together. I need to figure out how to calculate the square footage that is inside the area.
Answered by Penny Nom.
9, 8, 7 and 2 in the game of 24 2006-03-17
From Alana:
I just began student teaching in a 5th grade math class and found that the students play 24 in class, a game that I used to play in elementary school. The students choose a card which shows 4 numbers and then have to use any combination of addition, subtraction, multiplication, and division to reach 24. There was one card that we found that seems to have all of us stumped. The four numbers are 9, 8, 7 and 2. Can you figure out how to make 24 with those? Thanks so much for your help!
Answered by Claude Tardif.
A proof by contraposition 2006-03-16
From Eban:

1)by mathematical induction prove that 12 + 32 + 52 + ...... + (2k-1)2 = (1/3)k(2k-1)(2k+1) for all positive integers k.

2)show that the contrapositive of the following statement is true. if 1 + M7 is even, then M is odd.


Answered by Stephen La Rocque.
Profit 2006-03-11
From Wendy:
If I have a product I have bought for 33.58 and I mark it up by 30% sell at 43.65 how do I work out the gross profit.
Answered by Penny Nom.
Factor 2006-03-08
From Brad:
Factor:
x3 + 64m3 and 125p3 - q6

Answered by Penny Nom.
The meaning of numbers 2006-03-03
From John:
I'm having a philosophical debate on the meaning of numbers, equations, or the world of math in general. Would it be possible if you could help me by giving me a resource that talks about it or if you have your own opinion I would be most grateful.
Answered by Harley weston.
Profit 2006-02-25
From Vernessa:
ABC company wants to sell enough products to earn a profit of $40,000, if unit sales are $10 and variable costs $8 , fixed cost $80,000 how many units must be sold to earn a profit of $40,000.
Answered by Penny Nom.
Victor's profit 2006-02-13
From Millie:
victor is trying to make money buying and selling cars. he saved up and buys a car for $2500 and then sells it for $4327.he then buys another car using 2/3 of his profit and sells it for twice what he paid for it. how much money did victor make during these transactions?
Answered by Steve La Rocque.
Proof by induction 2006-02-10
From Victoria:

how do i prove by induction on n that
n
Σ 1/i(i+1) = n/(n+1)
i=1

for all positive integers n


Answered by Penny Nom.
Prove that p^n >= (p!)/(p-n)! 2006-02-02
From Rhydian:

PROVE:

pn >= (p!)/(p-n)!


Answered by Penny Nom.
More on the game of 24 2006-02-02
From Cathy:
10,16,9,4 this numbers have to equal up to 24. we can't figure it out.
Answered by Claude Tardif.
BEDMAS 2006-01-31
From Janielle:
What Do All the Letters In BEDMAS Stand for?
Answered by Penny.
How do you find the angles in a triangle? 2006-01-27
From Keith:
How do you find the angles in a triangle if you know the lengths of the sides?
Answered by Chris Fisher and Penny Nom.
Our client wants to make 25% from their sale... 2006-01-24
From Noelle:
I need to figure out how to figure pricing for our clients. If our client wants to make 25% from their sale and they purchase it from me for $35.95, how do I figure what their cost is?
Answered by Penny Nom.
One boundary of a pond is parabolic in shape. 2006-01-20
From Glenn:
One boundary of a pond is parabolic in shape. The boundary passes through the points A(-20,45), B(40,40) and E(30,35). The equation of the parabola is of the form y=ax2+bx+c. Find the equation of the parabola and the coordinates of the vertex of the parabola. Any assistance you could provide would be greatly appreciated.
Answered by Penny Nom.
3x^4 - 81 2006-01-02
From Julio:
How can I factor the following?:

3x4 - 81

Answered by Penny Nom.
Simultaneous Equations 2005-12-21
From Matt:
I have these two equations,
336 = 60a + 10b
and
432 = 84a + 6b
Am I right in saying both a and b are 4.8?

Answered by Penny Nom.
The game of 24 with 7, 5, 5 and 4 2005-12-18
From Nicole:
i was playing the 24 game the other day. you get 4 digits and you have to turn them into 24. You can use addition, subtraction, division, and multiplication. You have to use all the numbers but you can only use each number once. an example is 2 4 4 1 the answer is (2x4)(4-1)=24. I got stuck on one. The numbers were 7 5 5 4 .
Answered by Penny Nom.
The sum of the angels in a triangle 2005-11-25
From Rachel:
how do you prove, without knowing any of the measurements or degrees, that the three angles of a triangle equal 180? what are the steps for proving that?
Answered by Penny Nom.
BEDMAS 2005-11-24
From Judy:
My name is Judy and I am a grade 6 teacher.
We have just started our lessons on the order of operations and my students have asked me why we have BEDMAS and what is the logic to it.

Is there a reason that that we do math in this particular order?

Who invented this rule or how was it decided on and when?

Answered by Walter Whiteley.
An isosceles triangle 2005-11-14
From Chris:
PX and QY are attitudes of acute triangle PQR, and Z is the midpoint of PQ. Can you write a proof that triangle XYZ is isosceles?
Answered by Chri Fisher.
fog 2005-11-12
From Janice:
I am having problem with the (fog) function

(fog) (x). Given f(x)= 2x2 ; g(x)= 3-4x

Answered by Penny Nom.
Coefficient of variation 2005-10-19
From Jan:
I am currently teaching the coefficient of variation and am wondering if there are some guidelines as to the interpretation of this statistic. I understand that it measures the variation in a variable relative to the mean - but what is the cut off for "too much" variation expressed in this way???
Answered by Andrei Volodin and Penny Nom.
The coefficient of thermal expansion for steel 2005-10-14
From Jim:

Is the following statement true?

The coefficient of thermal expansion for steel is 0.00000645in/in/deg. Doesn't sound like much but when you run out the numbers it comes to .405504 ft/mile/deg. Still doesn't sound like much, only about 5". Then multiply by 40 degrees and you get a piece of rail that has grown by 16.22 feet in that one mile. It's not at all unusual for the rail temp to go from say, 40 deg to 80 deg on a spring or fall day. Remember that on a sunny day, the rail temp can be significantly higher than the air temp as well."

I ran the math and came up with an answer closer to 16 inches, instead of 16 feet. Which is closer to being correct?


Answered by Penny Nom.
The game of 24 2005-10-10
From Kim and Chris:
My 4th grade son came home with a math game called 24 game. In it each card has 4 numbers on it. Each number can only be used once, in any order, using multiplication, subtraction, division, or addition. We are stuck on one and wondered if you could help. The four numbers are: 9, 9, 9, 9. And they must equal 24. Any clues?
Answered by Claude Tardif.
Prove that a rhombus' diagonals are perpendicular 2005-10-02
From Tania:
How do you prove that a rhombus' diagonals are perpendicular using the 2 column proof method?
Answered by Walter Whiteley.
A system of equations 2005-09-02
From Gina:
How do you interpret the solution of a system of equations by the corresponding graph? Demonstrate your answer by the use of an example.
Answered by Penny Nom.
BEDMAS 2005-09-01
From A student:
I am a student and am wondering about the answer to this question.

56/2(31)

is the answer 7?


Answered by Harley Weston.
Proof by induction? 2005-08-10
From Peter:

I am a lecturer and am having a problem with the following Proof by
Induction.

If

(N x N x N x N) + (4 x N x N x N) + (3 x N x N) + (N) = -4000

Prove that N is even!


Answered by Chris Fisher and Penny Nom.
Profit of more than 100% of cost 2005-06-22
From sam:
I know I can work out what percentage profit I am making by : profit/cost x 100, and I can work out up to 99% profit (the way I currently calculate it is profit x 1.99) But how do I work out any amount over 100%, say for instance I have something that costs me $0.52 that I wish to sell for 250% profit?
Answered by Penny Nom.
20% profit 2005-06-12
From Shawn:
If I know that I will be paid, say, $150.00, for labor, and I want to make 20% profit on what I pay for the labor, what is the calculation and factoring process?

I see it like this

(x*.2)+x = 150

but I don't know how to factor it out from there.

Answered by Penny Nom.
The sum of the digits of 2^100 2005-06-11
From Richard:
The sum of the digits was calculated for the number 2100, then the sum of the digits was calculated for the resulting number and so on, until a single digit is left.
Answered by Penny Nom.
y = x+2 and y = 2x - 1 2005-05-03
From Kimberley:
Question: y=x+2 y=2x-1
Answered by Penny Nom.
40% profit 2005-04-19
From Carol:
Could you please tell me the formula to calculate for example a 25% profit on cost. I.e a sandwich cost .75p for the cost of ingredients, I wish to make a 40% profit on that cost so what do I multiply and divide it by to arrive at the selling price in order to achieve a 40% profit?
Answered by Penny Nom.
A flaw in a problem 2005-04-15
From Bryce:

Question:

(x2-x2) = (x2-x2)
x(x-x) = (x+x)(x-x) [divide both sides by (x-x)]
x = x + x
x = 2x [divide both sides by x]
2 = x/x = 1

Where is the flaw in this problem?


Answered by Paul Betts.
Dimensions of a roof 2005-03-18
From A roofer?:
A right triangle (roof of a house) has a base of 7 feet and a 22 degree angle. What is the height of the roof and what is the hypothenus of the triangle.
Answered by Penny Nom.
The game of 24 2005-01-10
From Manny:
HI - my daughter and I are being driven crazy by the following "24" question 7,7,3,3. She's good at the game (well, better than I am) and her teacher gave her this one. Any insight would be appreciated!
Answered by Claude Tardif.
Profit 2005-01-10
From Abraham:
The profit a coat manufacturer makes each day is modeled by the equation P(x)=-x2+120x-2000, where P is the profit and x is the price for each coat sold.For what values of x does the company make a profit?
I don 't understand this problem(how to do it) and hope you can help me.

Answered by Penny Nom.
An isosceles triangle 2005-01-03
From Abraham:
The question is,"Triangle ABC is not isosceles.Prove that if altitude BD were drawn, it would not bisect AC."My question is If an altitude is drawn wouldn\'t that mean automatically its isosceles because, In a triangle the sides opposite congruent angles(in this case the right angles)are congruent? What am I thinking wrong?
Answered by Harley Weston.
2^9833 days from today 2004-12-26
From Pauline:
Today is Monday, 1 October 2001. What day of the week will be 29833 days from today?
Answered by Penny Nom.
A geometric proof 2004-12-11
From Hanna:
Given: ABCD is a quadrilateral;
Prove: ABCD is a parallelogram

Answered by Penny Nom.
"a" cubed minus "b" cubed 2004-12-02
From Denise:
"a"cubed minus "b"cubed equal (a-b) times
("a"squared plus "ab" plus "b"squared)?

I know this is a formula, but why is it true?

Answered by Penny.
The substitution method 2004-11-28
From A student:
I Was working and i ran into this problem can you help me solve it using the substitution method?

-3x-2y=-10
x+5y=-27

Answered by Penny Nom.
Proof by induction 2004-11-20
From Vic:
Problem: Find the first 4 terms and the nth term of the infinite sequence defined recursively as follows:

a(1) = 3 and a(k+1) = 2a(k) for k -> 1.

Note: Quantities in brackets are subscripts
-> means 'equal to or greater than'.

Using the recursive formula, the first 4 terms are; a(1) = 3, a(2) = 6, a(3) = 12, a(4) = 24

The nth term a(n) = 2n-1 x 3 (equation 1)

Equation 1 must be proven using mathematical induction. This is where I am having a problem.

Answered by Penny Nom.
Order of operations 2004-11-10
From Andrew:
I'm trying to solve this question, and I can't seem to remember the rules back from my high school days.

(40 x 8 ) / 2 + 55 - 15 =

Can you help me with the answer?

Answered by Harley Weston.
Bedmas 2004-11-09
From Fariha:
i have a single line question and am not sure of the method it would be appreciated if u could send me a method and a solution Q : 1 + 9 / 2 + 5 * 1000

would this be solved by BEDMAS? if not, why?

Answered by Penny Nom.
Solving triangles 2004-10-30
From Allen:
Solve the following triangles.

Given

1. B = 20 Degrees, a = 25, b = 16
2. A = 35 Degrees, b = 2, c = 3
3. A = 32 Degrees, C = 44, c = 20

Answered by Harley Weston.
Bedmas 2004-10-30
From Gina:
I am just curious whether Bedmas would be used in the following question as it is not in the typical Bedmas format.

Multiply 12 x 24
Add 26
Divide by 2
Subtract 7


Would we go about doing it in the sequence it is given or in Bedmas ?

Answered by Penny Nom.
Maximize income 2004-10-24
From Connie:
A company that sells x units of a product generates an income (I, in dollars) which is a function of x. The income generated is described by the equation

I = (-1/2)x^2 + 100x.

Discuss how to determine the number of units that must be sold so that the company can maximize its income. What is the maximum income?

Answered by Penny Nom.
Solve for a and t 2004-10-23
From Justin:
How do I solve for "a" and "t" in the equations:

1000t= -4000 + 2000t + (1/2)at^2
1000=2000 + at

Answered by Penny Nom.
A rate of change problem 2004-10-15
From Frank:
Find the rate of change of the distance between the origin and a moving point on the graph of y = x(squared) + 1 if dx/dt = 2 centimeters per second.
Answered by Penny Nom.
Accelerating to the speed of light 2004-09-30
From Lars:
How long time would it take to accelerate up to the speed of light with an acceleration speed similar to 0-100 seconds in 5 seconds?
Answered by Penny Nom.
A theorem involving a trapezoid 2004-09-29
From Abraham:
Given:Trapezoid ROSE with diagonals RS and EO intersecting at point M
Prove:Diagonals RS and EO do not bisect each other.

Answered by Harley Weston.
The volume of a pile of mulch 2004-09-22
From Sam:
Is there a formula to determine the cubic feet of something in a pile. IE- I need to determine the cubic feet of a pile of mulch. The pile comes to a peak, so the length and width decrease as the pile increases.
Answered by Penny Nom.
The method of substitution 2004-09-20
From Kayla:
y=x-1
y=2x-3

Answered by Harley Weston.
The game of 24 2004-09-16
From Angelica:
how can you make 24 out of 7,7,6,5?
Answered by Claude Tardif.
The game of 24 2004-09-14
From Alan:
I was playing that math 24 game and i cant get 21,14,2,2 to equal 24
Answered by Claude Tardif.
The 24 game 2004-09-09
From Erik:
Ive spent around 40 hours on this one problem. I need help. 3,3,7,7
Answered by Claide Tardif.
Order of operations 2004-09-04
From Leanne:
6 + 3 - 2 x 3 =
Answered by Claude Tardif.
50/5x - y 2004-08-22
From Rick:
Niece has a question that was marked wrong but we are unable to determine how the teacher calculated and arrived at the answer? The problem was as follows:
50 / 5x - y =

x=5
y=1

Answered by Harley Weston and Leeanne Boehm.
an integer with three factors 2004-08-03
From A student:
What is the probability that a randomly chosen 3 digit number has exactly 3 factors
Answered by Penny Nom.
The law of sines 2004-08-01
From Joy:
How do you solve this? Do you solve this triangle using the law of sines of the law of cosines? (ASA)

A=120DEG. B=40DEG c=35 cm I keep getting different answers.

Answered by Penny Nom.
Rate of change problems 2004-08-01
From Jim:
I just want to check a couple average rate of change problems because i just guessed on how to do them. Can you tell me how to do them?

the question says if f(x) = sqrt(x + 3), find f( x + rx). I got sqrt( x + rx + 3)

the other two are : f(x)= 3x-1 (f(x) - f(1)) / (x-1) ... I GOT 3 &

f(x)= x^3 - x (f(x) - f(1)) / (x-1) ... I GOT x^2 + x

Answered by Penny Nom.
Factoring 2004-07-19
From A student:
Factor completely:
3x3 - 24y3
54x6 + 16y3
16xy - 4x - 4y - 1
0.09x2 - 0.16y2

Answered by Penny Nom.
A proof in geometry 2004-07-16
From An:
Im taking a geometry course for the summer , and we just started to learn about proofs for about one week. Today in class, we started to do this one proof but didnt finish it because class ended. the problem is as follows.
Answered by Penny Nom.
The Number of the Beast 2004-06-13
From A Heinlein reader:
Most people who have an interest in mathmatics are familiar with Robert Heinlein's novel "The Number of the Beast", where 666, or ((6 to the 6th power) to the 6th power) equals 1.0315 times 10 to the 28th power, which in the novel is the number of parallel universes in the cosmos. My question is what would the number of parallel universes be if you grouped 666 the other way: (6 to the power of (6 to 6th power)). I have tried this on my calculator, but it won"t register that high.
Answered by Penny Nom.
The game of 24 2004-06-04
From Samantha:
i have the game 24 (the one where you have to use multiplication, division, addition, and subtraction to get to the answer 24) I was playing when i came across the 4 numbers 20, 17, 7, 11 and i cant seem to get 24.
Answered by Claude Tardif.
Finding bearings 2004-05-24
From James:
This question is about finding bearings. A boat race starts from point A, goes North to Point B, a distance of 1000 meters. The course is triangular. The bearing from point B to point C is South 70degrees West. The distance from Point B to point C is 1500 meters. Find the course bearing from C to A.
Answered by Penny Nom.
Angles of elevation and depression 2004-05-18
From Anjum:
what is the difference between an angle of elevation and angle of depression?
Answered by Penny Nom.
Three dice 2004-05-10
From A student:
If one has 3, 6 sided dice what is the probability of the numbers that are rolled to total 4 through 10 inclusively?

Subsequent to this, what is the probability to do this consecutively...say 3 times?

Answered by Peny Nom.
3x squared - 27 / x + 3 2004-05-04
From Stef:
3x squared - 27 / x + 3
Answered by Penny Nom.
Factoring 2004-04-27
From Bipin:
FACTORISE:

a to the power of 6 + b to the power of 6

Answered by Penny Nom.
The problem of Apollonius 2004-04-25
From Mitja:
There are given 2 circles lying one out of another and one point out of both circles. How to construct a circle passing through a given point and internally tangent to one and externally tangent to the other cirlce?
Answered by Chris Fisher.
Some factoring problems 2004-04-15
From KJ:

Factor these:
x3+125 -----> (x+5)3
8x3-27 -----> (?)
x2+36 -----> (x+6)2
x4-5x2+4 --> (?)


Answered by Penny Nom.
x^2/3 - 7 x^1/3 + 12 = 0 2004-04-05
From Jackie:
I am having trouble solving this question for x:

1.) x^2/3 - 7 x^1/3 + 12 = 0

Answered by Penny Nom.
(3x50)+20/5=? 2004-04-03
From A student:
what is the answer to:

(3x50)+20/5=?

Answered by Penny Nom.
n! > n^2 2004-03-30
From Jose:
How can you prove by mathematical induction that:

n! > n2.

Answered by Penny Nom.
BEDMAS 2004-03-20
From Brad:
I am in grade 7. My teacher tells me brackets always first, well i know that but, 5 (-4) x 2

Does the (-4) count as a bracket or is it just telling you not to minus 4 from 5 but to multiply 5 x -4 ? Am i correct?

Answered by Penny Nom.
Sum of the angles in a pentagon 2004-03-11
From Ashish:
What is the sum of all the measures of the angles of a Pentagon
Answered by Penny Nom.
Proof by induction 2004-03-02
From Chris:
I need some help of how to solve the problem

"use the principle of mathematical induction to prove that the following are true for all positive integers"

cos(n x pi + X) = (-1)^n cosX

any help would be appreciated

Answered by Penny Nom.
Profit, markup and discount 2004-02-22
From Kick:
Using the calculation on your website, if I sell something for $885 and my cost is $296, I make $589 profit. How do I calculate the profit percent I made on that transaction?

Profit as a percentage of what it cost me?
Profit as a percentage of what I sold it for?

Answered by Penny Nom.
The substitution method 2004-02-14
From Keisha:
I need to solve this problem by using the substitution method.
3x+y=1
x=2y+5
Please help me understand the method to solve.

Answered by Penny Nom.
Profit 2004-02-12
From Mohamed:
if i bought some thing with 100$ and the saller told me that his profit was 30% how can i know the actual price of the good
Answered by Penny Nom.
Order of operations 2004-01-28
From John:

I am trying to find out any information concerning the development of the order of operations. for example, when (why/how) did it become the case that 2 + 3 x 5 =17, rather than 25.

any insight is appreciated. Thank you.


Answered by Penny Nom.
BEDMAS 2004-01-21
From Jessica and her mom:

my mom and I were wondering 2 things.

1. what is the reason for having bedmas.
2. 5+5-4+[6x3-(6+1-3x2) -5+9]


Answered by Penny Nom.
The sides of a circle 2004-01-07
From Helena:
My name is Helena and I am 10 years old. On a resent math exam I was asked the question" How many sides does a circle have?" and I wrote down none. The teacher said the answer was one side.
Answered by Chris Fisher.
How far can you see? 2003-12-15
From Judy:

How far apart, assuming no obstacles, can two people stand and still see each other?

i know this deals with the curvature of earth, but i can't figure out the formulas involved.


Answered by Chris Fisher.
Room and board fees 2003-11-29
From Jennifer:
A certain university wishes to determine room and board fees for the next academic year. For the current year, the fee is $3600 per student and 1800 students are living in the residence halls. Past data suggests that for every $300 increase in the semester fee, 200 fewer students will choose to live in the dorms. There are also costs to the university associated with the residence halls. The fixed costs total $2,000,000 per semester. the variable costs are currently $1000 per student but will fall $100 per student for each decrease of 100 students. Your task is to help the university determine the optimal fee. Assume linear relationships between the number of students and the fee and between the number of students and the cost per student
Answered by Penny Nom.
Mr.Carter is very cautious 2003-11-26
From Bob:
Mr.Carter is very cautious. He decides to invest in only three stocks: one low stock, one high stock, one medium stock. Given that the expected annual yields are 6% for low stock, 7% for medium stock, and 8% for high stock, he wants his investment in medium stock to be half of his total investment in low and high stock. How much should I invest in each type fo stocks to expect a total annual return fo $650 form my investments?
Answered by Harley Weston.
Markup, profit and margin 2003-11-24
From Ali:
I have question regarding Margins, Markups and Profit Purchase price = $ 10.00
Margin or Markup ?? = 40 %
Therefore Selling Price = 10/0.6 = $ 16.67
Profit ? = (16.67-10)/10 *100 = 66.67%
or is the profit = (16.67-10)/16.67*100 = 40 %
Can you please clarify What is Margin, Markup and Profit

Answered by Penny Nom.
Difference of squares 2003-11-24
From Susie:

Factor assuming that n is a positive #

Problem: (I will give it to you in words beacuse I don't know how to do exponents on the computer.) Forty-five r to the 2n power minus five s to the 4n power. I was hoping you could walk me through it not just give me the answer.


Answered by Penny Nom.
Laws of sines and cosines 2003-11-23
From A parent:
On the one side of a stream lines PA= 586.3 feet, PB = 751.6 feet are measures, angle APB being 167 degrees and 36 min. Q is a point on the opposite side of the stream. Angle PAQ=63 degress and 18 min and PBQ=49 degrees and 24 min. Find PQ.
Answered by Penny Nom.
Systems of equations 2003-11-19
From Scott:

I hope that u can help me....I am a college student taking a class in Pre Calculus.....I have homework due this Friday and it counts a BIG Percentage on my FINAL grade.....I am getting mixed up and can not figure out a few problems.....Please help me.....

Method Of Subsitution

Problem 1. y- 8x = -5
x(squared) + y(squared) = 25

Problem 2. y = x(squared) - 2x - 6
Y = x(squared) - 4

Answered by Penny Nom.

Adding fractions 2003-11-16
From Ken:
My name is Ken and I am taking my GED course for my High School and have not been in a class for 35 years. I am doing this for re-training. I am at the part about fractions. Here is an example that I am having trouble with.

1 3/7 + 4 2/3 + 11/21

They have no common denominators. Could you PLEASE help me. If you could send me a step by step explanation it would be greatly appreciated.

Answered by Penny Nom.
Divisibility by 7 2003-11-14
From A student:
how do you test a number to see if it is divisible by 7 or not?
Answered by Penny Nom.
What is a number that has exactly 13 factors? 2003-11-09
From Mary:
What is a number that has exactly 13 factors?
Answered by Penny Nom.
Which one has the most factors? 2003-10-31
From Kristi:
Of all the whole numbers less than or equal to 5000, which one has the most factors?
Answered by Claude Tardif.
Standard Deviation 2003-10-07
From Rebecca:

I have a task to complete, which is to calculate the mean and standard deviation of something. I have done this but am then asked to write a short explanation of my findings.

I know what the mean is about, and I thought I knew what the standard deviation meant too - shows the variation from the mean. However, on a task I completed earlier the feedback I got said 'you need to tell us that it is talking about the middle 66% of the data' - that has thrown me, I don't understand that. Can anyone help me get my head round this???


Answered by Penny Nom.
The Sieve of Eratosthenes 2003-09-18
From Lynn:
My daughter has been asked to find all the prime numbers by using the Sieve of Eratosthenes. I have no understanding what this means.
Answered by Penny Nom and Claude Tardif.
Substitution method 2003-09-14
From John:

Hello, my name is john and while I was finishing up my work I stumbled into these problems. a/4-b=-1
a+b=11
and
6a-b=-5
4a-3b=-8

Thanks a lot. I really need some help :)
John


Answered by Penny Nom.
Order of operations 2003-09-07
From Brian:

It has to do with the 4 rules of operations, Parentheses first before operations outside/evaluate all exponential expressions/all multiplication and divisions/then all additions and subtractions.

Who made these rules and when did they make them,,, I know its somewhat of an unorthadox question but I must know. I would really appreciate it.


Answered by Penny Nom.
The game of 24 2003-09-03
From A student:
We have a game where four numbers must equal 24 using addition, subtraction, multiplication and/or division.Ý I am stuck on the following four numbers - 12, 24, 9 and 17.
Answered by Penny Nom.
Profit margin 2003-09-03
From Chet:

My salesman calculates a customers profit margin by dividing what the customer paid by what he sells for, then subtracting that answer from 1. For example a customer sells a product for $15. He bought it for $10. 15/10=.66 Subtract that from 1 and get a profit % of 33%

Here's my question: If I want to figure what a reseller will sell a product for if he wants 40% margin(by the technique above), what formula do I use? Let's say I want to sell a product for 8.50, and I want the reseller to make 40%


Answered by Penny Nom.
A pyramid with its top cut off 2003-07-21
From David:
What is the name given to a 3D shape that looks like a pyramid with its top cut off?
Answered by Penny Nom.
4-3(m+1)=(-38) 2003-06-25
From Jamie:

I have a problem, like most of your mailers, I do remember BEDMAS but maybe I'm missing the finer points!

It's been a while, he goes

4-3(m+1)=(-38)

Answered by Penny Nom.
Two trig problems 2003-06-10
From Bett:

I have this ongoing trouble with trig and solving triangles with laws of cosines and sines!! For example if it asks to solve triangle FGH, given angle G=102.7 , side f=14.2, and h=18.6. Now do I use law of cosines because I don't have the measure of an angle and length of the opposite side??I don't know where to go from here,I am totally confused!!!

I also have a problem with this word problem I have been doing. It asks: An airplane flies 847.5 km at a bearing of 237.3 degrees. How far south and west fo its original position is it? Huh? Please help!


Answered by Penny Nom.
BEDMAS 2003-05-31
From Kristie:
(3x50)+20/5=?

I know bedmas but i forget how to do it.

Answered by Penny Nom.
x-6square root of x +8=0 2003-05-10
From Elizabeth:
x-6square root of x +8=0
Answered by Penny Nom.
2/sqrt(2) 2003-05-07
From Mike:
I was looking at an example of csc 45 deg where

csc = hyp / opp = 1 / (sqrt(2) / 2) = 2 / sqrt(2) = sqrt(2)

I just don't get this. If sqrt(2) = 1.4142145..... How can 2 / sqrt(2) = sqrt(2) as the example shows?


Answered by Penny Nom.
Rules of exponents 2003-05-05
From Carl:
Hi, I am a student who would like to recall how to multiply exponents. Here is such an equation:
6.02569 X 1025 X 5.254 =?

Also, adding exponents. Don't I just add subtract the exponents separately?
Such as 523 +15-12 =??

Answered by Penny Nom.
The volume of air flowing in windpipes 2003-05-02
From James:
The volume of air flowing in windpipes is given by V=kpR4, where k is a constant, p is the pressure difference at each end, R is the radius. The radius will decrease with increased pressure, according to the formula: Ro - R = cp, where Ro is the windpipe radius when p=0 & c is a positive constant. R is restricted such that:
0 < 0.5*Ro < R < Ro,
find the factor by which the radius of the windpipe contracts to give maximum flow?

Answered by Penny Nom.
positive multiples of 10 that are the sum of four consecutive integers 2003-05-01
From Taurus:
How many positive multiples of 10 that are less than 1000 are the sum of four consecutive integers?
Answered by Penny Nom.
Profit for a dance studio 2003-04-28
From Craig:
A dance studio charges $80 per student for a series of 2 hour lessons. The studio's costs are $30 per hour for the instructor, $15 per lesson for the room rental, and $3 per student for miscellaneous expenses. If x is the number of students enrolled in the class, express the studio's profit P(x) in terms of x. Find the profit if 10 students are enrolled in the class.
Answered by Penny Nom.
Can twice a square be a square? 2003-03-25
From Mike:
The other day it occurred to some students that they could think of no square number which is an integer, which can be divided into two equal square numbers which are intergers, Or put another way, no squared integer when doubled can equal another square integer. For example 5 squared plus 5 squared is 50, but 50 is not a square number.
Answered by Walter Whiteley and Claude Tardif.
Three proffs of a trig identity 2003-03-18
From Nadene:
Prove the identity. cos [x + (y-pi/2)] = sin (x+y)

A hint was also provided which is: "Apply cos (alpha + beta) first then within that apply cose (alpha-beta)"

Answered by Penny Nom.
Three digit number 2003-03-10
From Grace:
What three digit positive integer is exactly 32 times the sum of its digits?
Answered by Claude Tardif.
Order of operations 2003-02-16
From A student:
How is the order of operation used in everyday life other than in a math class or at school? Also .... Can you give me a list of all the mathematician that are still living that uses the order of operation?
Answered by Claude Tardif.
Order of operations 2003-02-15
From Debbie:
Question: 20-(9+4)x7=?
Possible answer -71 or 49?

Answered by Penny Nom.
BEDMAS 2003-02-09
From Stefanie:
I do remember the rules of BEDMAS, but for some reason this question puzzles me.

6 X 9 - 3 + 44

I started with the Multiplication

54 - 3 + 44

But then I got stuck, do I proceed with adding the 44 and subtracting the 3
or figure out what -3 + 44 is, but then how would that work with 54?

Answered by Penny Nom.
Dorothy and the wizard 2003-01-24
From Ken:

dorothy was going to see the wizard of oz...she stopped at the bakery and bought a box of cookies.

she met the scarecrow and gave him half the cookies. she ate half of the remaining cookies and threw one half cookie away.

she met the tin man and gave him half the cookies. she ate half the remaining cookies and threw one half cookie away.

she met the lion. she gave him half of her cookies. she ate half the remaining cookies and threw the LAST half cookie away.

how many cookies did dorothy leave the bakery with?


Answered by Penny Nom.
The intersection of conics 2002-12-19
From Glenda:
We are studying systems of equations where two conic sections are the two equations that we are solving simultaneously. We were studying the number of solutions that are possible if you have an ellipse and a parabola. We all agree that there can be none, one, two, three or four solutions. The question that the students had for me was whether or not a portion of an ellipse and a parabola can overlap and thereby allow an infinite number of solutions. What should I tell them?
Answered by Chris Fisher and Harley Weston.
Factoring 2002-12-11
From Larry:
Question:

how do u factor trinonmials

EX: X 3 + Y 3

X 3 - 8Y 3

8X 2 - 72

64A 3 - 125B 6

Answered by Penny Nom.
A two stage rocket 2002-11-26
From Hoda:
a two stage rocket accelerates in free space by ejecting fuel at a constant relative speed , v(ex). the full fuel load makes up 80% of the initial mass of the entire two stage rocket . the rocket accelerates from rest until at the end of the first stage when 75% of its fuel has been burnt. find an expression for the speed of the rocket at the end of the first stage in terms of v(ex).
Answered by Claude Tardif.
Factor completely 2002-11-21
From Shelley:
these two questions are to be factored completely but i have no idea how to factor them
  1. (x-4y) 2 - 3(x-4y) - 4
  2. x 6 + y 6

Answered by Penny Nom.
The 24 game 2002-11-16
From Michael:
Hello, my name is Michael and I am in elementary school .. We have a game we play in math class called 24 game. In it each card has 4 numbers on it. Each number can only be used once, in any order, using multiplication, subtraction, division, or addition. We are stuck on one and wondered if you could help. The four numbers are 10-9-16-4. And they must equal 24. Any clues?
Answered by Penny Nom.
The percentage grade of that hill 2002-11-05
From Cathy:
If there is an 80ft climb over a kilometer(about 3280ft) what is the percentage grade of that hill?
Answered by Penny Nom.
The repetition of doing many straight forward problems 2002-10-16
From Dan:
I am the father of a very bright 9 year old girl. She is very interested in math and she grasps concepts very quickly. I am concerned that she may become bored with the repetition of doing many straight forward problems. Her current math teacher has her working in a challenging math book and she is doing 3+ digit multiplication. She misses 2 or 3 out of ten of these problems due to simple addition errors. Should she continue to practice these problems until she can get them all correct? Or, should she move on?
Answered by Kathy Nolan and Claude Tardif.
Proof by induction 2002-09-26
From Pooh:
Use induction to show that

1 2 + 2 2 + .....+n 2 = (n 3)/3 + (n 2)/2 + n/6

Answered by Paul Betts.
Fractional exponents 2002-09-20
From Jill:
The problem is with fractional exponants:
10 1/3 mult. by 10,000 The 1/3 is an exponant of 10.

Answered by Penny Nom.
The number of decimal places in 1 over a power of 2 2002-09-12
From Allan:
Does anyone notice that the maximum number of decimal place of the number 2 dividing 1 and its increment (4, 8, 16...etc) is the same as the power of number 2? eg. 22=4, thus the max number of decimal of 1/4=0.25 which is 2 decimal place and 2 is the number of power of 2 take 64 as example: 26=64, and take 1/64=0.015625 which has 6 decimal place (and is the power 6)

Is there such a law in math? If yes, can you tell me what it is? Or is this my discovery?


Answered by Paul Betts.
Proof by induction 2002-08-31
From Tabius:
Use mathematical induction to prove that the following formulae are true for all positive integers:

a) 1 + 3 + 5+...+(2n - 1) = n 2

b) 2 n > n.


Answered by Penny Nom.
38-10+12divided by4multiplied by 16 2002-08-30
From Brenda:
my math question is as follows: 38-10+12divided by4multiplied by 16=?
Answered by Penny Nom.
Common Denominator 2002-08-26
From Slobodanka:
What is a Common Denominator?
Answered by Penny Nom.
Sole Mates Shoes 2002-08-19
From Robert:
Sole Mates Shoes has expenses of $9,592 per month. What must the store's total income be if it is to make a profit of 12%?
Answered by Penny Nom.
Volume of a sphere 2002-08-16
From Rahul:
How do i find the volume of a sphere without the knowledge of calculus?
Answered by Walter Whiteley.
Two equations 2002-07-26
From Derek:
1. 3x + 2y = 4
2. -7x + 2y = 24

finding x and y.


Answered by Penny Nom.
Some inequalities 2002-07-25
From A student:

1)-5 <= -3X+1 < 1

2)4X-2 < 6 OR X+2 > 9


Answered by Penny Nom.
Domain of a function 2002-07-20
From Andy:
I'm having difficulty in finding the domain of

1/(x2 -2x + 4)

one over x squared minus 2 x plus 4.


Answered by Penny Nom.
Order of operations 2002-07-18
From Danna:
I would like to know how to solve this type of problem; I already have the answer.

Problem: 2 [5 (4 + 6) - 2] = 96

Also, what do you call this type of problem? Thanks a lot.


Answered by Penny Nom.
Constructing the square root of 3 2002-06-07
From Allan:
I am a Math 7/8 teacher. I was wondering how you would show a student how to find the exact location of the square root of three on the number line using just a compass and a straight edge.
Answered by Penny Nom.
A spotlight shines on a wall 2002-05-25
From Barb:
A spotlight on the ground shines on a wall 12m away. If a man 2m tall walks from the spotlight toward the bldg at a speed of 1.6 m/s, how fast is his shadow on the bldg decreasing when he is 4m from the bldg?
Answered by Penny Nom.
How far apart are the transmitters? 2002-05-18
From Jeff:
A ship at sea is 70 miles from one transmitter and 130 miles from another. The measurement of the angle between the signals is 130 degrees. How far apart are the transmitters?
Answered by Penny Nom.
The law of cosines and obtuse angles 2002-05-09
From Bryant:
The question that I am pondering is that I need to derive the law of cosines for a case in which angle C is an obtuse angle.
Answered by Penny Nom.
8/13*26/27 2002-05-01
From Arias:
8/13*26/27=
Answered by Penny Nom.
An augmented matrix 2002-04-20
From A student:
Hi my math teacher asked us to solve an augmented matrix. I am in twelfth grade and need help. The book we are working on is college algebra. Here it is

a+2b+c=0
2a+5b+4c=-1
a-b-9c=-5


Answered by Penny Nom.
The square root of four 2002-04-13
From Frank:
In a recent quiz we where asked the square root of four. The quizmaster gave the answer as not only 2 but also -2. Being pedantic I queried this. Although -2 squared is 4, does it follow that the root of 4 is -2 Can you enlighten us?
Answered by Claude Tardif and Penny Nom.
Calculate profit 2002-04-13
From Allison:
If I sell something for $75; and my cost is $40, I make $35 profit. How do I calculate the profit percent I made on that transaction?
Answered by Penny Nom.
Composition of functions 2002-04-06
From Yvonne:
In our new text book, the following question occurs: State the domain and range of g(f(x))given that f(x) = -x2 - 4 and g(x) = sqrt(x)

The range of f(x), x<=-4, is the domain of g(x). BUT, there is no solution in the Real numbers for g(f(x))= sqrt(-x2 - 4).

In the solutions it says that this is not a function and therefore does not have a domain or range. Is it a relation? Is it anything?


Answered by Claude Tardif.
Pairs of equations 2002-04-04
From A student:

high school level
student is asking

y=4x x=-4y 
x+y=5 3x+2y=20   

y=x-1 3x-y=4 
x+y=3 2x-3y=-9   

x+5y=4 
3x+15y=-1 

. . . 

Answered by Penny Nom.
Some 5 card hands 2002-03-28
From A student:
From a standard deck of cards how many 5 card hands are possible consisting of a. exactly 4 hearts

b. two cards of one kind and three of another(like a full house).

Answered by Penny Nom.
A hamburger and a soft drink 2002-03-23
From A student:
A GROUP OF FRIENDS WENT OUT TO LUNCH. EACH BROUGHT A HAMBURGER AND A SOFT DRINK. TOGETHER A HAMBURGER AND A SOFT DRINK COST MORE THAN A DOLLAR. THE TOTAL COST FOR THE GROUP WAS $17.81. THERE WAS NO TAX OR TIP INCLUDED.

IF A HAMBUGER COSTS 2 CENT MORE THAN TWICE A SODA, FIND THE COST OF THE HAMBURGER.


Answered by Leeanne Boehm.
24x^4 + 3x 2002-03-18
From gary:
24x 4 + 3x
Answered by Penny Nom.
Testing a hypothesis 2002-03-14
From A student:
A large distriutor of cosmetics has kept his outstanding accounts receivable to a mean age of 18 days over the past year. This average is considered a standard by which to measure the efficiency of the credit and collections department. Management wishes to check if receivables in the current month is over standard and will do this at a significance level of 0.50. A random sample of 100 accounts yields an average of 20 days with a standard deviation of 9 days. what should management conclude?
Answered by Andrei Volodin.
The square root of i 2002-03-14
From Arlene:
what is the square root of i, if i=x+yi?

what is the square root of 1-i? i'm getting problems like these in which I do not understand.


Answered by Harley Weston.
The isosceles triangle of smallest area 2002-03-08
From Lettie:
can you find the isosceles triangle of smallest area that circumscribes a circle of radius of one?
Answered by Walter Whiteley.
The square root of 2 2002-03-05
From Roger:
Does two (2) have a square root or do the numbers just keep going? Are there any other numbers that behave like two when it comes to extracting the square root?
Answered by Penny Nom.
Proof by induction 2002-02-20
From Tamaswati:
How do I prove the assertion that "the determinant of an upper triangular matrix is the product of the diagonal entries" by mathematical induction? (Before I check this assertion for a few values of n how do I rephrase the assertion slightly so that n appears explicitly in the assertion?)
Answered by Penny Nom.
Rolling 5 sevens before rolling a six or an eight 2002-01-20
From Tony:
When rolling 2 dice, what is the probability of rolling 5 sevens before rolling a six or an eight?
Answered by Andrei Volodin and Penny Nom.
Making 24 2002-01-17
From Renee:
My 4th grade daughter and I need to find a simple math sentence using 5, 5, 3, & 7 to equal 24. You can add, subtract, divide or multiple.
Answered by Penny Nom.
What is a group of three numbers called in a large number? 2002-01-15
From Laurie:
What is a group of three numbers called in a large number?

My son has this question on his 4th grade worksheet. I've taught middle school math for 7 years and nver heard of it.


Answered by Paul Betts.
Adding vectors 2002-01-12
From Lena:
how do you add vectors together?
If you are given the length and angles of both vectors and are asked to add/subtract them, how do you do it? I know you are supposed to do the head to tail method, but whenever i try it i get the wrong answer. I need help setting it up.

example: A is 2.7cm, and 60 degrees, B is 1.6cm and 135 degrees, find the magnitude and amplitude


Answered by Penny Nom.
All of the Christmas trees are fifty-five percent off. 2002-01-08
From A teacher:
All of the Christmas trees are fifty-five percent off. Madison bought a tree and paid $103.50. Assuming no tax, what was the original price of the tree?
Answered by Penny Nom.
Catenary 2002-01-02
From Jason:
I am a high school math teacher. I was asked by a friend who is in architectural design for a method for determining the volume of what he called a Catenary.

The Catenary curve is modeled by the equation y=a cosh(x/a). I ran into a mess when I tried to compute the volume of the solid formed by revolving that curve around the y-axis.

Any help you can provide would be greatly appreciated.


Answered by Harley Weston.
Composition of functions 2001-12-16
From Paula:
  1. if f(x)= 3x-1 and g(x)= 1/2x + 3 find fog(2)

  2. find the values of x for which tanx=0

Answered by Penny Nom.
Piles of coins 2001-12-05
From A student:
Sharon has less than 20 coins. When she puts them in piles of 5, she has 1 left over. When she put them in piles of 3, she also has 1 left over. How many coins does Sharon have?
Answered by Penny Nom.
2=the square root of (2 + the square root of (2 + the square root of (2 +...))) 2001-11-05
From Cynthia:
justify algebreically, that:

2=the square root of 2 + the square root of 2 + the square root of 2 + the square root of 2 + the square root of 2 + and so on, .......


Answered by Penny Nom.
Squares of negative numbers 2001-11-03
From Susana:
I wanted to know if I can square a negative number..?
Answered by Leeanne Boehm.
Proof by induction 2001-10-16
From John:
Can you help me with any of these?
  1. For any natural number n > 1, prove that

    (4n) / (n + 1) < [(2n)!] / [(n!)2].

  2. For any natural number n > 1, prove that

    1/sqrt(1) + 1/sqrt(2) + 1/sqrt(3) + ... + 1/sqrt(n) > sqrt(n).

  3. For any natural number n and any x > 0, prove that

    xn + xn - 2 + xn - 4 + ... + x-n >= n + 1.

Answered by Penny Nom.
Some algebra 2001-10-15
From James:
I cannot figure these out I was wondering if you could help me? I have no one to answer my questions.
  1. (7x2 – 3yz)2 – (7x2 + 3yz)2

  2. Use Pascal’s triangle to expand (2x – y)4

  3. 8x3 y - x3 y4

  4. (m + 3n)2 – 144

  5. 12x4 y – 16x3 y2 – 60x2 y3

  6. p3 q2 – 9p3 + 27q2 – 243

Answered by Peny Nom.
Rules of exponents 2001-10-14
From Carissa:
how do you work this out? Investigate the relationship between a,b,c and d if 2a*2b=4c/4d?
Answered by Leeanne Boehm.
Proof by induction 2001-09-30
From Kyle:
I'm trying to learn induction and I need to see how this done please help with this problem...

20 + 21 + 22 +... + 2n = 2n+1 -1 is true whenever n is a positive integer.


Answered by Penny Nom.
Ciefficient of variation 2001-09-23
From Carmen:
I have a question from my OAC finite class. I've come across a problem with the coefficient of variation. I have taught my students that there are no units for coefficient of variation and it can be expressed as a percent. So, for example, a set of data with mean of 5 and standard deviation of 100 would have a CV of 5%. But what happens in this situation: the mean is 4meters and the standard deviation is 0.7mm. Is the CV 1.75% or 0.00175% or 0.0175%? I've had some students change leave the units as is, change them both to mm or change them both to meters...so which is it and why?
Answered by Penny Nom.
A 91-gon 2001-09-10
From Ashley:
Hi! My name is Ashley and I am an 8th grader and I couldn't find the name of a polygon with 91 sides. i was a question from my math teacher.
Answered by Penny Nom.
The best of 7 2001-09-08
From Michelle:
Professional basketball, hockey, and baseball championships are decided on the best 4 out of 7 games. The first team to win four games wins the championship. In how many possible ways could the team win the championship series after winning the first game?

 Example:  
Game   1    2    3    4    5     6     7         
Result W    L    L    W    W     W 

Answered by Leeanne Boehm.
Sharing a donut 2001-09-06
From Amanda:
You have invited 11 people over to your house one day, and your friends are hungry. You go into the kitchen and find out you only have 1 donut (with a hole in the middle). In order to feed 12 people (including you), you must cut the donut into 12 pieces with only using a straight knife and cutting 3 times. This is NOT a trick question.
Answered by Claude Tardif.
Fourier transform 2001-08-07
From Adbul:
  1. Sir, we have the Dirichlet's condition for the Fourier transform : " The function should be integral over the real line " But why we are we neglecting this for example when we take the Fourier transform of an impulse train?

  2. Suppose we want to travel from one corner of a square of side 'a' to the diagonally opposite corner. We can travel along the sides which gives a pah length of '2a'. We can also do it in steps as shown below:

      _ | |_PATH |   |_ |_____| 


    Suppose The step size =DELTA x Then the path length will be again '2a'. Now in the limit DELTA x -->0 again we get '2a' But when we take the limit we get the straight line diagonal whose length is 'SQRT(2)X a' Where did I go wrong?

Answered by Chris Fisher.
How far does the fly fly? 2001-08-07
From Harold:
 
6 MPH                            4 MPH 
Rachel  ----------------------     Eli                        
                  10 Miles apart  

The fly is on Rachels handlebars. The fly is scared so it flys back and forth at 20 MP H. How far has the fly flown when Rachel and Eli meet?

f

Answered by Penny Nom.
Linear regression 2001-08-01
From A student and a mother:
My mom is taking a correspondence course in Simple Linear Regression and Correlation Analysis and we've been arguing about the relationship between the correlation -rxy and standard estimate of error-Sy.x. I took statistics last year in high school and I remember something about the Sy.x being proportionate to the r. Are they inversely related, directly related, not related, or can only range from 0 to 1.00? Her book doesn't say very much and I believe they are inversely related. She says they're directly related.
Answered by Chris Fisher and Penny Nom.
e^pi > pi^e 2001-07-27
From Dusty:
What book(s) contain a proof that ePi > Pie? I think it might be in Problems in Analysis published by Springer-Verlag but I have not been able to check.
Answered by Chris Fisher.
Margin of error 2001-07-13
From Jim:
If a survey was completed by four different groups of people as indicated below what would the over-all margin of error be for this survey given the fact that the number of people in each group were different?
 
Group   # polled   Margin of error 
A.           779           +/- 4% 
B.           315           +/- 7% 
C.           323           +/- 6% 
D.           254           +/- 9% 

Answered by Andrei Volodin.
The speed of the boat 2001-07-12
From Sharon:
A motor boat is travelling in a southeasterly direction in water that is flowing from the south at 2km per hour. Show that the speed of the boat is (6 times the square root of 2) km per hour, given that it can travel at 10km per hour in still water.
Answered by Penny Nom.
Harmonic numbers 2001-05-23
From Leslie:
The harmonic numbers Hk, k = 1,2,3.....are defined by Hk = 1 + 1/2 + 1/3....1/k

I am trying to prove by mathematical induction:

H2n >= 1 + n/2 , whenever n is a nonnegative integer.

H8 = H23 >= 1 + 3/2

Can you help?


Answered by Harley Weston.
A pile of sand 2001-05-14
From Gul:
  1. Sand for use on icy roads is stored in a conical pile 14.2 m high and with a base diameter of 34.4 m
    • calculate the volume of the pile

    • if one sander can take 6.9 m of sand, how many sanders can be filled from the pile?

Answered by Penny Nom.
The angles in a triangle 2001-05-11
From Nikki:
Find the measure, to the nearest degree, of each angle of a triangle with sides of the given lengths.

26, 35, 40


Answered by Penny Nom.
Maximize profit 2001-05-09
From Brian:
The marginal cost for a certain product is given by MC = 6x+60 and the fixed costs are $100. The marginal revenue is given by MR = 180-2x. Find the level of production that will maximize profit and find the profit or loss at that level.
Answered by Harley Weston.
A sequence of even terms 2001-04-29
From A student:
A sequence c is defined recursively as follows:

c0 = 2
c1 = 4
c2 = 6

ck= 5ck-3 for all integers

Prove that cn is even for all integers.


Answered by Leeanne Boehm and Penny Nom.
A geometry proof 2001-04-18
From Melissa:
Extend the bisectors of angle A, angle B, and angle C of triangle ABC to meet the circumcircle at points X, Y, and Z respectively. Show that I is the orthocenter of triangle XYZ.
Answered by Chris Fisher.
A famous landmark 2001-03-23
From Corinne:
A family is traveling due west on a road that passes a famous landmark. At a given time the bearing to the landmark is N 62 degrees W, and after the family travels 5 miles farther the bearing is N 38 degrees W. What is the closest the family will come to the landmark while on the road?
Answered by Harley Weston.
Systems of equations 2001-03-16
From joy:
How do u solve problems using systems of equations?
~ finding x and y~

ex:

26 = 3x - 2y
42 = 4x + y


Answered by Penny Nom.
How can you prove a quadrilateral to be a parallelogram? 2001-03-16
From Joy:
How can you prove a quadrilateral to be a parallelogram?
Answered by Walter Whiteley.
Names of the polygons 2001-03-15
From Sami:
I was asked to find out what the names for different polygons were. The only two I cannot find are the names for a polygon with 30 sides and a polygon with 40 sides. I would really appreciate it if you could help me. Thank you very much.
Answered by Harley Weston.
The angle of elevation 2001-03-08
From Jeffrey:
At a Certain time, a vertical pole 3m tall cast a 4m shadow. What is the angle of elevation of the sun?
Answered by Harley Weston.
problem of the week 2001-03-06
From Peggy Allan:
My son has been challenged with "the problem of the week" and I am unable to assist him in finding a reasonable solution.

Problem 1,

Julio needs to draw a line segment 15cm long. He does not have a ruler. He does have some sheets of letter size paper 28 X 21.5 cm. Describe how Julio can use the paper to measur 15 cm.

Answered by Claude Tardif.
Powers 2001-03-04
From A student:
Hey, can you show me how you do ..

(2xy)to the 3rd power (x) to the 2nd power?


Answered by Penny Nom.
Difference of Squares 2001-02-22
From Bruno-Pierre:
I noticed the other day that if you substract two consecutive squared positive numbers, you end up with the same result as if you add up the two numbers.

Ex. 5 and 6 (2 consecutive positive numbers)
52 = 25
62 = 36
36 - 25 = 11 (Substraction of the squared numbers)
5 + 6 = 11 (Sum of the numbers)

A more algebric view:
a2 - b2 = a + b where a and b are consecutive positive positive numbers. (b = a + 1)

I wondered if this rule had a name, and who discovered it.


Answered by Penny Nom.
Mr. Moser's roof 2001-02-21
From Michelle:
Mr. Moser is planning to replace the roof of his home. He needs to order a pack of shingles. Each pack covers 100 sq. ft. of roof. Without a ladder, Mr. Moser can not climb to the roof to measure it. Instead, he measures his attic and finds it to be 40 ft. long, 24 ft. wide, and 5 ft. high at the peak of the roof which is in the center of the house. Although the roof is even with the side walls, he estimates the roof line continues 1.5 ft. beyond the front and back walls. How many full packs of shingles should Mr. Moser order to cover his roof?
Answered by Penny Nom.
Math game 24 2001-02-21
From Kristi:
We have a game we play in math class called 24 game. In it each card has 4 numbers on it. Each number can only be used once, in any order, using multiplication, subtraction, division, or addition. We are stuck on one and wondered if you could help. The four numbers are: 3, 3, 5, 7. And they must equal 24. Any clues?
Answered by Claude Tardif.
The law of cosines in the real world 2001-02-21
From Hope:
Do you have any examles and/or labs that show how the law of cosines is used in the real world?
Answered by Harley Weston.
Undecagon 2001-02-21
From Karen:
I need to know what kind of a polygon an undecagon is and how many sides it has. This is a problem in my math book.
Answered by Penny Nom and Michael Tsatsomeros.
A pole next to a building 2001-02-20
From Robert L. Haun:
A pole 100 feet tall is erected next to a building 10' X 10' X 10' . Object is to cut off the pole so the upper end remains touching the part cut off, while the cut-off portion touches the cutoff point, the edge of the building and the ground. Note assume the pole is touching the building and is "very thin" What is/are the cutoff points?
Answered by Penny Nom.
Law of cosines 2001-02-20
From Emily:
I missed a few days of class and I can't figure out how to solve Law of Cosines problems. I have the notes and formulas but I can't figure out how to do the math involved to answer the problems. I am also getting confused about how to use degrees and seconds in the Law of Sine and Cosine. I can't seem to get the correct answers and I don't really know how to solve them and enter them into my TI 86.
Answered by Harley Weston.
Difference of squares 2001-02-20
From Janna:
Hi! I was just wondering how you would factor x2 - 9y2.
Answered by Harley Weston.
Delivering firwood 2001-02-20
From Jackie:
Two college students earn extra money on weekends by delivering firwood in thier pickup truck. They have found that they can sell x cords per weekend at a price of p dollars per cord, where x=75-3/5 p. The students buy the firewood from a supplier who charges them C dollars for x cords to the equation C=500+15 x+1/5 x2.

a) find a function f such that P=f(p), where P dollars is the profit per weekend for the students if they charge p dollars per cord.

b)find the profit P dollars if p=$95

Answered by Penny Nom.
Buckets of golf balls 2001-02-08
From A student:
A bucket containing 40 golf balls weighs 135 ounces. The same bucket with 20 golf balls weighs 75 ounces. What is the weight of the bucket?
Answered by Claude Tardif.
1 + 1 = 1 2001-01-23
From Stephanie:
My friend has this as a bonus question the other day and I want to figure it out. I don't know how 1+1 in any form could equal 1. Please let me know how you come about geting that.
Answered by Claude Tardif.
64.5% profit 2001-01-12
From Ethel:
IF I PURCHASE AN ITEM FOR $2.00 AND I WANT TO RESELL IT WITH A MARGIN OF GROSS PROFIT OF 64.5% WHAT FORMULA DO I USE TO ARRIVE AT THAT RESELL FIGURE?
Answered by Penny Nom.
The laws of sines and cosines 2001-01-02
From Faydene:
Can the sine /cosine rule be applied to a right -angle triangle to find a particular solution or are these 'rules' applied only when the triangle is not right angled?
Answered by Penny Nom.
Factoring (u-v)3+v-u 2000-12-15
From A parent:
I am a middle school teacher and a parent. I am snowed in and trying to help my 9th grader get ready for 9 weeks exams. I have tried to factor this problem to no avail. (u-v)^3+v-u. I have the answer but I need to know how it is done.
Answered by Penny Nom.
An integration problem 2000-11-30
From A student:
If a>0 and the integral from b to 0 of 1/(1+x) equals 1/2 the integral from a to 0 of 1/(1+x), express "b" in terms of "a"
Answered by Claude tardif.
Vector Problem 2000-11-27
From Ben:
An aircraft can fly 260km/h in still air and the wind is blowing at 70km/h towards the West. In what direction should the aircraft head so that its actual velocity is on a bearing of 030 degrees?
Answered by Harley Weston.
Order of operations 2000-11-26
From Margaret Pratt:
My daughter has a math question and I am afraid I am of no help. Can you help? 2x5/2+1-5= She arrives at 8 as the answer and has been told this is incorrect. Any help you can give would be appreciated.
Answered by Penny Nom.
Pillows and Cushions 2000-09-27
From Fiona:

The following problem was given to grade eleven algebra students as a homework assignment. To manufacture cushions and pillows, a firm uses two machines A and B. The time required on each machine is shown. Machine A is available for one full shift of 9.6 hours. Machine B is available for parts of two shifts for a total of 10.5 hours each day.
Answered by Harley Weston.

A proof that 1=2 2000-09-19
From sporky:
Why does the proof for 1=2 not work?

x = 1
x2 = 1
x = x2
1 = 2x (derivitive)
1 = 2(1)
1 = 2 ???

please tell me where the false logic is.


Answered by Walter Whiteley.
Order of operations 2000-09-19
From Nicole:
the problem is, 4x8-18 divided by 6= do you solve this by doing 4x8=32 then by dividing 18by 6= 3, then subtracting 3 from 32 = 29? If not what is the order of operation?
Answered by Penny Nom.
Two linear equations 2000-09-14
From David Dean:
2a + 1b = 3.39 3a + 3b = 6.59
What formula do I use to find what a = ?

Answered by Harley Weston.
Induction 2000-09-07
From Joe Peterson:
How do I prove by the principal of mathematical induction?
1.n+2.(n-1)+3.(n-2)+.....+(n-2).3+(n-1).2+n.1=(n(n+1)(n+2))/6

Answered by Paul Betts.
Velocity of a pendulum 2000-08-28
From Mekca:
A pendulum hangs from the ceiling. as the pendulum swings, its distance,d cm, form one wall of the room depends on the number of seconds,t, since it was set in motion. assume that the equation for d as a function of t is: d=80+30cos3.14/3t, t>0.

estimate the instantaneous rate of change of d at t=5 by finding the average rates for t=5 to 5.1, t=5 to 5.01, and t=5 to 5.001.


Answered by Harley Weston.
The sum of the squares of 13 consecutive positive integers 2000-08-25
From Wallace:
Prove that it is not possible to have the sum of the squares of 13 consecutive positive integers be a square.
Answered by Harley Weston.
The angle of rotation 2000-08-03
From Jay:
I have the following information Given.

(X1, Y1) Origional Point
(X2, Y2) Origional Point After a Rotation
(Xa, Xb) Center of Rotation

What formula would I use to figure out the angle the point was rotated?


Answered by Chris Fisher and Harley Weston.
Two linear systems 2000-08-01
From A student:
please help me with this homework question. 5/x + 6/y= 19/6 3/x + 4/y =2
hint: let a = 1/x and b = 1/y substitute these expressions into the system to find a and b. Then find x and y.

This is a question I answered wrong on a test: solve the given system for x, y, and z. Express your solutions in terms of a, b, and c.


Answered by Penny Nom.
An equation involving x to the x 2000-07-22
From Joy Peter:
I am joy, a teacher teaching Maths at the secondary level and while solving a sum came to this stage when i got xx (1 + log x) = 0, by which we can conclude that xx = 0 or 1 + log x = 0. If xx = 0, than what should be the value of x? I feel that the value of x should then be 0 (zero) but then how do I explain this to the students as we also know that anything to the power of 0 is 1 but here 0 raised to 0 is 1. If this is not defined then how do I explain this?
Answered by Penny Nom.
Parallel tangents 2000-06-30
From Ebony Indalecio:
I need to prove the theroem: Tangents to a circle at the end points of a diameter are parallel.
Answered by Walter Whiteley.
Central Limit Theorem and Law of Large Numbers 2000-06-26
From Jonathan Yam:
The Central limit Theorem states that when sample size tends to infinity, the sample mean will be normally distributed. The Law of Large Number states that when sample size tends to infinity, the sample mean equals to population mean. Is the two statements contradictory?
Answered by Paul Betts and Harley Weston.
The speed of light 2000-06-19
From Benny Lau:
I have a personal questions. In physics,if a person is running on a train, his relative speed on the ground will be the addition of the speed of train and the person. If u have a train inside another train and inside another train, so on...... can u reach the speed of light? It would need a very long distance. But if we make all the train travel in circles. Then we don't need that much distance. So can we achieve the speed of Light?
Answered by Robert Coleman and Harley Weston.
Powers of i 2000-05-24
From Paul Fieldhouse :
What is the result of raising i to the googol power? is there a rule or pattern to raising i by increasing powers of 10?
Answered by Penny Nom.
The equation of a parabola 2000-05-22
From Ian Forsyth:
Given the points A(0,0) B(60,10) C(24,d) find the equation of the parabola. leave the equation in terms of x, y and d if the general form of a quadratic is y = ax2 + bx + c.
Answered by Penny Nom.
Calculus Research Questions 2000-05-22
From William Wright:
I am a Calculus Teacher, and me and my class ran into these two problems without solutions in my manual, we got answers, but are unable to check them. If anyone gets this email and can respond to this with the solutions it be greatly appreciated.
.
.
.

Answered by Harley Weston.
The square root of -1 2000-05-19
From Gary:
i am not a student i am just some one that heard something and i can't be sure on the answer...my ? is what is the square root of -1? i think it is -1 but not sure can you let me know please thank you
Answered by Harley Weston.
A matrix equation 2000-05-14
From A student:
Right now, we are dealing with matrices and we are supposed to solve the following problem on our graphing-calculators: 2a+3b-4c+d=20
a-2b+3c-5d=-14
3a+4b-2c+3d=19
5a-b+6c+4d=-5

Answered by Penny Nom.
The Village of Truth 2000-05-10
From Sean Pearson:
Our teacher gave us a logic problem involving two villages: the village of truth, and the village of lies. The problem is in asking two guards the right question to gain entrance to the fenced compound the villages are in without being shot. Have you heard of this problem ?
Answered by Harley Weston.
All the roots of x^6 - 64 2000-04-15
From Dakota:
Find ALL zeros of P(x) = x 6 - 64
Answered by Walter Whiteley.
The square root of 3 2000-04-04
From Mr. William:
Prove that root 3 is irrational
Answered by Harley Weston.
Taxis in Chicago 2000-03-27
From A high school student high school student:
Suppose that taxis pick up and deliver passengers in Chicago, which is divided into three zones. Records kept by the drivers show that of the passengers picked up in Zone 1, 50% are taken to a destination in Zone 1, 40% to Zone 2, and 10% to Zone 3. OF the passengers picked up in Zone 2, 40% go to Zone 1, 30% to Zone 2, and 30% to Zone 3. Of the passengers picked up in Zone 3, 20% go to Zone 1, 60% to Zone 2 and 20% to Zone 3. Suppose that at the beginning of the day, 600 of the taxis are in Zone 1, 100 in Zone 2, and 300 in Zone 3. What is the distribution of taxis in the various zones after all have had two riders?
Answered by Harley Weston.
Rule of 78 2000-03-22
From Dan Baumgarten:
Can you explain the rule of 78 and the reverse rule of 78? Thanks.
Answered by Claude Tardif.
Surface area of a sphere 2000-03-22
From Gina Wilkie:
How can I demonstrate to my middle school students the reason for the formula for the surface area of a sphere?
Answered by Walter Whiteley and Chris Fisher.
A system of equations in five unknowns 2000-03-20
From Will:
I have been having some problem with the following question for some time. I would appreciate any help on solving the problem or a solution.

Q: Assume that a system of equations in the unknowns x1, x2, x3, x4 and x5 when converted to row echelon form gives

.
.
.

Answered by Penny Nom.
Induction 2000-03-16
From William Tsang:
I am trying to prove a induction question

Sigam r=1 n (2r -1)cube = n square (2 n square - 1)


Answered by Harley Weston.
Eligibility in a contest 2000-03-14
From Ken Rabley:
Hi, hoping you can help. Dispute among co-workers. Tell me, what is the correct answer to the following question:
-1936 + (2406/4812) x (4756 - 3256) + 1250

Seems this is the question for eligibility in a contest...We have come up with various solutions, all which may be correct.


Answered by Penny Nom.
Simultaneous equations 2000-03-11
From Laura Molck:
My name is Laura Molck and I am in Year 11 in Australia. Please help me with the following. I know that they are all simultaneous equations which I can do but I have trouble with the formulae to work the equations. Can you please help!!

1. A tent manufacturer produces 2 models, Outback and Bushwalker. From earlier sales records it is known that 20% more of the Outback model is sold than the Bushwalker. A profit of $200 is made on each Outback sold, but $350 is made on each Bushwalker. If during the next year a profit of $177,000 is planned how many of each model must be sold?


Answered by Penny Nom.
Congruent parts of congruent figures 2000-03-10
From Erica:
Yesterday, I recieved a test problem asking to prove two line segments equal. Here is the problem as I was given it:

Given: paralleogram ABCD
AE is perpendicular to DB
CF is perpendicular to DB
Prove: AE is equal to CF

I answered the problem as follows:
.
.
.
Answered by Walter Whiteley.

Systems of linear equations 2000-03-10
From Ann Marie Devereux:
hi there!!, I guess I have a problem!!!
  • 3x+4y=10 (over)
    4x+y=9

  • 2x=5y+3 (over)
    x=3y+1


Answered by Penny Nom.
Angle of Intersection of Two Lines 2000-03-02
From Veronica Patterson:
I am having a real hard time trying to figure out this problem. Could you please help me! The homework question says to find the acute angle of intersection between the two lines y=3x+1 and y=(1/2)x-1. (It also says to use the results of a problem I had already figured out.) That problem was to use information from a picture shown that tan(theta sub1-theta sub2)= ((m sub2- m sub1)/(1+(m sub1 * m sub2))). I used the difference identity of tangent to figure out the answer. Any help on this problem would be greatly appreciated.
Answered by Harley Weston.
Order of operations 2000-02-27
From Mildred Saulsbery:
Find the value of the expression.
 6(5+1) - 9(1+1) _____________    5(8 - 6) -23 

Answered by Harley Weston.
Party favors 2000-02-22
From Krystina Fernandez:
Luanne was making party favors for her little sister's birthday party. Each party favor was in the shape of a cube. Luanne had pink and purple paint to paint the cubes and she could paint each face only solid pink or solid purple (no dots,stripes,ect.).For example, one cube may be all purple, another may have two purple faces and four pink faces. Her little sister wanted to have a different cube for each guest.(A cube is not considered different if it can be turned so that all it's sides match the corresponding sides of another cube.)How many different cubes was it possible for Luanne to make?
Answered by Claude Tardif.
The quotient rule 2000-02-21
From Charlene Anderson:
Question: I came across a question in our book that states: Let Q(x) = N(x) / D(x) Then re-write Q(x) in a form that can utilize the Power and Product Rules. Use this rearranged form to derive the Quotient Rule.

The Quotient Rule can be derived from the Power Rule and the Product Rule.

One must also use the chain rule too, right?


Answered by Harley Weston.
2 = 1 2000-02-16
From Chuck Kennedy:
Question:
  1. Assume a=b
  2. Multiply both sides by a, a2=ab
  3. Subtract b2, a2-b2=ab-b2
  4. Factor (a-b)(a+b)=b(a-b)
  5. Cancel like factors a+b=b
  6. Substitue b for a b+b=b
  7. Then 2b=b
  8. Therefore 2=1
Question; Were is the mistake?

Answered by Claude Tardif.
Play ball 2000-02-03
From Jessie:
Here's a calc question that is probably a lot easier than I am making it. If you have a legendary "baseball problem" for the related rates section of Calc I, and you are given that the runner is running from 2nd to 3rd base at a given rate, and the umpire is standing at home plate, and you are given the distance between the bases on the field, how do you find the rate of change of the angle between the third base line (from the point of the umpire) and the runner? Here is a sample prob: Runner is moving from 2nd to 3rd base at a rate of 24 feet per second. Distance between the bases is 90 feet. What is the rate of change for the angle (theta, as described previously) when the runner is 30 feet from 3rd base?
Answered by Harley Weston.
Names of the polygons 2000-01-29
From Janemath:
I have been doing research on the names of polygons with 11 to 20 sides. I have found conflicting names for some of them . for example 18 sided-octadecagon and octokaidecagon. 11sided-undecagon and hendecagon. Why is this? Is one more correct than another? Or are they equally acceptable?
Answered by Harley Weston.
Order of operations 2000-01-16
From Dorothy:
I was wondering if you could tell me where I can get some info. on how to solve the following problem. I was given the answer but I don't know how it is solved.

4+2x(6x2)-5=23


Answered by Penny Nom.
A roll of paper 2000-01-15
From Richard:
I have a roll of paper, wrapped around a corrugate core, whos diameter is 10.750 in. The outer diameter of the roll is approx. 60 in. The thickness of the paper is .014 in. I am trying to find out how much linear feet of paper is left on the roll, given only the diameter of paper remaining on the core.
Answered by Chris Fisher and Harley Weston.
Factoring ^6 2000-01-03
From Athena:

my name is Athena and I have a question on factoring: how would you figure this out:

(x6-y6) and (x6+y6)


Answered by Penny Nom.
zero 2000-01-01
From Jason:
What civilization first used zero?
Answered by Penny Nom.
Two algebra problems 1999-12-17
From Michael Standfest:
If x+4 is a factor of 2x4+kx3-3kx2+6x-40, find k

and

Prove that n2-n is even for all n, using the proof of contradiction
Answered by Penny Nom.

Sines & cosine laws 1999-12-10
From Pierre Boivin:
Triangle LMN, angle L=71 degree , LM= 7.2 , MN=8.3 , ln= 5.9

The questiion was to find angle M. Using the cosine law I found the answer to be 44 degree. It is also the book answer. Using the sines law I found the answer to be 42.2 degree. why can't I use the sines law.
Answered by Chris Fisher.

Systems of equations 1999-12-06
From Roger Hornbaker:
I am having problem figuring out x and y solutions.
  1. 5x + y = 4
    3x - y = 4

  2. 3x + 2y = 6
    - 3x + y = 0

Answered by Penny Nom.
order 4+ determinants 1999-12-06
From Joe Kron:
Why is it never shown how to calculate the value of 4x4 (or larger size) deteminants by the diagonal multiply methods that are generally shown for 2x2 and 3x3 determinants? The method I'm talking about is called Cramer's Rule??? Is this method not extensible to order 4+ and if not why not? Anyway the method always shown for order 4+ is called "reduction by minors" which is not the answer to this question.
Answered by Walter Whiteley.
How many 3/5 are in 3/4? 1999-12-06
From Whiteny:
How many 3/5 are in 3/4?
Answered by Penny Nom.
Zero to the zero 1999-12-02
From A middle school teacher:
I am a middle school teacher who is looking for a precise explanation of why zero raised to the zero power is undefined. I am hoping to get an explanation using something other than the fact that diividing by zero is undefined.
Answered by Penny Nom.
Estimating the population mean 1999-11-13
From John Barekman:
Statitistics: Estimating the population mean when the standard deviation is known:

I am not sure which n to use in the formula for the confidence interval equation:

x +/- z*(standard deviation/sqrt(n))

If we have data of ten people, and if we have the data of ten sets of ten people each, what is the difference in the n that we use? What is the difference between the standard deviation and the standard error? Are we using the number of sampling means or just the number of samples?
Answered by Harley Weston.

Inverses of functions 1999-11-01
From Leanne Hickey:
Let f(x) = 2x2 -3x + 2. Find f-1(4) given the fact that f(2) = 4. So the question is finding the inverse of 4, he said it's easier than it looks.
Answered by Penny Nom.
Powers of zero 1999-10-31
From Salina Young:
Hello! I have two questions to ask about the power of "0".

First, what is the value of "0' to the power of "0"? And why?

Second, what is the value of "0" to the power of "2" or '3" etc? And why?

Thank you for your help.
Answered by Walter Whiteley and Harley Weston.

Order of operations 1999-10-25
From Garrett:
99*(57+76)*9-1085/9
Answered by Penny Nom.
An odd number of factors 1999-10-22
From Melissa:
What is the common name used for numbers that have an odd number of factors? What is the least positive integer that has exactly 13 factors?
Answered by Penny Nom.
(-5)^2, -5^2 and -(5)^2 1999-10-13
From Jennifer Brown:
What is the difference between the following problems:

(-5)2, -52 and -(5)2

Our text book (Beginning Algebra, fourth edition, published by McGraw Hill, by Streeter, Huthison and Hoetzle) says the second and third problem are exactly the same. I don't see how that can be. Is there a mathematical rule that explains this?
Answered by Penny Nom.

Length of a line 1999-10-10
From Dagmara Sarudi:
My question has to do with the length of a diagonal. This problem came up when I thought about the shortest distance between two points, for example walking from one point to another in my neighborhood. I can choose a zig zag route and assuming the blocks I walk are exactly the same length, it shouldn't matter what route I took, the distance I travel should still be the same when I reached my goal. If, on the other hand I could travel in a diagonal line, the distance would be shorter. But what if, in my zig zag motion, the sections get so small the route approaches a diagonal. Shouldn't it be that each separate section added together equals the value of the two original sides? Or would it suddenly equal the value of the diagonal (which, of course was shorter than the two sides added together)?
What gives?


Answered by Chris Fisher and Harley Weston.

Trigonometry history 1999-09-25
From Nikki:
What is trigonmetry ? Who invented it ? What is it's purpose ? And anything else that you can tell me that is related to Trigometry.
Answered by Chris Fisher.
-8*-(-5a)=15 1999-09-22
From Shelby:
-8*-(-5a)=15

i cannot get this question!! please help me!!
Answered by Harley Weston.

Surface area of a cone 1999-09-18
From Frothy:
I don't understand how to find the surface area of a cone. The height is 12cm and the radius is 5cm.
Answered by Walter Whiteley.
Bales of hay 1999-09-13
From Ivy:
You are given 5 bales of hay. Two bales are weighed at a time, which equal the following weights:
110, 112, 113,114,115,116,117,118,120,121. What does each individual bale weigh?

Answered by Harley Weston.
2,5,8,1 yields 24 1999-09-12
From Terry Hill:
I was wondering how do I make the numbers 2,5,8,and 1 equal 24?
Answered by Penny Nom.
Towers of Hanoi 1999-09-03
From Jeni:
In the puzzle called the Towers of Hanoi there are three peg and you are asked to move the rings from one peg and stack them in order on another peg. You can make as many moves as you want, but each move must consist of moving exactly one ring. Moreover, no ring may be placed on top of a smaller ring. The minimum number of moves required to move n rings is 1 for 1, 3 for 2 rings, 7 for 3 rings, 15 for 4 rings, and 31 for 5 rings. Find a formula for this sequence. What is the minimum number of moves required to move 6 rings?
Answered by Penny Nom.
The sum of the cubes is the square of the sum 1999-08-25
From Bernard Yuen:
How to prove 13 + 23 + 33 + 43 + ... n3 is equal to (1+2+3+...n)2? (for n is positive integer)
Answered by Harley Weston.
Why is slope designated m? 1999-08-18
From Peter Komlos:
Why is the slope of a line is designated by the letter "m"?
Answered by Penny Nom.
From an airport control tower 1999-08-04
From Pammy:
Hi I am a 30 yo mature age student doing my HSC but am having difficulty understanding this, if you can help me.

From an airport control tower, a Cessna bears 023 degrees T and is 27km away. At the same time, a Boeing 767 bears 051 degrees T and is 61km from the tower. Both planes are at the same height.

i) What is the size of angle ATB?

ii) Using the cosine rule to calculate the distance the planes are apart, to nearest kilometre.

I figured out and drew the triangular diagram but can't figure out the rest and which formula to use. sorry about this, thankyou kindly
Answered by Harley Weston.

A difference of squares problem. 1999-07-24
From Michael and Stephanie Bixler:
If you have the equation x= n2 - m2 (ie 40= 72-32= 49-9) x must = a positive number

1) which squared numbers work as n and m
2) how does it work
3) if my teacher gave me the number for x; how could I figure out this problem
Answered by Harley Weston.

An Invalid Argument 1999-05-31
From Rod Redding:
Can an invalid argument have a true conclusion? If yes then why?
Answered by Walter Whiteley.
Introductory Algebra 1999-05-19
From Pat:
(2 + sq. root of 3) x (2 - sq. root of 3) = 1

Please show me the work.
Answered by Harley Weston.

A 1999-05-02
From Leah:
a=b
a^2=ab
a^2+b^2=ab-b^2
(a-b)(a+b)=b(a-b)
a+b=b
b
2=1

why is this proof wrong?
Answered by Penny Nom.

Parallel and perpendicular lines 1999-04-23
From Crystal Pilling:
My name is Crystal Pilling and I am in 9th grade algebra. We are currently studying parallel and perpendicular lines. I am having trouble with this problem: 3/4x - 5y= 16, (5,-6) I have to find a line that is perpindicular to this line on a graph.
HELP ME PLEASE!!!!!

Answered by Penny Nom.
Radius of an arc 1999-04-22
From Rusty Riddleberger:
I need to find the equation for finding the radius of an arc; I know the length of the arc (i.e the distance of the line connecting the two ends of the arc) and the height; (i.e the rise of the arc at its apex,) I had the formula years ago but it has lost me; this would be invaluable for work in new homes i.e. where we need to build an "arch" with a rise of 21" between two columns 11 feet apart
Answered by Chris Fisher.
Radius of convergence 1999-04-21
From Nowl Stave:
Why is the radius of convergence of the first 6 terms of the power series expansion of x^(1/2) centered at 4 less than 6?
Answered by Harley Weston.
The average rate of change of a function 1999-04-20
From Tammy:
Suppose that the average rate of change of a function f over the interval from x=3 to x=3+h is given by 5e^h-4cos(2h). what is f'(3)? I would appreciate any help with this question.
Answered by Harley Weston.
Volume of oil in a tank 1999-04-17
From Lars Waldemarsson:
My problem is to get an equation for the depth of the oil in a gastank formed like a cylinder. The cylinder is in a horizontal position and by a stick you will be able to get the depth of the oil in the tank. All I need is an exmaple which I can build on. By this equation you will be able to get the volume of the oil if you know the depth.
Answered by Harley Weston.
y=mx+b 1999-04-15
From Selena Royle:
When solving a problem on slopes,and in the formula y=mx+b what do the M and B stand for? Why M and B?
Answered by Penny Nom.
Factoring 1999-03-30
From Maggie Stephens:
I don't know anything about factoring would you plese help me.

3x4 - 48

54x6 + 16y3

125-8x3

12x2 - 36x + 27

9 - 81x2

a3 + b3c3

I would greatly appreciate any help you can give me thanks.
Answered by Jack LeSage.

0/11 1999-03-09
From Jacob:
What is the term applied to 0 divided by a number.

In calculating slope problem my answer was 0 divided by 11. I don't know what to do with this answer or how to interpret it.
Answered by Jack LeSage and Penny Nom.

Factoring 1999-03-08
From L. Sivad:
Question:
m2+6m+9-n2

Answered by Penny Nom.
Reversing the direction of an inequality. 1999-03-06
From Mallory White:
If the Problem was -4a plus -5 is less than or equal to 14, why would you change the sign to greater than or equal to?
Answered by Jack LeSage and Harley Weston.
The square root of two is never supposed to end 1999-03-06
From a wondering student:
i am algebra II and am in the 9th grade. today we were talking about rational and irrational numbers. When we were talking about square roots my friend and i were talking and we thought of something. if you have a square with sides of length one then the diagonal of the square is the square root of 2. Now the square root of two is never supposed to end. But the diagonal of the square ends so therefore doesn't the square root of 2 end. our math teacher did not really answer our question because it was not in the lesson plan and not to many people would see where we were coming from. the answer is really bugging me and i would like to have your input.
Answered by Jack LeSage and Penny Nom.
Root 17 is Irrational 1999-01-21
From John Murdock:
If you could help me out with this I would appreciate it. Prove that the square root of 17 is irrational.
Answered by Harley Weston.
Pythagorean theorem research project 1998-12-31
From Mohammed Hasan:
Hi my name is Mohammed Hasan. I am a math honors student in 8th grade. I have to do a research project in math. The only problem is that I have to do the research project at a 10th grade level. I am having trouble raising the project at a tenth grade level. Would you please kindly take your time to give me some tips and web sites that will help me raise the Pythagorean theorem to a 10th grade level.
Answered by Jack LeSage and Walter Whiteley.
Duplation method of multiplication 1998-11-26
From Sara Whitford:
I am wondering why 16 and multiples of 16 do not work in the ancient duplation method of multiplication used by the Egyptians. I discovered the method in the Journeys math text gr 7 level. Just curious. Am I doing something wrong??
Answered by Jack LeSage and Harley Weston.
Linear systems of equations 1998-11-16
From Crystal Girvan:
My name is Crystal Girvan.Im in grade 11. I have a question.We are solving linear systems of equations: comparison method. My questions is. it says

a)use the method of comparison to solve this system a=b-1, 3a+ b=3

b) verify your solutin in a

I dont understand it. please help
Answered by Jack LeSage.

A Sum or Product of Fractions 1998-10-06
From Emily Robb:
Find a pair of fractions that when they are multiplyed and added the sum /product is the same
Answered by Penny Nom.
Radicals 1998-09-15
From Lana Sabo:
Question:
fifteen times the square root of twenty, divided by the square root of 2.

nine subtract the square root of forty-five, divided by 3.

the square root of 18 plus the square root of 12, divided by the square root of 3.
Answered by Harley Weston.

Volumes 1998-08-29
From Lorraine Wall:
Consider the region in the first quadrant bounded by the x and y axes, the vertical line x=3 and the curve y = 1 / (x squared + 3). Determine the volume of the solid by rotating this region about the x-axis. Now that is the first part.
I then have to find the coordinates of the centroid of the solid by rotating this region about the x-axis.

Thanks.
Lorraine
Answered by Harley Weston.

Volumes of Revolution 1998-07-24
From Lorraine Wall:
I'm on the section fpr The Computation of Volumes of Solids of Revolution and the following question is giving me problems:

-Consider the region in the first quadrant bounded by the x-and y-axes, the vertical line x=3, and the curve y=1/(xsquared + 3) I can determine the volume of the solid by rotating the region about the y-axis using the shell method but I can't seem to be able to get started with the volume when rotated about the x-axis.
Answered by Harley Weston.

Difference of squares 1998-06-23
From Kristen Smelsky:
Solve the following using a difference of squares:

4x(squared) minus 4xy plus y(squared) minus m(squared) plus 2m minus 1
Answered by Penny Nom.

Graph question 1998-05-12
From Rose Seminary:
Why is the point of intersection of two lines the solution to the corresponding system of equations?
Answered by Penny Nom.
The day of the week, July 24, 1837 1998-03-21
From Colleen K.:
What day of the week (Mon. Tues. Wed. etc...) was July 24th, 1837?
Answered by Chris Fisher.
(50^20)(20^50) 1998-02-24
From fion:
50 power of 20 X 20 power of 50?

How many zero can be found in the answer and why?
Answered by Penny Nom.

Intersecting Lines. 1998-02-10
From Erik Heppler:
My typewriter frequently makes mistakes. Suppose I gave you the following system of equations to solve:

35.26X + 14.95Y = 28.35
187.3X + 79.43Y = 83.29

When I looked back, I realized that I meant to use 14.96 as the "Y" coefficient in the first equation instead of 14.95. Solving the system using 14.95 and then solving another system using 14.96 with all other values the same both times results in the intersection points (1776, -4186) and (-770, 1816) respectively.
How can that be?
Answered by Penny nom.

Cos(x) Cos(2x) Cos(4x)=1/8 1997-09-24
From Tan Wang:
How many distinct acute angles x are there for which cosx cos2x cos4x=1/8?
Answered by Chris Fisher Harley Weston and Haragauri Gupta.
The Range of a Function. 1997-09-12
From Karel Marek:
Math Is Book 6 (Ebos/Tuck) question 5(b) gives a surprising answer at the back of the book..

The question is:
Indicate the domain and range for this: y = sqr(x-9) (Square root of)

The answer for the domain is x>=9 which is not surprisingly, but the range is y>=0, yER ... which I could understand if you were not allowed to transform the equation into x as the subject..

But all example on the facing page DO TRANSFORM the equation at will with x and/or y as the subject..

Can you explain this...
??
Answered by Chris Fisher.

Proofs 1997-04-13
From Daniel:
I'm having trouble understanding proofs. I don't know how to come up the answers on my own. I search through the book looking for the answer. I understand what they are doing, but I don't know how to do it.
Answered by Walter Whiteley.
A Presidential Proof 1997-03-18
From Greg Smith:
Which US president developed a proof for the Pythagorean Theorem?

Where can a copy of the proof be located?
Answered by Chris Fisher and Harley Weston.

Ajax, Beverley, Canton and Dilltown 1997-03-14
From S. Johnson:
The following towns are placed on a coordinate system. Ajax at (-x,z), Dilltown at (-10,0), Canton at (0,0) and Beverly at (0,10). The roads from Beverly to Canton and from Canton to Dilltown are perpendiculat to each other and are each 10 miles in length. A car traveling at all times at a constant rate, would take 30 minutes to travel straight from Ajax to Canton, 35 minutes to travel from Ajax to Canton via Beverly, and 40 minutes to travel from Ajax to Canton via Dilltown. What is the constant rate of the car, to the nearest tenth of a mile per hour.
Answered by Chris Fisher and Harley Weston.
Calcul dun profil fictif 2005-09-29
From Mr madry loïc:
je suis actuellement en formation pour devenir dessinateur projeteur en BTP ; je rencontre un problème vis à vis d'une égalité qui normalement devrait mettre acquise ( elle doit être issue de thalès ou de l'égalité des proportion ?) cependant, je n'arrive pas trouver le fondement de cette égalité. pourriez vous m'aider à éclaircir cette relation fondamentale afin de poursuivre mon étude sur le calcul de profil en travers (pour les travaux publics) .
Answered by Claude Tardif.
Opérations de base 2003-07-05
From Anne-Marie:
Je m'appelle Anne-Marie et je suis enseignante de 5e année à l'école Domaine des copains de Balmoral au Nouveau-Brunswick. Récemment, j'ai participé à un concours intitulé: Budweiser King Pack Nascar Hat Promo. Afin de réclamer le prix, qui est une casquette, je devais répondre à une question réglementaire. Il s'agissait d'une phrase mathématique qui se présentait comme suit:

4 x 26 - 24 / 8 = qui se lit: 4 fois 26 moins 24 divisé
par 8

Answered by Claude Tardif.
A/B = C/D 2002-03-06
From Un eleve:
Démontrer que si A sur B et = à C sur D, alors AxD et = à BxC.
Answered by Claude tardin.
Resolution of the equation f(x)=0 2000-04-01
From Naoufal:
How can i evalf the equation f(x)=0 with f function . Can you ansear me in french thank you very much
Answered by Claude Tardif.
Le salinon d'Archimèdre 1999-03-11
From Don Craig:
I am trying to find the English translation of "Le salinon d'Archimèdre" and would appreciate any help. This is a figure, presumably studied by Archimedes, created from 4 semi-circles. Since I can't draw it for you, I will try to describe it with the help of the 5 collinear, horizontal points below.

. . . . . A B C D E 

A semi-circle is constructed on AE as diameter (let's say above AE).

Two more semi-circles are then constructed with diameters AB and DE on the same side of the line AE as the first semi-circle (above it). Finally, a fourth semi-circle is constructed on diameter BD, this time on the opposite side of the line AE from the others (i.e. below the line).

These semi-circles and the region enclosed by them constitute what is called in French "Le salinon d'Archimèdre".

If you know the English name of this curve I would appreciate it if you let me know.
Answered by Harley Weston.

 
Page
1/1

 

 


Math Central is supported by the University of Regina and The Pacific Institute for the Mathematical Sciences.

CMS
.

 

Home Resource Room Home Resource Room Quandaries and Queries Mathematics with a Human Face About Math Central Problem of the Month Math Beyond School Outreach Activities Teacher's Bulletin Board Canadian Mathematical Society University of Regina PIMS